Collected PathPhys Situations 2010

148
1 Russian Federation Ministery of Health State Educational Board on High Professional Education Sechenov’s Moscow Medical Academy Department of Pathophysiology Litvitsky P.F., Pirozhkov S.V. COLLECTED CLINICO-PATHOPHYSIOLOGICAL SITUATIONS AND CLINICO-LABORATORY PROBLEMS Moscow 2010

description

pathophysiology

Transcript of Collected PathPhys Situations 2010

Page 1: Collected PathPhys Situations 2010

1

Russian Federation Ministery of Health

State Educational Board on High Professional Education

Sechenov’s Moscow Medical Academy

Department of Pathophysiology

Litvitsky P.F., Pirozhkov S.V.

COLLECTED CLINICO-PATHOPHYSIOLOGICAL

SITUATIONS AND CLINICO-LABORATORY

PROBLEMS

Moscow 2010

Page 2: Collected PathPhys Situations 2010

2

Editors: LITVITSKY PIOTR FRANTSEVICH – Head of the Department of Pathophysiology of the Sechenov's Moscow Medical Academy, correspondent member of the Russian Academy of Medical Sciences, academician of the International High School Academy of Sciences, professor, M.D. PIROZHKOV SERGEI VICTOROVICH - professor of the Department of Pathophysiology, M.D.

This textbook is prepared for students and teachers of the English-speaking Division of the Department of Pathophysiology.

The textbook for students studying the basic course of pathophysiology in English contains clinico-pathophysiological situational and clinico-laboratory problems which are recommended for discussion during practical classes; also included are questions to prepare for classes, control tests and examination. The order in which the problems are placed in the textbook agrees with the plan of lectures and classes on pathophysiology for the third-year students. Versions of solutions of typical problems are presented.

Составители: ЛИТВИЦКИЙ ПЕТР ФРАНЦЕВИЧ – заведующий кафедрой патофизиологии Московской медицинской академии им. И.М.Сеченова, член-корреспондент РАМН, академик МАН ВШ, профессор, доктор медицинских наук. ПИРОЖКОВ СЕРГЕЙ ВИКТОРОВИЧ - профессор кафедры патофизиологии, доктор медицинских наук.

Учебное пособие предназначено для студентов, изучающих патофизиологию на английском языке.

В учебном пособии для студентов, изучающих базовый учебный курс "Патофизиология" на английском языке, содержатся клинико-патофизиологические ситуационные и клинико-лабораторные задачи, которые рекомендуется обсудить во время занятий, а также вопросы для подготовки к занятиям, контрольным работам и экзамену по патофизиологии. Порядок следования тематики задач соответствует тематическому плану лекций и занятий по патофизиологии на третьем курсе обучения. Представлены варианты ответов на типовые задачи.

Московская медицинская академия им. И.М.Сеченова, 2010

Кафедра патофизиологии ММА им. И.М.Сеченова, 2010

Page 3: Collected PathPhys Situations 2010

3

PREFACE Collection of clinico-pathophysiological situational and clinico-laboratory

problems is a constituent part of the educational methodical complex in

English, which also includes manual "Concise lectures and tests on

pathophysiology". - P.F.Litvitsky, S.V.Pirozhkov, E.B.Tezikov. All the

materials are prepared by the collaborators of the Pathophysiology

Department of the Sechenov’s Moscow Medical Academy in consideration

with the State educational standard of higher professional education (year

2000) qualificational (professional) characteristics of medical institutes'

graduates, modern Programmes on pathophysiology (year 2003) and clinical

pathophysiology (year 2002), candidates degree minimum examination on

speciality in "Pathophysiology" 14 00 16 (1997), confirmed by the Ministry of

Education of the Russian Federation and by the Ministry of Public Health of

the Russian Federation.

The aim of the collection of problems is to provide students with an

educational material to develop skill, as well as to improve the obtained

ability of pathophysiological analysis of real clinical situations.

Conducting of pathophysiological analysis is a model of the physician's

behavior with a patient (construction of the scheme of diagnostical search for

causes and risk factors of disease, mechanisms of its development,

substantiation of theraputical and prophylactical measures).

Pathophysiological analysis and formulation of medical comment on

specific data about the patient are a version of highly delicate and peculiar

phenomenon - intellectual modelling of physician's actions during solution of

professional problems.

Collection of situational and clinico-laboratory problems is intended for

three categories of English-speaking students: 1) the 3rd year students,

assimilating basic knowledge of pathophysiology; 2) students in their last year

taking a course of studies in clinical pathophysiology (dedicated to the most

common "typical" clinical syndromes, nosological forms, and emergency

conditions); 3) physicians specializing in specific branches of medical science

during internship, residency, or research post-graduate students within the

frameworks of special educational course in clinical pathophysiology

Page 4: Collected PathPhys Situations 2010

4

(dedicated to analysys of the most frequently observed diseases and

diseaseous conditions of humans).

Scheme of work over each problem presupposes pathophysiological

analysys of the data on patient with determination of the most probable cause

and conditions evoking illnes or pathological process, stages of their

pathogenesis, and mechanisms of the symptomes' development. This stage

of solving the problem shapes a pattern of one of the most important

physician's action – proper diagnosis and its substantiation.

Next stage involves formulation (if necessary) and substantiation of

etiotropic, pathogenetic and symptomatic therapy, and also prophylaxis of

illness and diseaseous state. Such (or similar) scheme of work over problems

is conditioned by model questions formulated after description of each

situation. They could be changed or added (by professor or partner on

situational game) with a purpose of problem's simplification or complication.

Situational problems could be used either as typical, standard ones or as

non-typical ones demanding ability to apply knowledge under new non-

standard conditions, or as unforeseen situations requiring creative approach

to their solution. The above approach will allow to achieve the main aim of

educational course of pathophysiology at medical high school - to form the

grounds of rational physician’s thinking, as well as effective performance of

the future doctor.

Examples of conduction of pathophysiological analysis and formulation

of conclusions for some problems (marked by “starlet”) are given at the end

of the collection. But sometimes other variations of problems solution are

possible (depending on additional questions being put by professor or by the

collegue on the situational "game").

"Concise lectures and tests on pathophysiology" (Moscow 2002) and the

manual of "Pathophysiology"(P.F.Litvitsky. - M. - GEOTAR -MED. - 2002)

serve as source of information, which is necessary and minimally sufficient

for analyzing and formulating conclusions on problems and also for the

answers with regard to tests. Besides that, other modern manuals, quides

and monographes can be used.

Authors of the edition would be grateful for useful advice, suggestion, or

recommendations.

Page 5: Collected PathPhys Situations 2010

5

Head of the Pathophysiology department of

Sechenov's Moscow Medical Academy,

correspondent member of the Russian Academy of

Medical Sciences, professor P.F.Litvitsky

Page 6: Collected PathPhys Situations 2010

6

CONTENTS

Page: I. CLINICO-PATHOPHYSIOLOGICAL SITUATIONS AND

QUESTIONS TO PREPARE FOR CLASSES:

1. Nosology. 8

2. Cell injury and cell death. 11

3. Hereditary disorders. 14

4. Disorders of regional circulation and microcirculation. 18

5. Inflammation. 22

6. Disorders of the host defence system 26

7. Disorders of thermoregulation. Fever. Infectious process. 34

8. Hypoxia. 36

9. Derangements of carbohydrate metabolism. 42

10. Derangements of lipid metabolism. Atherosclerosis. 45

11. Disorders of water exchange. Edema. 48

12. Disturbances of acid-base balance. 51

13. Abnormal tissue growth. Tumors. 52

14. Emergency states. 55

15. Pathophysiology of the red blood cell system. 56

Erythrocytosis. Anemia.

16. Pathophysiology of the white blood cell system. 61

Leukocytosis and leukopenia.

17. Malignancies of hematopoietic cells. Leukemia. 66

18. Pathophysiology of hemostasis. 69

19. Myocardial ischemia (coronary insufficiency). Arrhythmia. 74

20. Heart failure. 79

21. Disorders of the systemic blood pressure. 82

Hypertension and hypotension.

22. Pathophysiology of respiratory system. 87

Page 7: Collected PathPhys Situations 2010

7

23. Pathophysiology of digestion. 93

24. Pathophysiology of the liver. Jaundice. 96

25. Pathophysiology of kidneys. 98

26. Pathophysiology of the pituitary, adrenals and sex glands. 103

27. Pathophysiology of thyroid gland. 108

28. Pathophysiology of the nervous system. Disorders of 114

locomotion and sensation.

29. Neuroses. 117

II. CLINICO-LABORATORY PROBLEMS 119

III. EXAMPLES OF CASES’ DISCUSSION 139

Page 8: Collected PathPhys Situations 2010

8

I. CLINICO-PATHOPHYSIOLOGICAL SITUATIONS AND QUESTIONS TO PREPARE FOR CLASSES: NOSOLOGY Questions to prepare for classes and examination:

1. What is the specific subject matter of pathophysiology? What features

distinguish pathophysiology from other medical sciences?

2. Methods of pathophysiology. Modeling of pathological processes and

diseases. Means and limitations of experimental pahology.

3. The notion of "nosology".

4. What parts nosology is divided into?

5. The hallmarks of the notions of "well-being" and "disease".

6. The notions of "pathologic process", pathologic state", "pathologic

reactions", "typical pathologic processes", "remission",

“recurrence","complication".

7. Principles used in classification of diseases.

8. Factors of diseases: environmental, genetic, social.

9. The relation between causes and conditions in the development of

diseases.

10. The nature and characteristics of pathogenic factors.

11. Reactivity of the body and its role in the development of disease.

12. The notions of "pathogenesis", "vicious circles".

13. Stages of disease.

14. Adaptive reactions and their role in resuscitation and pathology.

N 1

A mouse is put in a small pressure chamber. Then, during 2-3 min

air is pumped out of it to decrease the pressure in the chamber down

to 170-180 mm Hg. After 0.5-1 min in the hypobaric conditions the

mouse shows signs of anxiety: running over, scratching its face. Two

or three minutes later clonic and tonic seizures develop; the mouse

Page 9: Collected PathPhys Situations 2010

9

is lying on its side, urinates; terminal breathing ("gasping")

develops. Soon afterwards, breathing stops, and the mouse dies. The

overall time of the mouse's life in the hypobaric atmosphere is 3-4

min.

Questions:

1. What factors affect the mouse in this experiment?

2. Which of these factors could be considered the leading cause of

the pathologic process - hypobaric hypoxia?

3. How can you experimentally test your presumptions about the nature

of the pathologic process?

N 2

Air is pumped out of the pressure chamber to bring the barometric

pressure to approximately 20-30 mmHg. Then the chamber is filled

with pure O2 up to normal atmospheric pressure 750 mm Hg. A mouse

is carefully put in the chamber so that the barometric pressure

inside remains unchanged. The mouse is then observed as in the

previous experiment. At first, seeking behavior is seen. Later the

mouse is sitting quietly, showing no signs of pathology. Within

approximately 10 minutes after the start of the experiment the mouse

is taken out of the chamber, and its normal behavior and condition is

established.

Questions:

1. Taking into account the results of this test what can you say

about the pathologic factors which caused death of the mouse in the

previous experiment?

N 3

A pressure chamber is filled with a mixture of gases composed

of 95% N2 and of 5% O2. Barometric pressure inside the chamber

is maintained equal to normal atmospheric pressure. A mouse is put

in the chamber. A thin jet of the same gas mixture is being passed

through the chamber. Partial pressure of O2 in this gas mixture is

about 37 mm Hg. After staying in the chamber for 10-15 min the mouse

Page 10: Collected PathPhys Situations 2010

10

develops seizures and then dies.

Considering the results of all 3 experiments, answer the

following questions:

1. Which factor caused the development of acute hypobaric hypoxia

and death of the mouse?

2. What is the significance of hypobaria itself, that is, decreased

pressure of the inhaled air?

N 4

Three mice are used in the following experiment. Mouse 1 is put

in the water basin with water temperature 30oC. Five minutes

later mouse 2 is put in the same basin. Ten seconds later both

mice are taken out of the water and put in the pressure chamber. The

third mouse (intact) is also put in the same chamber. The air is

pumped out as in the experiment N 1 to bring the barometric pressure

in the chamber down to 170-180 mm Hg. The intact mouse dies after

3-4 min of staying in the hypobaric conditions. Mouse 2 dies after

6-7 min, and the mouse 1 survives 15 min of hypoxia. At this point

the experiment is stopped. When mouse 1 is taken out no signs of

pathology are observed.

Questions:

1. What differences in resistance to hypoxia are seen in this

experiment?

2. How can we explain the observed variations in sensitivity to

hypobaric hypoxia?

3. Does hypothermia caused by evaporation of water from the surface of the

body affect the resistance of mice 1 and 2 to hypobaric

hypoxia? How can we name this factor in terms of pathophysiology?

N 5

Three mice are used in the following experiment. Mouse N 1 is

narcotized by a subcutaneous injection of morphine. This mouse is

used in the experiment after it loses righting reflex indicating

deep sleep. Mouse 2 is administered a central nervous system

Page 11: Collected PathPhys Situations 2010

11

stimulator phenamine. Mouse 3 serves as control. All three mice are

put in the pressure chamber, and the air is pumped out as in the

experiment N 1. Mouse 2 dies during the first 1-2 min of its staying in

the chamber, mouse 3 - during the 4th min; and mouse 3 survives 15

min in hypobaric conditions. At this point the last mouse is taken out

of the chamber and examined after it has waken up. It shows no signs

of pathology.

Questions:

1. What are possible mechanisms of differences in reactions of the mice to

hypobaric hypoxia?

2. How can we modify reactivity of the body toward hypobaric hypoxia?

CELL INJURY AND CELL DEATH Questions to prepare for classes and examination: 1. The notion of cell injury. Causes of cell injury.

2. Typical forms of cell injury.

a) Dystrophy.

b) Dysplasia.

c) Derangement of the subcellular structures and components.

d) Necrosis, apoptosis

3. General mechanisms of cell injury:

a) Derangements of the energy supply and utilization.

b) Loss of the integritiy of the cellular membranes.

c) Ionic and water dysbalance.

d) Changes in the cellular genome or disorders of its realization.

e) Disorders of intracellular regulatory mechanisms.

4. Three main etiopathogenic variants of cell injury.

I. Ischemic and hypoxic injury.

II. Free radical-induced injury.

III. Toxic injury.

5. Reversible and irreversible ischemic/hypoxic cell injury.

6. Ischemia/reperfusion injury.

7. Antioxidative system of the cell.

8. Components of apoptosis.

Page 12: Collected PathPhys Situations 2010

12

a) Signaling pathways that initiate apoptosis.

b) Control and integration.

c) Execution phase.

d) Removal of dead cells by phagocytosis.

9. Adaptive mechanisms operating during cell injury.

N 6

A technician of the chemical laboratory was not careful working with a

toxic volatile chemical. He dropped a flask containing the toxic substance, and

smashed it. Before he left the room he had inhaled the noxious vapours of the

chemical. Two days later he was admitted to hospital with the following

complaints: malaise, somnolence, headache, nausea, back pains, blood

in the urine. Blood analysis: erythrocytes 2.7*1012/L, Hb 80 g/L, platelets

120*109/L, leukocytes 3.1*109/L; compensated acidosis (metabolic and renal).

The resuls of the special blood biochemistry analysis: increased

concentration of free fatty acid, lipid hydroperoxides, and

adenosinephosphate; elevated total creatinephosphate kinase (CPK)

activity and potassium content.

Questions:

1. One of the consequences of the patient's poisoning is a significant

depletion of cells in the peripheral blood. What are the possible mechanisms

of this effect?

2. Given the results of the special blood tests what mechanisms of the cell

injury can you propose? Which of them caused damage to the membrane of

the formal particles? Explain your point of view.

3. What are the origin and consequences of acidosis in this case?

4. Why did the signs of intoxication develop in the patient not immediately but

2 days after the incident?

N 7

In two monozygous sibs (8 months of age) an examination revealed

a significant increase in the liver size (hepatomegaly), decreased

fasting level of blood glucose (hypoglycemia), and lack of fasting

blood glucose changes after the adrenaline administration. Liver

Page 13: Collected PathPhys Situations 2010

13

biopsy investigation showed an increased content of glycogen and a

significantly diminished activity of glycogen phosphorylase in the

hepatocytes.

Questions:

1. Give the definition of the pathologic process developed in the

infants?

2. What are the possible causes of this process?

3. Explain the sequence of events leading to glycogen accumulation

in the liver cells in this form of pathology.

4. Explain the association, if any, between the low activity of

glycogen phosphorylase in the patients' liver and a) an increased

content of glycogen in the hepatocytes; b) hepatomegaly; c)

decreased fasting blood glucose levels; d) lack of hyperglycemic

effect of adrenaline.

N 8

Two series of experiments were performed with an aim to determine the

effect of a new antibiotic drug on the skin epithelium. In in vivo studies using

rats the solution of the tested drug was applied to the skin surface. The

employed dose of the antibiotic was much higher than the therapeutic one.

The results of this experiment were assessed by in vivo microscopy studies

which were performed during the first 24 hours after the end of drug

application with 8-hour intervals. In the in vitro experiments the tested drug

was added to the suspension of epithelial cells isolated from the same stock

of rats. Six hours later the cells were washed out by repetitive centrifugation

and suspending in the same media containing no tested drug. The effects of

the antiobiotic were studied at the same time points using regular light

microscopy and electron microscopy.

The results of the experiments:

1. In vivo: 8 hours after the completion of the drug application the

morphological analysis of the epithelial cells revealed the signs of dystrophy

and focal necrosis; the extent of pathologic alterations increased by the end of

the 24 hour observation period.

Page 14: Collected PathPhys Situations 2010

14

2. In vitro: no signs of the cellular or subsellular damage were seen in the

isolated epithelial cells after the drug treatment. The only changes observed

included a reversible cell aggregation found in the first test (8 hours after the

drug withdrawal), but not in analyses that followed.

Questions:

1. How can you explain the difference in results of the in vivo and in vitro

experiments?

2. Was damage to the epithelial cells in the in vivo test direct or indirect?

Explain your point of view.

3. What are the possible mechanisms of the injurious action of the drug on the

epithelium?

4. Taking into account the results of the in vitro experiments, which of these

mechanisms are the most likely?

5. What processes or cell functions should be studied in the further in vivo

experiments to confirm your presumptions about the mechanisms of the

pathogenic effect of the tested drug?

HEREDITARY DISEASES Questions to prepare for classes and examination:

1. The concept of hereditary and congenital disorders, phenocopies.

Classification of genetic disorders.

2. Classification of mutations and mutagens.

3. Mendelian disorders: definition and patterns of inheritance.

4. Codominance, pleiotropism, genetic heterogeneity.

5. Definition of penetrance and expressivity. Factors determining

penetrance and expressivity.

7. Features of autosomal dominant disorders. Examples of disorders

transmitted by autosomal dominant trait. 8. Features of autosomal recessive disorders. Examples of disorders

transmitted by autosomal recessive trait.

9. Features of sex chromosome-linked inheritance. Examples of disorders

transmitted by X-linked trait.

10. Biochemical and molecular basis of single-gene (Mendelian) disorders.

11. Features of multifactorial (polygenic)disorders. The most common

Page 15: Collected PathPhys Situations 2010

15

multifactorial disorders.

12. The concept of chromosomal disorders, their mechanism. Mosaicism.

Types of chromosomal aberrations: deletion, balanced and Robertsonian

translocation, ring chromosome, inversion.

13. Examples of the most common chromosomal disorders. Monosomies and

trisomies.

N 9

N., a healthy woman, visited a genetic counseling unit for consultation.

She told her doctor that her father suffered from color blindness, but her

mother had no vision problems. N. was anxious about the risk of color

blindness for her future children.

Questions:

1. What are the manifestations of color blindness and the pattern of its

inheritance?

2. Can N. carry the gene of color blindness in her genotype?

3. If she can, what is the probability of disease and the probability that

her children (separately boys or girls) will be carriers of the abnormal

gene?

N 10

N., a healthy woman, whose father suffered from hemophilia A and

mother was healthy, went to a genetic counseling unit. She was anxious

about the risk of hemophilia to her grandsons. Her husband, as well as her

son and two daughters were healthy.

Questions:

1. What is the pattern of inheritance of hemophilia A? What are the main

features of this type of inheritance?

2. What is the risk of the disease descending from her son to her grandson,

and from her daughter to the grandchildren?

3. What are the etiology and pathogenesis of hemophilia A?

4. Can this disease have lethal and sublethal forms?

N 11

Sex linked
0%
3/16
Page 16: Collected PathPhys Situations 2010

16

Patients S., a pregnant woman, went to a genetic counseling unit for

consultation. She told her doctor that her sister suffered from

phenylketonuria. In her husband's pedigree there are marriages between

close relatives, but none of the children have had phenylketonuria. A

thorough examination of the patient and her husband revealed no signs of

pathology.

Questions:

1. How great is the risk of phenylketonuria to patient S.'s sons?

2. What are the etiology and basic mechanisms of phenylketonuria? Is

gender significant for its inheritance?

3. What are the main manifestations of the disease and their pathogenesis?

4. What is the approach to early diagnostics of phenylketinuria in the

newborns?

5. Is it possible to prevent phenylpyruvate oligophrenia in children?

N 12

Patient M., male, 21 years old, suffers from hereditary form of

hypogammaglobulinemia. His father also suffers from this disease; mother is

healthy. An examination showed a significant decrease of IgM and IgG levels

in the patient's blood.

Questions:

1. What are the etiology, basic mechanisms and clinical manifestations of

hypogammaglobuliemia?

2. What is the pattern of its inheritance?

3. Characterize the genotype of the patient's mother with respect to this

pathology, and also the possible genotypes of the patient's siblings. How

much is the risk for the patient's sibs to develop hypogammaglobuliemia?

4. Does the pattern of immune and allergic reactions change in

hypogammaglobulinemia? Substantiate your opinion.

N 13

In a genetic counseling unit female patient Z. told the doctor that her

sister suffered from severe sickle cell disease, but she and her husband

were healthy. Z. was concerned about the degree of risk of sickle cell

Sex linked
Yes. Lack of antibody. Autoimmune disease.
Autosomal recessive
Mutation of enzymes
Carrier 50%
1:10'000
Page 17: Collected PathPhys Situations 2010

17

disease to her future children. To answer this question the doctor

investigated the types of Hb in the patient's and her husband's blood. The

test revealed that in the patient's erythrocytes there were 70% of HbA and

28% of HbS; in her husband's erythrocytes there were only 98% of HbA

and no HbS.

Questions:

1. What is the pattern of inheritance of sickle cell disease?

2. What are chances for the patient's children to develop sickle cell

disease? What is the probability that her children would be healthy carriers

of the abnormal HbS gene?

3. Does the development of sickle cell disease depend on the sex?

4. What factors can aggravate the course of the disease?

N 14

Population studies of the frequency of various diseases in the city of N.

among monozygotic (MZ) and dizygotic (DZ) sibs revealed the following:

Frequency in MZ Frequency in DZ

a) schizophrenia 87% 4%

b) scarlet fever 95% 94%

c) poliomyelitis 44% 39%

Questions:

1. What is the value of the Holtzinger factor for these diseases?

2. Estimate the role of genetic and environmental factors in development of

these diseases?

N 15

A male patient, 3 years old, was admitted to a pediatric department.

On examination: signs of growth retardation, flat facial profile, half-open

mouth, oblique palpebral fissures and epicanthic folds are evident;

transverse skin folds on the palms. Study of the patient's karyotype revealed

the following: 46,XY,+t(+14,21).

Questions:

1. What disease does the patient suffer from?

Recessive
Too small to give out disease but a carrier.Prob:0%
Require 2 carrier. Recessive.
No!
Hypoxia. High altitude. Severe physical exercise.Narcosis. Acidosis.
Blood loses
Down syndrome
Translocation from 21 to 14
Page 18: Collected PathPhys Situations 2010

18

2. Describe and characterize the patient's karyotype? By what features does

it differ from the normal one?

3. What are the possible causes and pathogenesis of this disease?

4. Are other variants of karyotype modification possible in this disease?

Which of the variants is the most common?

DISORDERS OF THE REGIONAL CIRCULATION AND MICROCIRCULATION

Questions to prepare for classes and examination:

1. Arterial hyperemia: definition, causes, types (physiologic and pathologic),

mechanisms (neurogenous, neuromyoparalytic and humoral) and

consequences. Reactive hyperemia. Manifestations of arterial hyperemia

and its mechanisms.

2. Venous hyperemia: definition, causes and consequences. Manifestations

of venous hyperemia and their mechanisms.

3. Ischemia: definition, causes and consequences. Manifestations of

ischemia and their mechanisms, factors that influence on the

consequences of ischemia.

4. Stasis: types and causes.

5. Disturbances of microcirculation: types and causes.

6. Types of transmural disturbances of microcirculation, mechanisms of

increased vascular permeability. Mechanisms of extra and intravascular

disturbances.

7. Sludge syndrome: types, causes, and mechanisms.

N 16

A 68-year-old male patient D. presented with chronic hepatitis and liver

cirrhosis. The doctor introduced a needle into the abdominal cavity to

perform aspiration of ascitic fluid. By the fifteenth minute, after 5 L of fluid

had been removed, the patient felt bad, complaining of weakness, dizziness,

and nausea. But the procedure was continued. After 1.5 L more fluid had

been evacuated the patient developed syncope and lost consciousness.

Several minutes later, after the emergency treatment the patient regained

Mozaic???
Nondisjunction
Partial half syndrom
Page 19: Collected PathPhys Situations 2010

19

total consciousness but still complained of weakness, dizziness, and

nausea.

Questions:

1. What was the doctor's mistake during the performance of ascitic

fluid aspiration?

2. What are the mechanisms of syncope after the removal of ascitic fluid?

3. What are the possible mechanisms of adaptation of the brain circulation in

this case?

4. Why did the adaptive mechanisms turn out to be insufficient in this

patient?

N 17

A 61-year-old female patient has been suffering from arterial hypertension

for many years. During the last 2 years she began to feel cold intolerance in

the legs; numbness and pain in the gastrocnemius muscles on walking, and

then at rest. These symptoms were more intense at night and caused sleep

disturbances. Six months ago she developed a skin lesion on the front surface

of her right shin; later it transformed into ulcer. The ulcer was painless but

resistant to therapy. During the visit to the doctor the patient complained of

dryness in the mouth, constant feeling thirsty, frequent urination. On

examination: the skin of the right shin was dry, pale, and cold; no pulsation of

the sole artery was detected. Repeated blood tests showed increased levels

of cholesterol, fibrinogen; high platelet counts; glucose 190-210 mg/dl. The

urine is positive for ketone bodies and glucose.

Questions:

1. What forms of pathology can be observed in the patient?

2. Name the likely type of lesion affecting the patient's arteries? Name and

characterize the main mechanisms of pathology underlying changes in the

patient's arterial wall.

3. What factors caused the development of erosion and ulcer on the patient's

shin?

4. Can you assume the presence of the microcirculatory disorders in the

vascular system of the patient's right leg? What are the likely causes of these

disorders?

Page 20: Collected PathPhys Situations 2010

20

N 18

A 44-year-old male patient K. suffering from atherosclerosis and

coronary heart disease resistant to pharmacologic treatment underwent an

excision of the upper cervical sympathetic ganglion and a dissection of the

sympathetic nerves innervating the heart. Deep thermography of the

myocardium performed during the surgery revealed temperature

normalization in the previously "cold" (compared to the adjacent tissue)

region of the middle one third of the left ventricle anterior wall. The surgery

results in a reduction of frequency and severity of angina pectoris episodes.

Questions:

1. What are the possible mechanisms of the ischemic heart attacks in this

patient? Substantiate your opinion?

2. How can you explain the fact of severe ischemia in only the middle one

third of the left ventricle anterior wall, despite the systemic nature of the

atherosclerotic lesions involving coronary arteries?

3. What caused the decrease in temperature in the "cold" (compared to the

adjacent regions) zone of the patient's heart before the surgery?

4. Can we consider the "sludge phenomenon" as one of the possible

mechanisms of reduced temperature in the ischemic regions of the heart? If

you think so, explain its possible causes and mechanisms in this case.

N 19

A 56-year-old male patient presented with complaints on fatigability and

pains in the gastrocnemius muscles when walking. The symptoms were

relieved by rest. This is called the symptom of "intermittent claudication". In

addition, he complained of increased sensitivity to cold, numbness, pins-and-

needles, tingling sensation in his legs at rest. The patient had a long history

of heavy smoking since his teens. His occupation required working outdoors

even in cold seasons when he sometimes suffered from cold. The patient's

examination showed that skin on both soles was pale and felt cool and dry;

the nails crumbled; no pulse was felt on posterior tibial arteries on both legs.

The preliminary diagnosis was thrombangitis obliterans.

Questions:

Page 21: Collected PathPhys Situations 2010

21

1. What form of organ circulation disorders is observed in the presented

patient? Name its characteristic features.

2. What are the possible causes and mechanisms of this form of the

circulation disorders?

3. What is the possible outcome of the circulation disorder in this case?

4. What are the likely mechanisms of development of each symptoms

seen in the patient?

N 20

An ambulance came 10 min after the vehicle accident, and the doctor

started the emergency aid to the accident victim with a blunt trauma of the

chest and open fracture of the right leg. A tourniquet was applied to the

injured leg to stop severe hemorrhage; then an analgesic (morphine) was

injected subcutaneously into the injured leg to prevent shock. In addition,

the heart stimulating drugs were injected. However, despite the morphine

injection the patient kept groaning of pain and suffered from progressing

shortness of breath. The patient's blood pressure was 60/35 mm Hg, pulse

rate 126/min. Auscultation of the lungs revealed frequent and depressed

breathing on the left side; no sounds were detected on the right side. The

consciousness became obtunded. Cyanosis of the skin and mucosal

membranes was progressing. The doctor subcutaneously injected a drug

stimulating the respiratory center to activate breathing. However, it had no

effect, and the patient lost consciousness.

Questions:

1. What pathologic processes developed in the victim of the accident?

Substantiate your answer by the reported data.

2. Which of the processes is the most pathogenic? Explain its etiology.

3. Describe the main stages of pathogenesis of the mentioned pathologic

process.

4. Why did the injections of morphine and the analeptic drug fail to improve

the patient's condition?

Page 22: Collected PathPhys Situations 2010

22

N 21

A 36-year-old female patient who has been suffering from diabetes

mellitus for more than 13 years presented with visual losses manifested as

blurring of vision; "sand in the eyes" when reading small prints. On

examination: a significant decrease in vision acuity, narrowing of lateral

visual fields in both eyes; irregular thickening of the retinal microvessels wall;

capillary microaneurysms and mural thrombi in the retinal microvessels; focal

serous exudates and hemorrhages in the retina, and its neovascularization.

The doctor informed the patient that her vision impairment was the result

of the diabetic microangiopathy. The latter includes pathologic alteration of

the orbital microvessel wall. The patient received appropriate advice and

treatment.

Questions:

1. What types of microcirculatory disorders can you distinguish in the

patient's retina?

2. What specific types of microcirculatory disorders can arise from:

- capillary microaneurisms;

- irregular microvessels wall thickening;

- mural microthrombi.

Explain their pathogenesis.

3. Which form (or forms) of microcirculation disorders may give rise to focal

retinal edema?

4. Can the microcirculatory changes described produce vision impairment in

the patient? Explain the relevant mechanisms.

INFLAMMATION Questions to prepare for classes and examination:

1. Definition and significance of inflammation. General features of acute and

chronic inflammation. Causes of inflammation.

2. Primary and secondary alteration in the focus of acute inflammation:

causes, mechanisms of development, manifestation. Physico-chemical

changes in the focus of inflammation.

2. Exudation. Definitions of an exudate and transudate.

3. Sequence and mechanisms of vascular changes in acute inflammation.

Page 23: Collected PathPhys Situations 2010

23

4. Mechanisms of vascular leakage in acute inflammation. 5. Stages of extravasation. Mechanisms of margination, rolling, firm

adhesion and transmigration. Inflammatory mediators that influence on

these processes. 6. Chemotaxis: definition, exogenous and endogenous chemoattractants,

regulatory (receptor) mechanisms of chemotaxis.

7. Phagocytosis: stages, major opsonins, opsonic and nonopsonic

phagocytosis, mechanisms of bacterial killing (oxygen free radicals,

products of secondary and lysosomal granules). Regurgitated and

frustrated phagocytosis.

8. Defects in leukocyte function. Defects in leukocyte adhesion and

intracellular digestion.

9. Local and systemic signs of inflammation.

10. Definition of inflammatory mediators and their classification.

11. Sources, nature and effects of inflammatory mediators: vasoactive

amines, complement, kinin, and clotting systems, arachidonic acid

metabolites, platelet-derived factor, cytokines and chemokines, nitric oxide,

lysosomal constituents.

N 22

A 46-year-old patient B. suffers from gastric ulcer. He has been admitted

as an emergency into the surgery department with a preliminary diagnosis of

gastric perforation. On admission: acute pain in the epigastrial region, rigid

abdomen, tenderness, signs of peritoneal irritation; body temperature 38.5oC;

absence of bowel sounds; marked leukocytosis; increased erythrocyte

sedimentation rate. Analysis of the aspirated fluid (400 ml of opalescent fluid

was evacuated during the needle drainage of the abdomen) showed the

presence of a large amount of leukocytes, protein 4%, various types of

bacteria including anaerobic. A plain radiograph of the abdomen

demonstrated the presence of air under the diaphragm. The patient was

transported to the operation room.

Questions:

1. Can you confirm or disprove the preliminary diagnosis of gastric

perforation? Substantiate your conclusion.

Page 24: Collected PathPhys Situations 2010

24

2. Define the type of pathologic process in the patient on admission.

3. What are the causes and mechanisms of each of the symptoms presented

by the patient?

4. Define the type of fluid evacuated from the patient's abdominal cavity.

Describe the characteristic properties of this fluid and explain the mechanisms

of its formation.

N 23

Two industrial workers visited a primary care physician with heat burns of

the shin area. They received them during the boiler breakdown accident. They

presented with a similar set of complaints of headache, pain and swelling of

skin in the injured area. On examination: patient A. had red, hyperemic shins

with marked edema; patient B. had, in addition, blisters filled with a yellowish

transparent fluid. Both patients received treatment recommendations, and

were accorded a sick-leave. However, both of them ignored the doctor's

advice. Three days later the patient A.'s condition improved, and the patient

B.'s condition worsened. The latter developed extensive edema and pain in

the injured areas, his body temperature increased up to 38.90C; the area of

the heat burn was covered with numerous blisters filled with pus. Bacteriologic

analysis of the purulent contents revealed the presence of staphylococci.

Questions:

1. What pathologic process(es) developed in the given patients?

What blood test can confirm the presence of this process in the victims of the

industrial accident?

2. How can you explain the different course of development of the same

pathologic process in the patients?

3. What mechanisms underlie the clinical symptoms in the patient

B.?

4. Why did the noninfectious pathogenic factor (high temperature) cause

the emergence of blisters with pus in the patient B.?

N 24

A 25-year-old patient A. visited gastroenterologist with complaints of acid

belching, discomfort in the epigastrium, sensation of fullness in the stomach

Page 25: Collected PathPhys Situations 2010

25

after meal, and frequent diarrhea. Gastroduodenoscopy and the complete

blood count test with serum protein analysis were performed to specify the

diagnosis. The results of the gastroduodenoscopy: gastric mucosa is swelled

with enlarged, tortuous, closely adjoining folds; sulci between the folds are

deep. Overt signs of arterial and venous hyperemia of mucosa, and focal

erosions are observed. The volume of mucus is increased. In the blood: signs

of anemia and leukocytosis; dysproteinemia due to increase in globulins

fraction.

Questions:

1. What pathologic process(es) can be distinguished in this patient?

Substantiate your opinion.

2. Which of the pathologic processes is primary, and which of them are

secondary?

3. What are the possible causes and mechanisms of symptoms observed in

the patient?

N 25

With an aim to perform a pathophysiologic analysis of factors

determining the unfavorable course of posttraumatic inflammation physicians

of the Traumatology Department studied medical histories of their patients.

All patients were distributed into 3 groups depending on the type of the

observed physiologic dysfunction. The first group included patients who

exhibited signs of liver damage; the second group consisted of patients with

cardiovascular pathology (atherosclerosis, essential hypertension etc.), and

the third group comprised elderly patients with various cerebral disorders

(stroke, concussion of the brain, cerebrovascular atherosclerosis).

Questions:

1. What pathophysiological grounds has distribution of patients into these

three groups? Substantiate your opinion.

2. What other criteria (parameters) could be used to classify patients into

groups for studies of various aspects of inflammation?

DISORDERS OF THE HOST DEFENCE SYSTEM Questions to prepare for classes and examination:

Page 26: Collected PathPhys Situations 2010

26

1. Typical forms of disordered immunologic reactivity.

2. Principles of classification of immunodeficiency states.

3. Primary immunodeficiencies: classification and clinical manifestations.

4. Forms of primary immunodeficiency (causes, mechanisms, clinical

manifestations):

a) B system-dependent:

b) T system-dependent:

c) A-system-dependent;

d) complement-dependent;

e) combined.

5. Secondary (acquired) immunodeficiency states. AIDS

6. Physiologic and pathologic tolerance.

7. Graft-versus-host disease.

8. The notion of the "hypersensitivity reaction", "allergy", "atopic allergy".

9. Types of allergens. Exogenous and endogenous allergens.

Heterologous, homologous and autologous allergens.

10. Stages of allergy.

a) Immunologic stage.

b) Pathochemical stage.

c) Clinical manifestations.

11. Anaphylactic (I) type of allergy. The nature of allergen and antibody.

Mediators of the anaphylactic reaction.

12. Clinical examples of the IgE-medited diseases. Systemic and local

anaphylaxis.

13. Type II hypersensitivity. The nature of allergen and antibody. Mechanisms

and clinical examples of type II allergies. Complement-dependent

reactions and antibody-dependent cell-mediated cytotoxicity.

14. Type III hypersensitivity. The nature of allergen and antibody.

Mechanisms and clinical examples of type III allergies. Systemic and local

immune complex disease.

15. Type IV hypersensitivity. The nature of allergen and antibody.

16. Autoimmune diseases.

17. Principles of diagnostics and therapy of allergic diseases. Desensitization

and nonspecific approach in the treatment of anaphylactic diseases.

Page 27: Collected PathPhys Situations 2010

27

N 26

A patient who had undergone a surgical intervention for gallstones

presented with a slow healing of the postoperative wound, its infection, and

fever. The use of antibiotics (after sensitivity tests) had positive effect.

Investigation of the patient's blood showed leukopenia due to decreased

neutrophils and monocytes, decreased mobility of leukocytes and low activity

of leukocytic myeloperoxidase. Similar abnormalities were found in the

patient's sister and brother.

Questions:

1. Taking into account the observed changes in the leukocyte counts, name

the syndrome(s) which the patient suffers from. Substantiate your answer.

2. What are the possible causes, manifestations, and mechanisms of this

syndrome? Are there grounds for its hereditary or congenital origin?

3. What are the mechanisms of the decreased neutrophil mobility and low

activity of the neutrophil myeloperoxidase?

4. What approaches can be used to increase the antimicrobial resistance of

the patient's body?

N 27

A 20-year-old patient who suffered from diabetes mellitus presented with

slow healing of a surgical wound, purulence, and an increase in the body

temperature up to 37.20C after he had undergone the appendectomy.

Treatment with antiobiotics for 6 days (after the appropriate sensitivity tests)

had no effect. With an aim to find the reason for the treatment failure blood

glucose determination and special neutrophil tests were performed. The

results of the tests are the following: hyperglycemia (320 mg/dl) is

accompanied by a decrease in the number, mobility, and microbicidal activity

of neutrophils.

Questions:

1. What parameters, in your opinion, should be investigated to elucidate

mechanisms of the decreased mobility and microbicidal activity of

neutrophils?

Page 28: Collected PathPhys Situations 2010

28

2. What are the most likely causes and mechanisms of impairment of the

neutrophils’ function in this patient?

3. Is there any association between the abnormal phagocytosis, the infection

of wound and its slow healing? If you think there is, name and characterize

possible mechanisms of such an association.

4. Why is the body temperature in the patient with a purulent wound increased

only moderately?

5. Why did the treatment with antiobiotics in this case have no effect?

N 28

During examination of an 11-year-old patient with albinism of skin and

irides a physician found multiple skin blisters filled with pus and multiple

petechiae. Blood tests revealed the presence of marked anemia,

thrombocytopenia, and neutropenia. Special investigation of neutrophils

showed a decrease in mobility and microbicidal activity, and also the presence

of large granules in the cytoplasm. Hyporesponsiveness of NK cells was also

detected.

Questions:

1. What type of pathology does the patient suffer from?

2. What are the likely causes of abnormal mobility and microbicidal activity of

neutrophils in this patient?

3. What is the cause of this pathology and mechanisms of its main

manifestations: anemia, thrombocytopenia, and leukopenia?

N 29

A 28-year-old patient K. visited his physician with complaints of recurrent

stomatitis, tonsillitis, tracheitis, otitis, and repeated pneumonia even in the

summer time. Procedures aimed at enhancing the nonspecific body

resistance to infection, such as an increase of cold endurance, had no effect.

The results of the laboratory tests are the following: lymphocytes reactivity

toward phytohemagglutinine (PHA) and tuberculin is normal; activity of the

complement factors and levels of IgM, IgG, and IgA in the blood serum are

within the normal range. The complete blood count test with differential

showed no changes in erythrocytes count or Hb concentration; leukocytes

Page 29: Collected PathPhys Situations 2010

29

count is low due to a decrease in monocytes, but the content of granulocytes

is normal. The phagocytic activity of macrophages is decreased by 45%.

Questions:

1. Define the type of pathology found in the patient.

2. What part of the immune surveillance system is impaired in the given

patient: the specific immune response or the nonspecific host defence?

Substantiate your answer.

3. If the specific immunity is abnormal what part of it is defective: A, B, or T?

Explain the origin of symptoms presented by the patient.

4. If the host-defence system is deficient what subsystem is responsible for

the defect? Explain the mechanisms of symptoms seen in the patient.

N 30

An 8-year-old patient was admitted to the Pediatrics Department. His

parents were concerned about the frequent occurrence of otitis, quinsy,

rhinitis, conjunctivitis, bronchitis, pneumonia, and enterocolitis in their child.

The current admission was associated with a high risk of bacterial

endocarditis and sepsis in the young patient.

A test performed in the hospital showed the presence of leukopenia due to

a decrease in lymphocytes, especially in T cells, and, to a lesser degree, in B

cells, a decrease in the blood levels of IgA and IgE (by 40% and 50%

respectively, compared to normal values); concentration of IgG is at the

lowest normal value; proliferation of lymphocytes in the presence of

phytohemagglutinine is decreased.

Questions:

1. Define the type of pathology developed in the patient. Substantiate your

answer.

2. What are the possible causes of this pathologic state?

3. Taking into account the results of the laboratory tests explain the

mechanism of development and consequences of this type of pathology.

4. How can you explain a decrease in the lymphocytes’ reaction to

phytohemagglutinine, and a considerable reduction in the blood content of IgA

and IgE when the levels of IgG are in the normal range.

Page 30: Collected PathPhys Situations 2010

30

5. What symptoms in this child could be a direct result of the IgA and IgE

depletion?

N 31

A 32 year-old male patient visited his dermatologist with complaints of

weakness, rigor, long-lasting sustained fever, and night sweating. About a

month ago he noticed the emergence of pink-violaceous spots and nodules

on his right shin with a tendency to expand and coalesce. The use of

antihistamine drugs and glucocorticoids had no effect on the skin lesions.

On examination: the similar lesions were found in the mouth mucosa

and conjunctiva; the subclavian, supraclavicular, submandibular,

preauricular, retroauricular, and tonsillar lymph nodes were enlarged.

Complete blood count test showed no changes in erythrocyte count, and the

presence of leukopenia due to a decrease in lymphocytes and monocytes.

Over the 3-year period before the onset of the disease the patient had

been working in Africa where he experienced pneumonia three times during

the last year of his business trip.

Questions:

1. What pathologic process(es) developed in the patient? Substantiate your

answer.

2. If you see several pathologic processes which of them is primary, and

which are secondary? Explain their interrelationship.

3. What is the cause, and what are the main stages of development of the

primary pathologic process? Substantiate your opinion using the given data.

4. What are mechanisms of development of leukopenia, lymphopenia, and

monocytopenia? What type of lymphocytes, T or B, contributes more to the

of lymphopenia? Substantiate your answer.

N 32

After spending an hour and a half in the countryside a 30-year-old man

noticed that his eyelids became red and swollen, lacrimation and nasal

secretions appeared, the voice became hoarse. He had a feeling of tightness

in his chest. When he returned home the intensity of these symptoms

decreased, but they were still present.

Page 31: Collected PathPhys Situations 2010

31

Questions:

1. Define the type of pathologic process developed in the patient. Substantiate

your opinion.

2. What methods can be used to determine the cause of this pathologic

process?

3. What are the main components of the mechanism of the given process?

4. What treatment approaches and preventive procedures can be used in

this case?

N 33

A 38-year-old patient K. who had ingested gold-containing drugs

according to doctor's prescription, noticed, first, singular, and then multiple

petechiae on the skin of his forearms, chest, back, and also in the oral

cavity mucosa. Minor contusions were accompanied by extensive

subcutaneous hemorrhages.

Blood tests showed normal count of erythrocytes, leukocytes, and Hb

concentration, but a significant decrease in the platelet count, and an increase

in the IgG and IgM content. Therapeutic procedures used by the doctor

improved the patient's condition, and he continued to take gold-containing

drugs since other forms of medication had poor effect.

Questions:

1. Define the type of pathologic reaction developed in the patient.

Substantiate your answer.

2. What was the cause and mechanisms of this reaction? Substantiate your

opinion by the given data.

3. What procedures could be used to inhibit such pathologic reaction to the

gold-containing drugs?

N 34

A 30-year-old female patient K. visited her dermatologist several times

with complaints on the red, itching spots on the face, neck and hands

appearing in the cold environment, such as windy cold weather. The

physician prescribed her an ointment which alleviated these symptoms but

did not eliminate them. One day, when she was in a hurry to get to her work,

Page 32: Collected PathPhys Situations 2010

32

she washed her face with cold water. Thirty minutes after she had gone

outdoors the sites of the skin exposed to cold water turned red, swelled and

itching. The patient had to visit her doctor and later was admitted to hospital

Questions:

1. What pathologic process(es) developed in the patient after she had

washed her face with cold water? Substantiate your answer.

2. What is the mechanism of this process?

3. What pathologic process should it be distinguished from, and what is the

key difference between these two processes?

4. What groups of medicines can be used to prevent or inhibit this pathologic

process?

N 35

A patient with an open lower extremity trauma was repeatedly injected

tetanus antitoxin in combination with antihystamines. On the ninth day after

the last antiserum injection he presented with high body temperature (up to

380C), severe weakness, swelling and tenderness of the shoulder and knee

joints; intensely pruritic disseminated urticarial eruptions, and the enlarged

tender popliteal and inguinal lymph nodes.

Questions:

1. What form(s) of pathology may be suspected in this patient?

2. What additional data are necessary for a final conclusion about the

form of pathology which the patient suffers from?

3. Taking into account the given data, outline the possible cause and

mechanisms of development of this pathology.

4. What approaches can be used to prevent the development of this

pathologic process?

N 36

On the sixth week of his stay in hospital after extensive myocardial

infarction a successfully recovering patient started to suffer from a dull pain in

the chest which was aggravated by deep breathing movements, swallow, and

changes in the body position. He also presented with fever (body temperature

390C) and a pericardial friction rub. Blood tests showed the presence of

Page 33: Collected PathPhys Situations 2010

33

eosinophilic leukocytosis and an increased titer of the "antimyocardial"

antibodies. A physician made a diagnosis of postmyocardial infarction

syndrome (the Dressler's syndrome).

Questions:

1. Taking into account the immunogenic nature of the Dressler's syndrome,

explain the type and origin of antigens causing this disease.

2. Define the type of a pathologic reaction developed in the patient using the

classification system of Gell and Coombs.

3. Describe the mechanism of this pathologic reaction. To what type of

immunoglobulin do the "antimyocardial" antibodies belong?

4. Prove (or disprove) the fact that the Dressler's syndrome

represents an allergic reaction of the delayed type.

N 37

A patient with an extensive full-thickness burn of the thigh got an i.v.

infusion of blood plasma. Soon after this, he presented with hyperemia of the

face and neck, psychomotor agitation, restlessness, fear of death, an intense

throbbing headache, buzzing in the ears, and nausea. A physician suspected

the development of allergic reaction and gave the patient an injection of an

antihistamine drug. However, the patient's state continued to worsen. He

developed choking sensation, acute systemic hypotension (blood pressure

65/45 mm Hg), confusion, pallor, cold sweat, and finally lost his

consciousness. Later, seizures and spontaneous urination occurred.

Questions:

1. What pathologic process (or reaction) and what type of it developed

in the patient after the infusion of blood plasma? Substantiate your

answer.

2. Name and characterize the main steps of pathogenesis of this pathologic

process.

3. Why did the parenteral injection of the antihistamine drug not improve the

patient's condition? What procedures should be performed to prevent this

pathologic process (reaction)?

Page 34: Collected PathPhys Situations 2010

34

4. What factors caused the development of respiratory, hemodynamic and

psychoneurologic disorders? Name these factors and characterize

mechanisms of their action.

5. What emergency treatment should be employed to help the patient with this

pathologic process (reaction)?

DISORDERS OF THERMOREGULATION. FEVER. INFECTIOUS

PROCESS. Questions to prepare for classes and examination:

1. Definition of fever. Differences between fever and hyperthermia.

2. Types of pyrogens.

3. Pathogenesis of fever. Features of thermoregulation at different stages of

fever.

4. The biological significance of fever.

5. Symptoms accompanying fever, and their mechanisms.

6. Types of fever:

- based on the extent of temperature rise;

- based on the circadian temperature fluctuations.

7. Types of hyperthermia: exertional, nonexertional, drug-induced,

neurogenic, endocrinopathic.

8. Principles of treatment of fever.

9. The concept of infectious process. Three stages of infectious process.

10. Entry and colonization of the host.

11. The host defence mechanisms against the entry of infectious agent.

12. Invasion and growth of parasitic organisms in host tissues.

13. The host response.

14. Pathologic components of infectious process.

N 38

A 39-year-old patient Z. had been surgically treated 8 days before in the

surgical department for perforating duodenal ulcer. In the morning he has

presented with progressive weakness, dizziness, nausea; complained of

fullness in the epigastrium. Body temperature 39.20C. Ultrasonic studies of

the abdomen showed collection of fluid, 4x4x8 cm of size, on the left of the

Page 35: Collected PathPhys Situations 2010

35

umbilicus. Bacteriologic analysis of the fluid, removed by the needle

aspiration, revealed the presence of anaerobic organisms. The patient

received the appropriate antibiotic therapy. Three days later the patient's

body temperature decreased to 37.40C, and persisted at this level.

Questions:

1. What factor caused the increase of the patient’s body temperature 8 days

after the surgery? Substantiate your opinion.

2. What are the specific mechanisms of fever in this case? Use the given data

to substantiate your answer.

3. Why was the administration of antibiotics for 3 days ineffective to bring the

patient's body temperature to normal values?

N 39

A 25-year-old HIV-positive female patient B. was admitted to hospital

because of fever (body temperature 38.90C), cough with rales. She

complained of pain in her right chest during breathing. Laboratory tests

showed leukopenia due to a decrease of lymphocytes and monocytes;

bronchial secretions contained large amounts of desquamating epithelium,

leukocytes, various strains of bacteria; blood was positive for treponema

antigens.

Questions:

1. What were the possible sources of pyrogens in this case? Substantiate

your answer.

2. How can you explain the development of fever in the patient suffering from

leukopenia?

3. Can fever in this case be associated with AIDS? Substantiate your opinion.

N 40

A 29-year-old patient M. has been admitted to the hematologic

department of hospital two weeks after he had started treatment with

cytostatic drugs for chronic myelogenous leukemia. The reason of

admission was worsening of condition and increase in the body

temperature up to 390C. Examination of the patient revealed a moderate

hypochromatic anemia and marked leukopenia. Bacteriologic analysis of

Page 36: Collected PathPhys Situations 2010

36

his biologic fluids showed the absence of pathogenic strains of

microorganisms.

Questions:

1. What type of thermoregulation disorder is observed in this patient?

Substantiate your answer by arguments for or against the development of

fever or hyperthermia (for example, an effect of cytostatic medication)?

2. If you believe that the patient suffers from fever, or if you believe that he

has hypothermia, explain its possible causes and mechanisms.

3. What is the main criterion of distinction between fever and hypothermia?

N 41

A 18-year-old patient M. felt weakness, dizziness, throbbing headache,

chills, and nausea when he returned home from the beach where he had

spent 6 hours. Thirty minutes later he developed vomiting, and his body

temperature increased up to 390C. He ingested aspirin but it had no much

effect. Despite a moderate decrease of body temperature down to 370C his

condition continued to worsen, and he called in an ambulance. On the way

to hospital he lost his consciousness and was brought to the intensive care

unit.

Questions:

1. Define the type of a pathologic state developed in the patient.

Substantiate your opinion.

2. What are the most likely causes, stages, and mechanisms of this

pathologic state?

3. Why did the patient's state continue to worsen despite a decrease in body

temperature?

4. What was the cause of loss of consciousness in this patient?

HYPOXIA Questions to prepare for classes and examination:

1. Definition of hypoxia and its classification

2. Exogenous hypoxia: causes, types and typical changes of PaO2, PvO2,

Pa-vO2, oxygen blood capacity, PaCO2 and pH in the given type of

hypoxia

Page 37: Collected PathPhys Situations 2010

37

3. Respiratory hypoxia: causes, types and typical changes of PaO2, PvO2,

Pa-vO2, oxygen blood capacity, PaCO2 and pH in the given type of

hypoxia

4. Anemic hypoxia: causes, types and typical changes of PaO2, PvO2, Pa-

vO2, oxygen blood capacity, PaCO2 and pH in the given type of hypoxia

5. Circulatory hypoxia: causes, types and typical changes of PaO2, PvO2,

Pa-vO2, oxygen blood capacity, PaCO2 and pH in the given type of

hypoxia

6. Histotoxic and substrate types of hypoxia: causes, types and typical

changes of PaO2, PvO2, Pa-vO2, blood oxygencapacity, PaCO2 and pH in

the given type of hypoxia

7. Overutilization hypoxia: causes, types and typical changes of PaO2,

PvO2, Pa-vO2, oxygen blood capacity, PaCO2 and pH in the given type of

hypoxia

8. Conditions that determine the susceptibility of cells to hypoxia.

9. Signs of acute hypoxia.

10. Adaptive reactions in acute hypoxia.

11. Mechanisms of long-term adaptation to hypoxia.

N 42

In the study of various aspects of pathogenesis of hypoxia and

edema a laboratory rat was intravenously injected a high dose of

adrenaline. Immediately after the injection paws and ears of the rat turned

pale; arterial blood pressure increased from 120/70 mm Hg to 210/175

mm Hg; heart rate and respiration rate increased sharply. The level of

PaO2 remained unchanged, but the levels of PvO2 and PaCO2 decreased.

Nine minutes after the adrenaline injection despite hyperventilation the rat

developed acrocyanosis. The content of gases in the arterial blood

remained normal, but a progressive decrease in the PvO2 level was noted.

Four minutes later dyspnea and moist rales in the lungs developed; blood

pressure decreased sharply; pulse pressure dropped, and heart beats

became irregular. At the same time, PaO2 started to decrease, and PaCO2

increased. By the end of the 18th minute the rat developed clonic and

Page 38: Collected PathPhys Situations 2010

38

tonic seizures, agonal breathing (gasping) and foamy, reddish discharge

from the respiratory tract. Soon it died.

Questions:

1. Can we state that despite activation of the heart function, increased

vascular tone, and high blood pressure the rat suffered from hypoxia

immediately after the adrenaline injection? If you think so, substantiate

your opinion. If you do not agree, name the time point by which hypoxia

has developed.

2. What are the type(s) and pathogenesis of such hypoxia?

3. Did the rat develop edema? If you think so, name the organs where it

occurred and the time point after the adrenaline infusion when the edema

became evident?

4. If, in your opinion, edema developed in the rat, what forms of pathology in

humans may lead to edema with similar localization?

N 43

An accident occurred with the aircraft during its flight at the height of

10,000 m. The cabin of the aircraft lost its hermetic state, and in several

seconds the barometric pressure inside fell down to atmospheric level of 170

mm Hg at this altitude. The crew failed to make an urgent lowering

maneuver, and the aircraft continued the flight at the same height for several

minutes.

Questions:

1. What pathologic processes developed in the body of passengers during

the aircraft accident? Which of them were the most dangerous for their well-

being and can lead to death?

2. What are the likely causes and mechanisms of these pathologic

processes?

3. What types of preexisting pathology could include the highest risk for life in

this situation?

N 44

A 30-year-old-woman has been brought to hospital by ambulance as

emergency case with a preliminary diagnosis of "rupture of a uterine tube

Page 39: Collected PathPhys Situations 2010

39

with internal bleeding due to ectopic pregnancy". On the way to hospital the

patient lost consciousness several times. Examination in hospital showed

that the patient was conscious, but confused. She was markedly pale, had a

weak, rapid pulse; ausculation revealed muffled heart sounds; blood

pressure 80/35 mm Hg. Examination showed the presence of varicous veins

in the extremities. The patient told the physician that her condition worsened

suddenly when she was eliminating hair on her legs with a depilator.

Unexpectedly she felt short of breath, palpitations, and pain in the heart.

Later on in hospital, despite inhalation of oxygen, she went on complaining of

shortness of breath.

Questions:

1. Which of the typical pathologic processes and what type of it could develop

in the patient? Substantiate your answer.

2. Name the symptoms caused by this pathologic process. What are the

mechanisms, underlying each of the symptoms?

3. Describe the causative factors of this pathologic process. Which of them

are most likely to occur?

4. Why was the inhalation of oxygen ineffective to alleviate shortness

of breath?

N 45

When a physician of the emergency service arrived at the site of a

vehicle accident he found that the victim of the accident developed shock

associated with an open fracture of the right leg and massive blood loss. The

patient also had a closed chest trauma. When bleeding was stopped by

application of a tourniquet the physician injected the patient subcutaneously

an appropriate dose of morphine (to alleviate pain) and a stimulant of the

cardiac function. However, the treatment had no effect, and the symptoms of

shock were progressing. Pulse beats were detectable only in the large

arteries (60 beats/min), blood pressure decreased to 60/25 mm Hg. On

auscultation breathing was depressed over the left chest, and there were no

breathing sounds over the right chest. Several minutes later respiration

became shallow and ineffective; the patient started to "gasp air". To stimulate

Page 40: Collected PathPhys Situations 2010

40

breathing the physician injected subcutaneously a stimulant of the respiratory

center. However, this treatment also had no effect.

Questions:

1. What type (or types) of the emergency condition developed in the patient?

2. What types of hypoxia and in what order developed in the patient?

3. Describe the possible causes and pathogenesis of various types of

hypoxia. Explain the typical changes in pH, PaCO2, SaO2, SvO2, blood

oxygen capacity, the minute volume of breathing, and the minute volume of

circulation during these types of hypoxia.

4. Why was the treatment provided by the physician ineffective?

N 46

A 30-year-old patient K. has been treated in the intensive care unit after

amputation of uterus which was done under an endotracheal ether narcosis.

Suddenly the patient's condition worsened sharply. She developed choking

sensation, dyspnea, chills, psychomotor retardation and apathy. Her skin

turned pale; acrocyanosis became evident. Physical examination showed an

increased respiration rate of 28 per minute, muffled heart sounds, no rales

during auscultation of the chest; the pulse was regular 120 beats/min; blood

pressure 65/30 mm Hg; concentration of Hb 100 g/L; hematocrit 0.30 (N:

0.36-0.42). The patient was intubated and supplied by oxygen but this

measure had no effect.

Questions:

1. What pathologic process(es) developed in the patient after the surgery?

Substantiate your answer.

2. Can the worsening of the patient's condition be associated with the

postsurgery development of hypoxia of:

a) respiratory type?

b) circulatory type?

c) anemic type?

d) histotoxic type?

What are the possible causes and mechanisms of development of

each of the presumed types of hypoxia in this case?

Page 41: Collected PathPhys Situations 2010

41

3. Are there any signs of the urgent adaptive mechanisms that could

compensate hypoxia in this patient? Describe them, if they are present. Why

do they appear to be ineffective in this case?

N 47

A 50-year-old patient K. has undergone a gastrectomy under narcosis

with the use of mechanical ventilation. The surgery was performed urgently

due to extensive stomach bleeding which developed suddenly as a result of

erosion of the large blood vessel in the decaying tumor. During the treatment

of shock and in the course of the surgery the patient received infusions of

volume expanders (approximately 1 L) and transfusion of 2.5 L of whole

donor blood which had been stored for 2 days. By the 3-d day after the

surgery, despite the recovery of the normal blood values of Hb, the patient

complained of weakness, headache, dizziness. On examination: the skin of

the extremities was cold; severe dyspnea and renal insufficiency were

observed; in the conjunctiva a tint of jaundice appeared. Eventually,

mechanical ventilation was initiated to treat the patient.

Questions:

1. What pathologic process(es) developed in the patient after the surgery?

Substantiate your answer.

2. What are the possible causes and mechanisms of development of

each of the pathologic processes:

a) before the surgery?

b) in the course of the surgery and immediately after its completion?

c) by the 3-d day after the surgery?

3. What adaptive reactions could be activated in the patient as a result of

hypoxia? Why were these adaptive reactions ineffective in this case?

N 48

A 60-year-old patient C. had an extensive transmural infarction in the

anterior wall of the left ventricle 2 weeks before. During his stay in hospital

he woke up at night with choking sensation and called for help. When a

nurse came she helped the patient to sit up on the bed with his feet down

on the floor. Then she opened the window. The patient felt a little better.

Page 42: Collected PathPhys Situations 2010

42

However, 10 min later he began complaining of shortness of breath, the

need coughing regularly (without secretions), and audible rales in the

chest. The nurse gave the patient supplemental oxygen by face mask, and

called a physician.

Questions:

1. What types of hypoxia did the patient suffer from? Substantiate your

answer.

2. In what succession developed these types of hypoxia?

3. How and why did the parameters PaO2, PaCO2, PvO2, PvCO2, SaCO2, SvO2,

pH change in the patient's blood? Substantiate your answer.

4. What are the possible mechanisms of immediate adaptation to hypoxia in

patient C.?

DISORDERS OF CARBOHYDRATE METABOLISM. DIABETES MELLITUS.

Questions to prepare for classes and examination:

1. Typical forms of derangement of carbohydrate metabolism.

2. Hypoglycemia: definition, metabolic consequences, adaptive

reaction of the endocrine system.

3. Symptoms of hypoglycemia:

- derived from autonomic response;

- derived from neuroglycopenia.

4. Causes of hypoglycemia.

5. Hypoglycemic encephalopathy and coma.

6. Glycogenoses: classification, manifestations,

mechanisms of pathology.

7. Aglycogenoses.

8. Hexosemias. Galactosemia: manifestation and mechanisms of

pathology.

9. Pentosemias. Fructose intolerance: manifestations and mechanisms of

pathology.

10. Hyperglycemia: causes and mechanisms.

11. Diabetes mellitus: characteristic features, classification, pathogenesis of

type I and type II forms.

Page 43: Collected PathPhys Situations 2010

43

12. Metabolic disorders (carohydrate, protein, lipid) in diabetes mellitus.

13. Complications of diabetes mellitus. Ketoacidotic and hyperosmolar coma.

N 49

A 60-year old patient M. was admitted to hospital in an unconscious state.

On examination: the skin appears dry, turgor of the skin and eye ball is

reduced; shallow breathing; heart rate 96 beats/min; the tongue is dry;

recurrent cramps of the extremities and face muscles. Blood tests show

hyperglycemia (600 mg glucose per dl), hyperazotemia, hypernatremia, pH

7.32.

The relative who accompanied the patient to hospital told the physician

that the patient had been suffering from diabetes mellitus and had ingested

small doses of hypoglycemic medicines. During the last month he

experienced exacerbation of chronic cholecystitis and colitis, suffered from

vomiting and diarrhea which occurred quite often. The patient also felt

constantly thirsty and frequently urinated.

Questions:

1. Define the pathologic state of the patient on admission.

2. What was the cause of this pathologic state? Describe the main stages of

its pathogenesis.

3. Why do patients lose consciousness during the development of this

pathologic state and the similar ones?

4. What methods are used to treat patients in this pathologic state?

N 50

A 28-year-old male patient visited his physician with complaints of

intermittent muscle weakness, dizziness, headache, episodes of poor vision,

tremor of hands, irritability and, occasionally, confusion. The paroxysms of this

condition occurred more frequently during the last 4 months. The patient

attributed his malaise to the psychologic stress that accompanied his

professional activity, and also associated it with an acute feeling of hunger.

After evaluation of the patient the physician made a diagnosis of neurasthenia

and recommended an approprioate treatment. However, the disease went on

Page 44: Collected PathPhys Situations 2010

44

progressing, and 1.5 months later the patient was brought by ambulance to

the emergency room with a diagnosis of coma of unclear etiology.

On admission: consciousness is lost; there is evidence of midriasis,

muscle cramps; tachycardia, arterial hypotension; irregular breathing; blood

glucose level 30 mg/dl.

Questions:

1. What form of pathology caused the clinical manifestations presented by the

patient during his first visit to the physician? Substantiate your answer.

2. Define the pathologic state of the patient in the emergency room.

3. What are the main steps of pathogenesis of this pathologic state?

4. What other forms of pathology should be kept in mind when you are

making a differential diagnosis of this pathologic state?

N 51

A 45 year-old patient E. with an excessive body mass suffers from

diabetes mellitus. To control his blood glucose he uses hypoglycemic drugs.

Two weeks before he was admitted to hospital he had had an episode of

excessive alcohol drinking, and soon after that noticed an enhanced feeling

of dry mouth; drank a lot of water (up to 8-10 L daily); urinated frequently. He

felt general weakness and pain in the legs. On the night before the admission

the patient was delirious. When he woke up in the morning he was agitated,

restless, and confused. The patient's relatives called in the ambulance. On

admission: consciousness is absent; the skin is dry and pale. The results of

the blood tests: glucose 1300 mg/dl (N: 80-120 mg/dl), lactic acid 29 mg/dl,

pH 7.29. Urine glucose level 4 mg/dl.

Questions:

1. What other signs, in addition to the observed in this patient, may be found

in a) diabetes mellitus, and b) diabetic coma? Describe these signs and

explain their pathogenesis.

2. What are the main stages of the pathogenesis of hyperosmolar coma?

3. What are the main principles of therapy for diabetic hyperosmolar coma?

Page 45: Collected PathPhys Situations 2010

45

N 52

A female patient, approximately 40-years of age, has been brought to the

emergency room in an unconscious state. Witnesses of the incident told the

physician that she had suddenly lost her consciousness in the bus on her way

back from her summer house in the countryside. A card that was found

among her papers indicated that she suffered from diabetes mellitus and took

a slow-release form of insulin.

On examination: consciousness is absent, corneal and deep tendon

reflexes are not observed, the pattern of breathing is unremarkable; blood

pressure 80/60 mm Hg; tachycardia is present; the skin appears moist; turgor

of the eye balls is increased; general trembling is alternating with episodes of

clonic and tonic seizures.

The patient was treated with insulin, but no improvement was observed;

the patient's condition even worsened: breathing became irregular, blood

pressure decreased to 70/50 mm Hg, tachycardia progressed, and the

duration of seizures increased.

Questions:

1. What pathologic state developed in the patient before and after

administration of insulin? Describe causes and mechanisms of this state.

2. What therapeutic procedure would have been appropriate for the patient

before and after the administration of insulin?

DISORDERS OF LIPID METABOLISM. ATHEROSCLEROSIS.

Questions to prepare for classes and examination:

1. Typical forms of disorders of lipid metabolism.

2. The main classes of lipoproteins and their functions.

Atherogenic and antiatherogenic lipoproteins.

3. Characteristics of hyperlipidemia. Classification of

hyperlipoproteinemias according to Fredrickson.

4. Mechanisms of hyperlipidemia.

5. Characteristics and mechanisms of hypolipidemia.

6. Obesity: definition, diagnostic approaches, pathological

significance. Types and pathogenesis of obesity.

7. Features and mechanisms of lypodystrophy.

Page 46: Collected PathPhys Situations 2010

46

8. Characteristics of lipidoses.

9. Atherosclerosis: description and pathological features.

10. Stages of atherogenesis:

- initiation;

- formation and evolution of atheroma;

- complications.

12. Risk factors of atherosclerosis.

N 53

A 15-year-old-boy visited a doctor with complaints of recurrent pain in

the region of the heart. The pain was exacerbated at exertion. Examination

of the patient showed the presence of small, firm, yellow nodules

(xanthomas) over the course of the hand muscles tendons, the presence of

corneal lipid arcus. Angiographic examination revealed the coronary artery

stenosis. Blood tests results demonstrate: total plasma cholesterol 22

mmol/L (normal values: 3.1-6.4 mmol/L), triglycerides 1.7 mmol/L (normal

values: 0.55-1.65 mmol/L), HDL-cholesterol 0.7 mmol/L (normal values: >

0.9 mmol/L), LDL-cholesterol 9 mmol/L (normal values: 3.0-4.5 mmol/L).

The patient's parents also have increased plasma levels of cholesterol -

more than 8 mmol/L. Immunocytochemical analysis of the patient's

leukocytes revealed abnormality of the LDL-receptor.

Questions:

1. What type of hyperlipidemia is observed in the patient?

2. Is heredity important in the development of this pathology? If it is, what is

its type of inheritance. What is the prevalence of this pathology in general

population?

3. What mechanisms underlie hyperlipidemia in this patient?

4. Assess the atherogeneicity of the patient's plasma by calculating the index

of atherogeneicity.

5. What are the pathologic consequences of hyperlipidemia of this type? What

therapeutic approaches may be used to treat this pathology?

Page 47: Collected PathPhys Situations 2010

47

N 54

During examination of a 5-year-old boy a physician found an enlargement

of the liver and spleen (hepatosplenomegaly), swelling of the tonsils which

had red-orange color, and an enlargement of inguinal, axillary, and other

superficial lymph nodes. Also, clouding of the cornea was observed.

Biochemical tests revealed: total cholesterol of the blood plasma 0.1 mmol/L,

(normal values: 3.1-6.4 mmol/L), triglycerides 2.3 mmol/L (normal values:

0.55-1.65 mmol/L), HDL fraction almost absent. During electrophoretic

separation of the patient's plasma apoproteins A-I and A-II bands were not

observed.

Questions:

1. Disorders of lipid metabolism of what type is observed in the patient?

2. What mechanisms underlie the development of this disorder?

3. Why is HDL fraction considered antiatherogenic? What factors determine

HDL levels in the blood plasma?

4. What role do apoproteins of the A class play in the cholesterol metabolism?

N 55

During experimental studies of atherosclerosis one group of rabbits was

fed cholesterol added to a standard pelleted food in a dose of 5 g per day.

Cholesterol was previously purified to remove contaminating oxides. The

second group was treated similarly, but cholesterol was not purified, and the

third group received the same diet with cholesterol that was previously

subjected to peroxidation by hydrogen peroxide in the presence of ferrous

chloride (FeCl2). Six months after the start of the experiment the rabbits were

killed by a lethal dose of a narcotic drug; the abdominal portion of the aorta

and coronary arteries were isolated, and their internal surface were examined

for the presence of lesions. Simultaneously, blood samples were obtained

from all rabbits of the experimental groups; plasma was separated from formal

particles and used to isolate LDL fraction. In the following in vitro studies

samples of the isolated LDL fractions from various groups were added to a

media containing cultures of murine peritoneal macrophages, and the extent

of intracellular accumulation of cholesterol esters was assessed following

incubation for 3 days. LDL fraction of the intact rabbits was used as a control.

Page 48: Collected PathPhys Situations 2010

48

Questions:

1. Which group of the rabbits is supposed to incur the maximum damage to

their arteries due to the excess of cholesterol in the food?

2. Describe the pathways by which LDL can enter peripheral cells, including

macrophages.

3. What differences in accumulation of lipids in murine macrophages can be

expected with LDL fraction isolated from various experimental animals?

Compare LDLs of the control and experimental groups of rabbits.

4. What role do monocytes/macrophages play in atherogenesis?

N 56

During his visit to the doctor a 55-year-old male patient complains of

anginal pain which recently has increased in frequency and intensity. The

patient is the president of a big trading company. During the last months he

has experienced great psychologic stress, spent 12-14 hours at his office

daily, and slept little; he began smoking more than one pack of cigarettes per

day. The patient is a hearty eater, preferring high-calorie fat food. His father

died of myocardial infarction. On examination: his height is 173 cm, weight

89 kg, blood pressure 175/100 mm Hg. Blood tests show the following: total

plasma cholesterol 6.8 mmol/L (normal values: 3.1-6.4 mmol/L), triglycerides

1.9 mmol/L (normal values 0.55-1.65 mmol/L), HDL cholesterol 0.8 mmol/L

(normal values > 0.9 mmol/L), LDL cholesterol 5.3 mmol/L (normal values <

4.9 mmol/L); glucose tolerance is decreased.

Questions:

1. What form of pathology is the cause of angina pectoris in this patient?

2. What risk factors of this pathology are observed in the patient?

3. Does the patient have signs of lipid metabolism disorder? If he does,

what forms of it?

4. What therapeutic approaches can be used to treat lipid metabolism

disorders in this case?

WATER DYSBALANCE. EDEMA. Questions to prepare for classes and examination:

1. Typical forms of water dysbalance.

Page 49: Collected PathPhys Situations 2010

49

2. Hypohydration: causes, types, mechanisms of development,

manifestations, consequences, and pronciples of treatment.

3. Overhydration: causes, types, mechanisms of development,

manifestations, consequences, and pronciples of treatment.

4. Edema: definition, types, the main factors of development, principles of

treatment.

5. Edema during cradiac failure: etiology, pathogenesis, and manifestations.

6. Pulmonary edema: etiology, pathogenesis, manifestations, and

consequences.

7. Renal edema: etiology, pathogenesis, manifestations, and consequences.

8. Disorders of sodium, potassium, calcium, manganese, and phosphate

metabolism: causes, manifestations, and mechanisms of develpment.

N 57

Examination of a 32-year-old patient revealed various signs of pathology,

including excessive body mass: his height is 168 cm, and weight 84.5 kg. The

patient also has a pasty face, periorbital puffiness, pale skin; he had slow

rebound of tissue to its original contour after pressing the feet or shin with the

fingertip. The patient told the physician about tightness of a ring and shoes in

the evening. An investigation of the cardiovascular system revealed the

following: minor arterial hypotension, areas of cardiac dullness are slightly

increased; other parameters are unremarkable. The daily urine volume is

within the normal range.

Questions:

1. What is the possible cause of the patient's excess of body mass?

2. Can we assume that water-ionic balance is deranged in this patient?

3. What type of edema is observed in the patient?

4. What additional data are required to specify the type of edema in this case?

N 58

A 42-year-old patient has been admitted to hospital with a diagnosis of

uncompensated chronic heart failure due to valvular disease. The patient has

normal constitution with paucity of subcutaneous tissue. His height is 165 cm,

body weight 81 kg. On examination: the patient needs to sit in bed; he has

Page 50: Collected PathPhys Situations 2010

50

dyspnea, acrocyanosis, marked lower extremities edema, rales and wheezes

during auscultation of the chest. X-ray investigation of the abdominal area

shows an accumulation of fluid; the liver is enlarged; stroke volume and

cardiac output are decreased; hematocrit 38%; daily urine volume is

decreased. Biochemical tests reveal increased plasma activity of renin and

increased sodium concentration.

Questions:

1. Are there any signs of derangement of water balance in this patient?

2. What type of dyshydria is observed in this case?

3. Is there any association between the accumulation of fluid in the

subcutaneous tissue, the abdomen, and the lungs?

4. Explain the pathogenesis of increased blood levels of renin and Na+ in this

patient.

5. Explain the pathogenesis of edema in this patient.

6. Explain the role of edema in deterioration of the patient's condition.

7. What therapeutic approaches can be used to treat the edema in this case?

N 59

A 22-year-old patient who recovered from severe scarlet fever 2 weeks

ago complains of headache, pain in the back, dyspnea, and palpitations.

During the last week she has increased her body weight by 11.5 kg. On

examination: her face is pale; she has periorbital puffiness, and edema of

the shins and feet; the boundaries of the heart dullness are increased;

blood pressure is 180/100 mm Hg; daily urine volume is reduced. Urine

tests show the presence of erythrocytes and protein. An increased titer of

antistreptolysin O antibodies is found in the blood.

Questions:

1. Is there evidence of the kidney damage in this patient? What is the

possible mechanism of this pathology?

2. What is the cause of hyperhydration in this case: a decrease in water

excretion or an increase in water retention?

3. Explain the mechanisms of edema in this patient.

N 60

Page 51: Collected PathPhys Situations 2010

51

A 7-year-old boy developed a progressive swelling of the soft palate with

a swallowing difficulty, and then asphyxia after he had drunk mango juice.

The mucosal membrane in the swelled area is hyperemic without tenderness;

a moderate increase in eosinophils is seen in the blood. The patient's body

temperature is normal. His senior sister suffers from attacks of bronchial

asthma.

Questions:

1. Is edema in this case the result of ordinary inflammation?

2. What is the cause of edema in this patient?

3. Explain the pathogenesis of the given pathology.

4. Does this type of edema lead to life-threatening condition?

ACID-BASE DISORDERS Questions to prepare for classes and examination:

1. The concept of acid-base homeostasis.

2. The main buffer systems of the blood.

3. Physiologic mechanisms responsible for the normal acid-base balance:

- role of the respiratory system;

- role of kidneys.

3. Simple and mixed acid-base disorders.

4. Typical forms of disorders of acid-base balance.

5. Metabolic acidosis: causes and symptoms.

6. The specific examples of metabolic acidosis:

- High-anion gap acidosis;

- Hyperchloremic metabolic acidosis.

7. Metabolic alkalosis: causes and symptoms.

8. Mechanisms and examples of metabolic alkalosis.

9. Respiratory acidosis: causes and mechanisms.

10. Respiratory alkalosis: causes and mechanisms.

11. Primary and compensatory changes in various types acidosis and

alkalosis.

12. Clinical manifestations of alkalosis and acidosis.

13. Laboratory parameters used in diagnostics of acid-base dysbalance.

Page 52: Collected PathPhys Situations 2010

52

N 61

A group of mountain-climbers in which a physician was included planned

to climb a mountain peak of 6200 m. Climbing up to the level of 2900 m was

successful, but at this altitude the physician, being an inexperienced climber,

felt fatigue, muscle weakness, dizziness, buzzing in the ears (tinnitus). During

a one-hour halt the physician collected samples of the capillary blood of his

own (B1) and his mate (A1). Then the group continued the ascent. At the

height of 5000 m the physician felt acute shortness of breath, severe

weakness, heaviness in the body, vision disturbances, and became ataxic.

For this reason he decided to stop climbing. The state of the experienced

climbers was satisfactory. After the repeated withdrawal of blood samples

(A2,B2) all the climbers returned to the camp at the level of 3000 m.

The results of laboratory tests of the blood samples are the following:

A1 A2 B1 B2

pH 7.43 7.35 7.48 7.32

pCO2 (mm Hg) 32 30 26 42

pO2 (mm Hg) 74 48 70 38

SB (mmol/L) 20.5 18.5 20.0 18.5

BE (mmol/L) +2.5 -3.6 +4.5 -5.5

Questions:

1. Are there any signs of hypoxia and acid-base disorders in the physician or

his mate at the given altitudes? If you think there are, define the type of

hypoxia and acid-base disorder and explain the cause of its development.

Substantiate your answer by the given data.

2. What adaptive mechanisms, if any, were initiated in the body of the

physician and the experienced climber when they were ascending? Were

there any differencies in the pattern of adaptive mechanisms in the physician

and the experienced climber?

3. Why did the adaptive mechanisms fail in the physician?

DISORDERS OF THE TISSUE GROWTH. NEOPLASMS.

Questions to prepare for classes and examination:

1. Definition of neoplasms. Principles of classification of neoplasms.

Page 53: Collected PathPhys Situations 2010

53

2. Forms of neoplastic cell abnormality.

3. Characteristic features of benign and malignant tumors.

4. Genetic hypothesis of cancer; classes of genes that incur mutations during

neoplastic transformation of a cell.

5. The concept of oncogenes.

6. Mechanisms of transformation of protooncogene to oncogene.

7. Role of antioncogenes in carcinogenesis.

8. Role of genes that regulate DNA repair in carcinogenesis.

9. Chemical carcinogenesis: types of carcinogens; initiation and promotion.

10. Radiation carcinogenesis.

11. Viral carcinogenesis.

12. The concept of tumor progression.

12. Antineoplastic mechanisms. Principles of prevention and treatment of

neoplastic diseases.

N 62

A 48-year-old patient N. who has been smoking for 25 years works at

the chemical plant where he deals with dyes and organic solvents. He

visited his primary care physician with complaints of malaise, weakness,

decreased appetite, and persistent cough without secretions. During the

previous 3 weeks he noted the presence of blood in the urine, burning

sensation in the lower abdomen aggravated by urination. The results of

computer tomography scanning, x-ray, and ultrasonography attest against

renal or ureter pathology in the patient. Cystoscopy shows proliferation of

the bladder's mucosa with erosion of its surface. Biopsy of the lesion in the

bladder shows the presence of malignant cells.

Questions:

1. What factors could be a potential cause of cancer of the bladder mucosa

in this case? Substantiate your answer.

2. What mechanisms of the anticancer defence system could have been

deficient in this patient and failed to prevent the development of tumor?

3. Characterize stages of carcinogenesis after the first contact

of the bladder mucosa with carcinogen and up to the emergence of the tumor

cells.

Page 54: Collected PathPhys Situations 2010

54

4. Can you exclude the possibility of metastases of the lung cancer in the

bladder? Substantiate your opinion.

N 63

Seven months after a patient had undergone a surgical removal of the

stomach carcinoma and completed a course of chemotherapy he presented

with an enlargement of a supraclavial lymph node. Biopsy of the swelled

lymph node showed the presence of malignant cells. Some of these cells

were similar to the removed tumor cells by their morphology.

Questions:

1. Can you attribute the appearance of the tumor cells in the lymph node to

the tumor progression phenomenon? Substantiate your answer and

characterize the phenomenon of tumor progression (its mechanisms and

biological significance).

2. Do you think that the presence of the tumor cells in the lymph node is the

result of the primary tumor metastases?

- a multicentered tumor growth?

- recidivation of the stomach carcinoma?

- a new tumor growth?

Substantiate your opinion.

3. What factors of the anticancer defence system were ineffective in this

case? What are the possible mechanisms of their action?

N 64

A 56-year-old patient M., who has been suffering from gastric atrophy and

hypoacidity for more than 20 years, complains of fatigability, weakness, pain

in the epigastrium, decreased appetite, rapid satiety during meal, nausea, a

great loss of body mass during the last 4 months, and a persistent fever.

Laboratory tests showed anemia, leukocytosis, hypochlorhydria, and a

decreased activity of gastric juice enzymes. Gastroscopy revealed flattening

of the mucosal folds in the pyloric area, and the presence of a saucer-like

tumor with ulcerative alterations in the center.

Questions:

Page 55: Collected PathPhys Situations 2010

55

1. Why does chronic gastric atrophy promote the development of a

gastric tumor?

2. Can we suspect the insufficiency of the antineoplastic mechanisms in this

case? If we can, describe these mechanisms.

3. What are the possible causes and mechanisms of fever and anemia in this

patient?

4. What are the possible causes of cachexia in this patient?

N 65

A 40-year-old patient B. visited his physician with complaints of

considerable weakness, dizziness, persistent cough with minor secretions. He

had participated in the liquidation of the nuclear power station accident 1.5

years before. B. told the physician that he had been a heavy smoker for 20

years, but 2 years before had quitted smoking. During the last 6 months he

repeatedly suffered infectious diseases, such as quinsy, bronchitis,

pneumonia. During bronchoscopy a tumor of the main right bronchus was

found. Histology of the tumor showed the presence of the squamous

epithelium malignant cells.

Questions:

1. What factor was the most likely cause of bronchial cancer in this

patient? Substantiate your answer.

2. What factors could potentiate the effect of carcinogens in this patient? What

are their possible mechanisms of action?

3. What antineoplastic mechanisms should have become activated in this

patient:

- under the effect of the carcinogens?

- in the course of neoplastic transformation of the bronchial epithelial cell?

- during the formation of the primary tumor nodule?

Why did the antineoplastic mechanisms fail in this case?

EMERGENCY STATES

Questions to prepare for classes and examination:

1. Shock: characteristics, types, etiology, pathogenesis, manifestations,

principles of treatment.

Page 56: Collected PathPhys Situations 2010

56

2. Coma: characteristics, types, etiology, general pathogenesis,

manifestations, principles of treatment.

PATHOLOGY OF THE RED BLOOD CELLS SYSTEM. ANEMIA. Questions to prepare for classes and examination:

1. Definition of anemia and its classification.

2. Anemias of blood loss: types, causes, hematological signs.

3. Hemolytic anemias: types, common causes, manifestations,

hematological signs.

4. Hereditary spherocytosis: pathogenesis, manifestations, hematological

signs.

5. Sickle cell disease: pathogenesis, manifestations, hematological signs.

6. Thalassemia syndromes: types, pathogenesis, manifestations,

hematological signs.

7. Megaloblastic anemias: types, causes, pathogenesis, manifestations,

hematological signs.

8. Iron deficiency anemia: causes, pathogenesis, manifestations,

hematological signs.

9. Aplastic anemia: causes and pathogenesis, hematological signs.

10. Erythrocytosis: causes and mechanisms

N 66

A 5-year-old female patient Zulfia was admitted to hospital with a

preliminary diagnosis of pneumonia. On admission: the skin and conjunctiva

appeared jaundiced, the body temperature was 390C, the spleen and liver

were enlarged, the urine was dark; the patient complained of an acute pain

in the back, in the right anterior abdomen, in the leg's muscles; X-ray

investigation of the chest revealed signs of bronchopneumonia. Blood tests

showed the following: Hb 69 g/L, erythrocytes 2.3*1012/L, MCH (mean

corpuscular hemoglobin) 30 pg; the presence of poikilocytosis and

anisocytosis of erythrocytes. The urine test revealed the presence of Hb.

During electrophoretic investigation of the patient's Hb the HbS type was

found in addition to the HbA.

Questions:

Page 57: Collected PathPhys Situations 2010

57

1. What forms of pathology are observed in the given patient?

Substantiate your answer.

2. Explain the cause-effect and chronologic relationship between the

observed forms of pathology.

3. What is the cause of the primary pathology? Which of the given

symptoms suggest the presence of this pathology? What are the

mechanisms of their development?

4. Explain the mechanisms of the following symptoms:

- acute pains in various organs and tissues;

- jaundice;

- poikilocytosis;

- anisocytosis.

5. What measures would you recommend to the girl's parents to prevent the

recurrence of the described symptoms?

N 67

A 60-year-old female patient K. was admitted to hospital with complaints

of weakness, persistent headaches, dizziness, "shakiness", numbness in the

hands and legs, minor dyspnea, decreased appetite, tenderness in the oral

cavity during salty or acidic meal, a burning sensation in the tongue. The

patient is a vegetarian. Two weaks before she visited her physician because

of anorexia, pains and heaviness in the epigastium, and diarrhea. Gastric

juice investigation showed severe hypochlorhydria. In the blood test there

were hyperchromatic anemia, macrocytosis and poikilocytosis of

erythrocytes, leukopenia, hypersegmented neutrophil nuclei, and a low

platelet count.

On examination in hospital: the patient has marked paleness of the skin

and mucosal membranes, the conjunctiva appears jaundiced, blood pressure

110/75 mm Hg, the surface of the tongue is bright red, smooth, and shiny

("the polished tongue"); there are signs of stomatitis. The patient has

unsteady gait. Concentration of folic acid in the blood serum is normal.

Questions:

1. What forms of pathology are observed in the given patient?

Page 58: Collected PathPhys Situations 2010

58

Substantiate your answer. Explain the likely chronologic and pathogenetic

relations between these forms of pathology.

2. Which of the given pathologic processes is primary? Substantiate your

answer.

3. Characterize anemia developed in this patient by the following criteria:

- type of hemopoiesis;

- erythrocyte count in the peripheral blood;

- mean corpuscular hemoglobin;

- the size and shape of erythrocytes;

- the presence of inclusions in the erythrocyte's cytoplasm.

4. Explain the origin of the following symptoms:

- shaky gait;

- numbness of hands and legs;

- stomatitis;

- "the polished tongue".

5. Which of the symptoms presented by the patient refer to hypoxia? Prove or

disprove the presumption about the presence of hypoxia of the circulatory,

respiratory, anemic, or cytotoxic type in this patient?

N 68

A 26-year-old pregnant woman visited her physician with complaints

of intense muscle weakness, fatigability, swallowing difficulty, choking

during meal, and a burning sensation in the tongue. These symptoms

developed in the third trimester of pregnancy. On examination: the patient

has marked paleness of the skin and mucosal membranes, signs of

stomatitis and hypotrophy of the tongue papillae, flattened,

spoon-shaped nails (koilonychia). The results of blood tests: Hb 60 g/L,

erythrocytes 2.8*1012/L, reticulocytes 0.2%, poikilocytosis and microcytosis

of erythrocytes, leukocytes 3.6*109/L, platelets 200*109/L; total serum

iron-binding capacity is increased; hypoferritinemia and increased level of

free protoporphyrin are found.

Questions:

1. What form of pathology developed in the patient? Explain your answer.

2. What is the cause of this pathology? What other factors can cause it?

Page 59: Collected PathPhys Situations 2010

59

3. What mechanisms underlie each of the symptoms developed in the

patient?

4. What approaches may be used to treat this form of pathology?

N 69

A patient has been transported to hospital 40 min after he got a gunshot

wound. On admission: the patient is conscious, but confused; his skin is

pale, breathing is rapid and shallow; he has a running pulse; blood pressure

is 65/35 mm Hg. The results of blood tests: Hb 142 g/L, erythrocytes

4.5*1012/L, MCH 31.5 pg. Since the patient had signs of internal bleeding

and accumulation of the blood in the peritoneal cavity, he was operated and

underwent ligation of the injured mesenteric artery branch. On the third day

of treatment in hospital the patient's blood test data were the following: Hb

68 g/L, erythrocytes 2.8*1012/L, reticulocytes 10%. His blood pressure

reached 115/70 mm Hg.

Questions:

1. What pathologic state was observed in the patient on admission?

Substantiate your answer.

2. Evaluate changes in the blood test data on the 1st and the 4th days after

the injury, and make a conclusion. Explain the differences in the results of

these tests?

3. Was the pathologic state observed in the patient on admission still

present on the 4th day of treatment in hospital? If it was, substantiate your

opinion. If it was not, define the pathologic state developed on the 4th day

after the patient's injury.

4. What approaches would you use to treat the patient on the 4th day after the

injury?

N 70

A 42-year-old female patient D. visited her general practitioner with

complaints of headache, insomnia, weakness, fatigability, episodes of uterine

bleeding not associated with menses (dysmenorrhea). The latter occurred

about 6 months ago. The results of blood tests: Hb 95 g/L, erythrocytes

Page 60: Collected PathPhys Situations 2010

60

3.3*1012/L, reticulocytes 8.5%, microcytosis of erythrocytes, leukocytes count

4.9*109/L; platelets 240*109/L.

Due to family problems the patient failed further investigation and

treatment. Meanwhile, episodes of the uterine bleeding recurred, and her

condition continued to deteriorate. Three months later she was admitted to

hospital with a diagnosis of uterine myoma. Blood tests made on admission

showed: Hb 45 g/L, erythrocytes 2.2*1012/L; erythrocytes are hypochromic

with signs of anisocytosis; microcytes are abundant; reticulocytes constitute

0.05%; leukocytes count is 1.9*109/L, and platelets count is 110*109/L.

Questions:

1. What forms of blood cells pathology developed in the patient during each of

the follow-up periods (outpatient's and inpatient's)? Substantiate your answer.

2. What additional tests are required to specify the pathogenesis of the

patient's disease developed by the moment of admission?

3. Explain the reason why the results of the first blood test differ from that of

the second one. What is the prognostic value of the observed differences?

4. What approaches may be used to treat this form of pathology?

N 71

A 36-year-old-patient underwent an ultrasonic evaluation of the

abdominal area. The results of it suggested an enlargement of the right

kidney. The later performed computer scanning revealed a mass that had the

size of 1.2x1.5x1.5 cm and localized to the superior pole of the right kidney.

For this reason the patient was hospitalized. The patient's blood test showed:

Hb 180 g/L, erythrocytes count 7.5*1012/L, reticulocytes 10%, leukocytes

4.0*109/L, platelets 250*109/L, hematocrit 0.61; erythropoietin concentration

is 20% above the normal values; blood pressure 150/90 mm Hg. A histologic

investigation of cells, obtained by needle aspiration of the right kidney's mass,

confirmed the presence of hypernephroma. The patient underwent surgical

removal of the tumor. Three weeks later his condition improved; the results of

his clinical tests were normal.

Questions:

1. What form of pathology developed in the patient in connection with

hypernephroma? Substantiate your opinion using the given data.

Page 61: Collected PathPhys Situations 2010

61

2. What is the specific cause of this pathology in the patient?

3. Describe the mechanisms of this pathology and of the symptoms presented

by the patient?

4. What other factors may cause the development of this pathology?

PATHOLOGY OF THE WHITE BLOOD CELL SYSTEM. LEUKOCYTOSIS, LEUKOPENIA. Questions to prepare for classes and examination:

1. The main stages of leukopoiesis and its regulation by humoral factors.

2. Classification of disorders of white blood cells.

3. Leukopenia: definition and causes.

4. General mechanisms of leukopenia.

5. Causes and mechanisms of decreased leukocytes production.

6. Causes and mechanisms of increased peripheral destruction of leukocytes.

7. Signs and consequenses of neutropenia.

8. Leukocytosis: causes and mechanisms.

9. Causes of increased production in the bone marrow, mobilization, and

defective margination of neutrophils.

10. Types of leukocytosis; patterns of shift to the left.

11. The biological significance of leukocytosis.

12. Types and mechanisms of abnormal neutrophil function.

13. Leukemoid reactions: typical features, causes, mechanisms of

development, physiologic significance.

14. Changes in the complete blood test with differential. Absolute and

relative shifts in leukocytes count.

N 72

A 38-year-old male industrial worker visited his general practitioner. The

patient was a victim of fire at his working place. During the accident he had

received epidermal and partial thickness burns of the right side of the body

(about 10% of the total body surface area). His condition was fair; body

temperature 37.80C. On the fifth day the patient's condition deteriorated due

to an intercurrent infection of the burn wound. He developed signs of marked

purulent, exudative inflammation of the derma and subcutaneous tissue; his

Page 62: Collected PathPhys Situations 2010

62

body temperature elevated to 40.20C. Blood tests were taken twice at different

time points. The following results were obtained:

Analysis "A" Analysis "B"

Hemoglobin

Erythrocytes

Reticulocytes

Leukocytes:

Neutrophils

Myelocyte

Metamyelocyte

Band

Segmented

Eosinophils

Basophils

Lymphocytes

Monocytes

125 g/L

4.5 *1012/L

0.5% 10.5 * 109/L

0

0

6%

68%

2%

0

21%

3%

125 g/L

4.7*1012/L

0.8% 16 * 109/L

1%

4%

14%

60%

0

0

16%

5%

Questions:

1. Evaluate the results of the tests "A" and "B" and make a conclusion to

each of them. What is the difference between these two analyses?

2. One of these tests was made on the same day and the other on the fifth

day after the trauma. Define the time when each of the tests were made.

Substantiate your opinion.

3. What are the causes and mechanisms of the observed changes in the

patient's blood?

N 73

A 26-year-old female patient N. was treated for obsessive-compulsive

disorder and hyperthyroidism in the out-of-town hospital. She was prescribed

sedative and antithyroid drugs. Several days after the onset of treatment she

visited her physician with complaints of malaise, sore throat, burning

sensation in the mouth, and dizziness. On examination: signs of follicular

tonsillitis and stomatitis. Blood test showed: Hb 120 g/L, erythrocytes

Page 63: Collected PathPhys Situations 2010

63

4.2*1012/L, reticulocytes 0.5%, leukocytes 0.5*109/L, eosinophils and

basophils are not found, segmented neutrophils 20%, other neutrophils or

precursors - not found; lymphocytes 75%, monocytes 5%. During a bone

marrow examination a great number of immature leukocytes without abnormal

features were observed.

Questions:

1. What forms of pathology developed in the patient? Substantiate your

answer.

2. What are the causes of these forms of pathology?

3. What are the causes and mechanisms of changes in the peripheral blood

and bone marrow in this case?

4. What mechanisms underlie the symptoms observed in the patient?

5. What approaches can be used to treat the patient?

N 74

Complete blood test with differential in a 80-year-old patient who had

recurrent episodes of bleeding associated with unresectable gastric

carcinoma showed the following:

Hb 85 g/L

Erythrocytes 3.4*1012/L

MCH to be calculated

Reticulocytes 0.1%

Leukocytes (total) 53*109

promyelocytes 2.5%

myelocytes 8.0%

metamyelocytes 11.0%

band neutrophils 18.0%

segmented neutrophils 51.0%

basophils 0

eosinophils 0.5%

lymphocytes 5.0%

monocytes 4.0%.

Page 64: Collected PathPhys Situations 2010

64

Neutrophils have toxic granules and increased alkaline phosphatase

activity; no Philadelphia chromosome is found in the leukocyte precursors

during a bone marrow examination.

Questions:

1. Analyze the blood test data and make a conclusion.

2. What are the causes and mechanisms of changes in the red blood and

white blood cells? What other forms of blood pathology should the given

disorder be distinguished from? Substantiate your answer.

3. Prove or disprove an assumption that changes in the blood test can be the

result of metastases of gastric carcinoma in the bone marrow.

4. Is it possible that the patient developed leukemia under the effect of the

same carcinogen that had caused gastric carcinoma.

N 75

A 29-year-old male patient has signs of moderate radiation injury incurred

during accident at the nuclear power station. His blood test taken the next day

after the accident shows:

Leukocytes 11*109/L

Neutrophils:

band 7%

segmented 79%

Eosinophils 2%

Basophils 0

Lymphocytes 7%

Monocytes 5%

The patient’s blood test made on the 14th day after the accident shows:

Leukocytes 1.0*109/L

Neutrophils:

Band 2%

Segmented 53%

Eosinophils 0

Basophils 0

Lymphocytes 40%

Monocytes 5%.

Page 65: Collected PathPhys Situations 2010

65

Questions:

1. Make a conclusion on the results of the two blood tests.

2. What are the mechanisms of changes in white blood cells (for each type of

leukocytes) at the beginning and later stages of the radiation disease.

3. Compare the time-course of changes in granulocyte and lymphocyte during

the progression of the radiation disease.

4. What type of leukocytes (granulocytes or lymphocytes) is more susceptible

to ionizing radiation?

N 76

A 48-year-old patient D. has been admitted to hospital with signs of

severe tonsillitis and stomatitis. She has a history of chronic headaches and

prolonged (many months) intake of amidopyrin at average therapeutic doses.

Complete blood count test shows the following: leukocytes 1.8*109/L,

neutrophils: segmented 10%, band - not detected; eosinophils - not

detected, basophils – not detected, lymphocytes 75%, monocytes 15%; red

blood cells and plateletes are within the normal range.

Questions:

1. Make a conclusion on the results of the blood test.

2. What is the cause-effect association between the clinical symptoms and the

blood test findings in this case? Substantiate your answer.

3. What is the most likely cause of changes observed in the peripheral blood

of the given patient? Describe the possible mechanism of these changes.

4. What additional tests would be helpful to specify the mechanism of the

hematologic changes in this patient?

N 77

Blood test has been performed in a patient with unresectable carcinoma

of the stomach. It shows the following results:

Hb 95 g/L

Erythrocytes 3.4*1012/L

Leucocytes 53*109/L

promyelocytes 2.5%

myelocytes 8.0%

Page 66: Collected PathPhys Situations 2010

66

metamyelocytes 11.0%

band neutrophils 18.0%

segmented neutrophils 51.0%

Basophils 0

Eosinophils 0.5%

Lymphocytes 5.0%

Monocytes 4.0%

Questions:

1. Characterize the presented hematologic data.

2. What form of pathology the observed abnormality of the white blood should

be distinguished from? What methods can be used for this?

3. Make a conclusion on this blood test.

4. What are the likely causes of the observed changes in the peripheral

blood?

MALIGNANCIES OF HEMATOPOIETIC CELLS. LEUKEMIA Questions to prepare for classes and examination:

1. Typical features, types, etiology, and general pathogenesis of hematopietic

cell tumors.

2. Leukemia: general characteristics, causes, classification and mechanisms

of development.

3. Characteristics of the leukemic reaction of the peripheral blood; types of

leukemic reaction.

4. Lymphoid tumors: pathological basis, clinical manifestations, and

peripheral blood picture in acute lymphoblastic leukemia and chronic

lymphocytic leukemia.

5. Myeloid tumors: classification, pathological basis, clinical manifestations,

and peripheral blood picture in acute myeloblastic leukemia and chronic

myelogenous leukemia.

6. Phenomenon of tumor progression in hematopoietic cell tumors.

N 78

A 62-year-old female patient has been hospitalized with the diagnosis of

"exacerbation of chronic bronchopneumonia". The patient complains of

Page 67: Collected PathPhys Situations 2010

67

weakness, dyspnea, fever. On examination: the patient is pale; she has a tint

of jaundice in the conjunctiva, and a moderate enlargement of the regional

lymph nodes, spleen, and liver. An X-ray examination reveals the signs of

right-sided bronchopneumonia, a significant enlargement of the hilar lymph

nodes. Complete blood counts with differential shows:

Hb 80 g/L

Erythrocytes 3.0*1012/L

MCH to be calculated

Reticulocytes 0.6%

Platelets 105*109/L

Leukocytes (total) 36*109/L

myelocytes 0

metamyelocytes 0

band neutrophils 0.5%

segmented neutrophils 6.0%

basophils 0

eosinophils 0

lymphoblasts 1.5%

lymphocytes 89.5%

monocytes 2.5%.

In the blood an increased content of unconjugated bilirubin, and the

presence of anti-RBC and antiplatelet antibodies are found.

Questions:

1. What forms of pathology developed in the patient?

2. Is there association between these forms of pathology? Substantiate your

answer.

3. What are the causes and mechanisms of symptoms presented by the

patient:

- weakness and fatigability?

- dyspnea?

- bleeding diathesis?

- fever?

- jaundice in the conjunctiva?

- enlargement of the liver, spleen, and regional lymph nodes?

Page 68: Collected PathPhys Situations 2010

68

- increased blood content of unconjugated bilirubin?

N 79

A 52-year-old male patient L. suffers from leukemia, and takes the

appropriate treatment. He has visited his physician with complaints of

exacerbation of his disease. Recently he has begun to experience

progressive weakness, dyspnea, sleep problems, gum bleeding, and fever.

He marks the emergence of bruises on the forearms and shins. He also

complains of pain in the heart. On examination: paleness of the skin and

mucosal membranes, signs of multiple subcutaneous hemorrhages of various

age, pustular rash in various locations of the body; the subcutaneous lymph

nodes, the liver, and the spleen are enlarged. CBC (complete blood count)

with differential has been compared with the test performed two months ago.

The results of these tests are the following:

A B

___________________________________________

Hb 86 g/L 125 g/L

Erythrocytes 3.3*1012/L 4.1*1012/L

MCH 0.78 0.91

Reticulocytes 0 0.4%

Platelets 100*109/L 260*109/L

Leukocytes 36*109/L 12*109/L

Myeloblasts 65% 3%

Promyelocytes 10% 6%

Myelocytes 0 10%

Metamyelocytes 6% 15.5%

Band neutrophils 3.5% 12.5%

Segmented neutrophils 11% 36%

Eosinophils 2% 5%

Basophils 0 3.5%

Lymphocytes 2.5% 6%

Monocytes 0 2.5%

Questions:

Page 69: Collected PathPhys Situations 2010

69

1. Make an analysis of the results of the tests A and B. Which of the tests has

been done recently and which one some time ago? Substantiate your answer.

2. Define the clinical significance and the pattern of hematologic shift in the

patient's blood.

3. What are the causes of the hematologic shift and acute deterioration of the

patient's condition?

4. What mechanisms underlie the development of hemorrhages, fever,

enlargement of the spleen, the liver, and the lymph nodes; pain in the heart?

PATHOPHYSIOLOGY OF HEMOSTASIS Questions to prepare for classes and examination:

1. The system of hemostasis: components and their function.

2. Typical forms of pathology of the hemostatic system.

3. Thrombotic syndrome: its main causes, mechanisms of development,

manifestations and consequences.

4. Hemorrhagic syndrome: causes, mechanisms of development,

manifestations and consequences.

5. Causes and mechanisms of thrombocytopenia and thrombocytopathia.

6. Hemophylias and von Willebrand disease.

7. Thrombohemoarrhagic conditions. The syndrome of disseminated

intravascular coagulation: stages, mechanisms of development,

manifestations and consequences.

8. Laboratory parameters used to characterize hemostatic disorders:

bleeding time, prothrombin time, partial thromboplastin time.

N 80

A 62-year-old patient was admitted to hospital with a diagnosis of

"myocardial infarction of the left ventricle and ischemic stroke associated with

systemic atherosclerosis". On the next day his state was aggravated by

development of thromboembolism of the left popleteal artery and acute renal

insufficiency with signs of uremia. On the third day of his stay in the hospital

the patient's condition was complicated by the emergence of multiple

subcutaneous hemorrhages and gastrointestinal bleeding. Blood tests

showed marked thrombocytopenia, a considerable decrease in fibrinogen and

Page 70: Collected PathPhys Situations 2010

70

prothrombin levels, increased concentration of heparin and fibrin split

products, and the enhancement of the blood fibrinolytic activity.

Questions:

1. What forms of pathology are obseved in this patient? Substantiate your

answer.

2. Which of these forms of pathology caused the development of the

myocardial infarction and stroke?

- renal insufficiency?

- thromboembolism of the popleteal artery?

- hemorrhages and bleeding?

3. How can you explain the development of the formaly opposite forms of

pathology in this case, such as myocardial infarction and stroke on the one

hand, and hypocoagulative disorders – petechiae and gastrointestinal

bleeding on the other hand?

4. Can we consider these various forms of pathology as parts of a common

disorder, or do they occur independantlly in this patient? Substantiate your

answer.

N 81

Parents of a 3-year-old boy noticed that he often had inflammation of his

knees and elbows. They told the doctor that their boy often has extensive

nasal bleedings and develops large bruises after he falls down or hurts

himself. During teething he had transient gums bleeding. On examination:

physical and mental development of the boy is commensurate with his

chronological age; knees and elbows have signs of inflammation (swelled,

hyperemic, and tender). The results of the complete blood count test are

unremarkable; coagulogram shows normal capillary bleeding time and

prothrombin time, but considerably increased partial thromboplastin time.

Questions:

1. What pathology can be suspected in the patient? Substantiate your answer.

2. What tests should be done to make an accurate diagnosis?

3. What is the likely cause of this pathology?

4. Describe the variants of this pathology and their mechanisms.

Page 71: Collected PathPhys Situations 2010

71

5. Explain mechanisms underlying each of the symptoms observed in the

patient.

N 82

A 50-year-old female patient K. with decompensated insufficiency of the

aortic valve underwent an implantation of the prosthetic valve. During the

surgery extracorporeal circulation was employed. Three weeks later the

patient's condition deteriorated. She developed marked dyspnea, high fever;

complained of a pain in the heart. For this reason one more operation was

performed to replace the current prosthesis and implant a new one.

Examination of the removed valve prosthesis showed thrombotic vegetations

with microbial colonies on its surface. A day later the patient died. At autopsy:

multiple focal hemorrhages in the brain and other organs, signs of systemic

vasculitis, mural thrombi; intravascular conglomerates of formal particles and

fibrin sheath were observed.

Questions:

1. What disturbances of the blood clotting system are present in this case?

What syndrome may be suspected on the ground of cinical signs?

2. What pathologic processes or medical interventions could have caused the

given disturbances of the clotting system?

3. What are the possible mechanisms of the clotting system disorders in this

patient, and mechanisms of the observed symptoms?

N 83

A 38-year-old patient A., an alcohol addict, was admitted to hospital with

gastrointestinal bleeding and hematuria. On examination: the patient had a

tint of jaundice in the skin and mucosal membranes, the hard and nodular liver

which extended 2 cm below the right costal margin. The results of complete

blood count test were unremarkable. Blood biochemistry tests revealed an

increased concentration of conjugated and unconjugated bilirubine,

decreased levels of clotting factors II, VII, IX, X, increased plasma aspartate

transaminase (AST) activity, increased prothrombin time and partial

thromboplastin time. Suspecting a vitamin K-dependent hypocoagulative

Page 72: Collected PathPhys Situations 2010

72

state, the physician prescribed the patient a vitamin K containing medicine,

but this treatment had no positive effect.

Questions:

1. What forms of pathology developed in the patient? Substantiate your

answer.

2. Which of them are primary ("triggering"), and which are

secondary? What is the cause-effect relationship between these forms of

pathology?

3. What are the causes and mechanisms of coagulopathy in this patient?

What role does chronic alcohol intoxication play in it? Substantiate your

answer.

4. Why was the vitamin K treatment ineffective in this case? What treatment

would be appropriate to this patient?

N 84

A 65-year-old patient N. with unresectable gastric carcinoma was admitted

to hospital with signs of significant worsening of his condition. He complained

of severe dyspnea after a minor physical exertion, pain on the right lateral side

of the chest, non-productive cough, and fever. Six hours after the admission

he presented with signs of angina pectoris that were resistant to

nitroglycerine, and also, cerebral vascular insufficiency manifested by right-

sided hemiparesis. The patient was delivered to the intensive care room.

When he was being transferred from the gurney to the bed he developed

emesis with blood. Considering the risk of myocardial infarction and

aggravation of the cerebral vascular problems, the physician planned to treat

the patient with anticoagulative and fibrinolytic drugs. Before the start of the

antithrombotic therapy immediate blood tests were performed, and the

following results were obtained: Hb 105 g/L, erythrocytes 3.5*1012/L,

leukocytes 12*109/L, platelets 40*109/L; decreased concentration of

fibrinogen; an increased prothrombin time and partial thromboplastin time; a

50% decrease in antithrombin III blood level. Taking into account the blood

test data the physician changed his mind about the treatment schedule.

Questions:

Page 73: Collected PathPhys Situations 2010

73

1. What forms of pathology are observed in this patient? What is the cause-

effect relationship between them? Substantiate your answer.

2. The results of the blood tests suggest the presence of coagulopathy. Define

the type of this coagulopathy and explain its cause, mechanism, and clinical

course. Substantiate your opinion by the clinical and laboratory data.

3. Why did the physician quit the antithrombotic therapy? What can you

recommend as an alternative treatment in this case?

N 85 Patient N. has been admitted to hospital with complaints of intense

retrosternal pain. The use of nitroglycerin orally did not relieve it. Moreover the

patient's condition worsened, and he was transported to the intensive care

unit. Taking into account the risk of myocardial infarction the physician

decided to administer anticoagulant and fibrinolytic drugs to the patient.

Before the start of the anticoagulant therapy an immediate blood test was

performed. The blood test data: Hb 105 g/L, erythrocytes 3.5*1012/L,

leukocytes 12*109/L, platelets 80*109/L, hypofibrinogenemia, an increase in

prothrombin time and partial thromboplastin time, antithrombin III

concentration is decreased by 50% below normal values. Having considered

these data the physician withdrew from administration of anticoagulants and

fibrinolytics.

Questions:

1. What type of hemostasis disorder developed in the patinet?

2. Is this hemostasis disorder associated with cellular mechanisms? or blood

plasma factors? or both?

3. Why did the physician change his plan with respect to anticoagulant

treatment?

N 86 A 20-year-old patient A. and a 25-year-old patient B. have come to see

their physician with similar complaints of easy formation of bruises, and

prolonged bleeding after laceration. Patient A. noted often gums bleeding

during cleaning the teeth, and both of them mentioned that after tooth

extraction the wound was oozing more than a day. Patient B. also complained

Page 74: Collected PathPhys Situations 2010

74

of swelling and inflammation of his knee joints, and pain in the knees during

walking. Patient A. informed the physician that his mother has the same

symptoms as he does, while the patient B.'s parents have no signs of

hemostatic disorders. On examination: A. shows increased capillary bleeding

time; in B. this parameter is within the normal range. Patient B. has a

significantly increased partial thromboplastin time (PTT), but in A. this

parameter is normal. In both patients prothrombin time, platelet count,

prothrombin and fibrinogen content in the blood are within normal limits.

Questions:

1. What components of the hemostatic system are abnormal in these

patients? Substantiate your answer.

2. What disorders can be suspected in patients A. and B.?

3. What additional laboratory data are required to confirm the preliminary

diagnoses, and what are their likely results?

4. What role does heredity play in manifestation of these diseases?

5. Describe the mechanisms of symptoms presented by these patients.

CORONARY INSUFFICIENCY. MYOCARDIAL INFARCTION. ARRHYTHMIAS.

Questions to prepare for classes and examination: 1. Definition of coronary insufficiency. The main cuases of coronary

insufficiency.

2. Mechanisms of injury to the myocardial cells during ischemia and

reperfusion: role of Ca2+, oxygen free radicals, and leukocytes.

3. Adaptive mechanisms developing during acute and chronic coronary

insufficiency.

4. Biochemical, mechanical and electrical changes elicited in the myocardium

during ischemia. Signs of reversible and irreversible ischemic injury.

5. The syndromes of coronary insufficiency: angina pectoris, myocardial

infarction, chronic ischemic heart disease, sudden coronary death.

6. Methods of diagnostics of myocardial infarction.

7. Cardiac arrhythmias: types, mechanisms of development, hemodynamic

consequences.

Page 75: Collected PathPhys Situations 2010

75

8. Reentry arrhythmias: conditions required for reentry, types and

mechanisms of reentry, examples of arrhythmias based on reentry.

N 87

A 46-year-old patient was admitted to the intensive care unit with

complaints of intense retrosternal pain and squeezing in the chest which

lasted 1.5 hours. During the last week the patient worked hard, slept little, and

smoked more than usually; he also drank too much coffee and tea. The

patient reported that before the current episode he had no health problems

and went in for sports. On examination: the patient's condition is guarded; he

is pale, and has evidence of acrocyanosis. Auscultation shows an

unremarkable pattern of breathing at the rate of 28 per minute, no evidence of

rales; the heart sounds are muffled and arrhythmic; blood pressure is 100/70

mm Hg. The ECG reveals recurrent atrial fibrillation at the rate of 360 per

minute, right bundle branch block; S-T segment elevation in leads I, Vl, V1-V4.

Blood test shows: leukocytes 9.2*109/L, other parameters are unremarkable.

Questions:

1. What forms of pathology developed in the patient? Substantiate your

answer.

2. What is the cause-effect association between these forms of pathology?

3. What is the most likely cause of the pathologic state accompanied by the

retrosternal pain?

4. What metabolic and electrophysiologic changes in the myocardium

promoted the development of atrial fibrillation?

5. What was the immediate cause of the patient's pathologic state?

N 88

A 56-year-old male patient B. was brought by the ambulance to the

intensive care unit. He complained of intense retrosternal pain and a pressure

sensation which lasted approximately 3 hours, and was not relieved by

repeated doses of nitroglycerine. On examination: the patient's condition is

fair; he has a hypersthenic constitution, a hyperemic face; there are signs of

acrocyanosis. Auscultation reveals the presence of rales; breathing rate is 20

per minute; the heart sounds are muffled; heart rhythm is regular, 80 beats

Page 76: Collected PathPhys Situations 2010

76

per minute; blood pressure is 90/65 mm Hg. ECG shows sinus rhythm,

elevation of the S-T segment in leads II, III, Vf. Cardiac index measured 1

month before was 3.3 L/min/m2 (normal values: 2.7-3.0 L/min/m2). Blood test

shows the following: Hb 196 g/L, erythrocytes 6.5*1012/L, leukocytes

12.0*109/L, platelets 450*109/L, ESR 15 mm/h; blood serum activity of the

creatine kinase MB fraction 70 IU/L (normal values: 0-12 IU/L); prothrombin

index 120% (normal values: 95-105%). Bone marrow examination suggests

the presence of polycythemia vera.

Questions:

1. What forms of pathology developed in this patient?

2. What are the likely causes of each of these forms of pathology? Explain the

possible association between these forms.

3. What mechanisms contribute to heart symptoms in this patient?

4. Is there association between the heart pathology and polycythemia vera in

this case?

N 89 A 58-year-old man, who suffered from ischemic heart disease, took several

tablets of nitroglycerine to alleviate prolonged (more than 30 minutes)

retrosternal pain which radiated to the epigastrial area. However, the pain

persisted and became burning. Soon after this, the patient started to feel

weakness, dizziness, and nausea. The ambulance physician, called in by the

patient's relatives, suspected a myocardial infarction and gave the patient an

injection of a combination of drugs including an anesthetic, an antiarrhythmic,

and an anticoagulant. Two hours after the onset of the heart attack, despite

vigorous treatment, the patient developed arterial hypotension (blood

pressure 70/55 mm Hg), low rate of urinary excretion (25% of the normal

value per hour), acrocyanosis, dyspnea, rales during auscultation. Blood test

showed the following: erythrocyte count 6.0*1012/L, leukocytes 11.9*109/L.

Questions:

1. Define the pathologic state developed in the patient:

- before the arrival of the ambulance?

- 2 hours after the onset of the heart attack?

Substantiate your answer.

Page 77: Collected PathPhys Situations 2010

77

2. Which factors caused the heart pathology in each of these pathologic

states?

3. Both of these pathologic states can cause cardiac insufficiency. What are

the main mechanisms of its development in each of the cases?

4. What actions could be undertaken to prevent cardiac insufficiency and its

consequences in these pathologic states?

N 90 A 52-year-old patient S. has been admitted to the hospital's Cardiology

Department. He had a history of syncope preceded by a prolonged period of

recurrent episodes of palpitations. They were usually accompanied by

weakness, dizziness, and choking sensation. The day before the admission

the patient incurred a severe psychologic stress related to death of his close

relative, who was heavy smoker, and suffered from ischemic heart disease.

On examination: blood tests are unremarkable. Holter ECG monitoring

shows 11 episodes of arrhythmia lasting from 20 to 60 sec. During these

episodes P wave is poorly exhibited and occasionally superimposes on QRS

complex. The frequency of P wave is usually about 70 per minute; QRS

complexes are regular, and occur at rates of 190 per min; they are often

distorted and look similar to ventricular extrasystoles. Episodes of arrhythmia

are accompanied by a decrease in systemic blood pressure.

Questions:

1. Define the form of heart pathology developed in the patient. Substantiate

your answer.

2. What are the possible causes of this pathology?

3. What electrophysiologic mechanisms underlie the ECG changes observed

in the patient? What metabolic disorders in the myocardium could promote the

observed ECG changes?

4. Is there a risk of sudden death during the described episode of heart

dysfunction? If there is, what could be the immediate cause of death?

N 91 A 62-year-old patient K. experienced myocardial infarction in the

posterosuperior area of the left ventricle and interventricular septum 5 days

Page 78: Collected PathPhys Situations 2010

78

ago. Suddenly, he felt weakness, dizziness, nausea, turned pale, and lost

consciousness. ECG shows regular atrial waves at rates of 109 per min, and

regular ventricular rhythm with a frequency of 42 per min; there is no

association between P wave and QRS complexes; systemic blood pressure is

65/50 mm Hg.

Questions:

1. Define the form of cardiac pathology developed in the patient? Substantiate

your answer using the clinical data and ECG.

2. What electrophysiologic mechanism underlies this form of pathology?

3. What metabolic changes and in what area of the heart can produce the

electrophysiologic disturbancies developed in the patient? Substantiate your

answer.

4. Describe the principles of adequate treatment in this case.

N 92

A 58-year-old patient M. was hospitalized 3 days ago with a diagnosis of

myocardial infarction in the middle one third of the anterior wall of the left

ventricle. Suddenly, he felt weakness, dizziness, discomfort in the heart. ECG

registered during the attack revealed regular atrial "sawtooth" waves at rates

of 350 per min; each two atrial waves were accompanied by a QRS complex

of a normal shape. Systemic blood pressure was 90/50 mm Hg. The physician

gave the patient an intravenous infusion of beta-adrenoblockers and calcium

antagonists. Twenty minutes later the patient's condition improved. His blood

pressure increased up to 110/75 mm Hg, but changes in the ECG were still

present, though the rate of atrial waves and QRS complexes decreased. The

next day after meal a similar episode of cardiac disorder occurred. In this case

pharmacologic treatment had no effect. The patient was urgently transported

to the intensive care unit.

Questions:

1. Define the type of cardiac disorder developed in the patient. Substantiate

your answer using the clinical data and ECG.

2. What electrophysiologic disturbances and in what area of the heart underlie

changes in ECG in this case? Substantiate your answer.

Page 79: Collected PathPhys Situations 2010

79

3. What metabolic disorders in the myocardium could result in the indicated

electrophysiologic changes?

4. Is the described episode of cardiac disorder life-threatening? Explain why, if

it is.

HEART FAILURE Questions to prepare for classes and examination:

1. Definition, general causes and classification of heart failure.

2. Myocardial and nonmyocardial cardiac insufficiency.

3. Adaptive reactions during acute and chronic heart failure: Frank-Starling

mechanism, myocardial hypertrophy, redistribution of cardiac output, salt

and water retention, activation of the sympathetic system.

4. Pathogenesis of the myocardium decompensation during hypertrophy.

5. Structural and hemodynamic abnormalities in heart failure.

6. Clinical features of heart failure: the syndromes of cardiac edema and

cachexia.

N 93* A 40-year-old patient Z. suffers from tuberculosis. He has come to see his

physician with complaints of dyspnea, pains in the right upper quadrant, and

subfebrile fever. These symptoms emerged 4 weeks ago and were

progressing. On examination: the face is pale and rounded, orthopnea is

evident (the patient is sitting upright with his hands resting upon chair; this

posture is necessary to ease breathing); the area of cardiac dullness is

increased to the left and to the right by 2 cm, heart rate is 100 per min, blood

pressure is 90/60 mm Hg. On auscultation: muffled heart sounds, crackles at

the base of the lungs. Breathing rate is 26 per min. The neck veins are

distended; the liver is tender on palpation, and extends 3 cm below the costal

margin; there is trace pretibial and feet edema. X-ray examination of the chest

shows the globular shadow of the heart.

Questions:

1. Does the patient have signs of cardiac insufficiency? Substantiate your

answer.

Page 80: Collected PathPhys Situations 2010

80

2. What additional tests are required to specify the form of the heart pathology

in this case?

3. Make a conclusion about the form of cardiac insufficiency in this patient.

N 94

A 48-year-old patient N. suffered from polycythemia vera for 7 years. Two

years ago he was examined by the cardiologist, who found a hyperkinetic type

of his blood circulation: stroke volume was increased by 35%, and systemic

vascular resistance decreased by 30% compared with the corresponding

normal values; systemic venous return to the heart was significantly elevated.

Two days ago the patient was admitted to hospital with complaints of

weakness, dizziness, dyspnea in the supine position (orthopnea), nocturia,

palpitations, lower extremities edema in the evening, poor appetite. On

examination: the patient is pale; he has evidence of acrocyanosis; the heart

size is increased. Ausculation reveals displacement of the point of maximal

impulse down and to the left, and the presence of rales. The results of blood

tests are the following: Hb 205 g/L, erythrocyte count 7.0*1012/L, platelets

520*109/L, leukocyte count 14.5*109/L; few normoblasts are present. Cardiac

index is 2.5 L/min/m2 (normal values: 2.7-3.0 L/min/m2); systemic vascular

resistance is reduced by 17% compared to the normal. An X-ray investigation

of the chest demonstrates an increase in the size of the heart cavities and

signs of congestion in the lungs.

Questions:

1. What forms of pathology developed in the patient? Substantiate your

answer.

2. What specific form of pathology was the immediate reason for the patient's

hospitalization? Explain your opinion.

3. What are causes and mechanisms of this pathology in the given patient?

4. What mechanisms underlie each of the symptoms in the given patient?

N 95

A 26-year-old female patient, who suffered from rheumatic fever since

childhood, was admitted to hospital with complaints of recurrent pain in the

heart, palpitations, particularly in the supine position, dyspnea, syncope.

Page 81: Collected PathPhys Situations 2010

81

These symptoms appeared at the end of pregnancy and increased after child-

birth; they were provoked by strenuous exercise. Blood tests show the

following: Hb 129 g/L, erythrocytes 4.1*1012/L, leukocytes 4.2*109/L; platelets

250*109/L; blood serum cholesterol is within the normal range. Blood pressure

is 165/50 mm Hg; cardiac dullness is increased to the left; apical impulse is

enhanced and displaced down and to the left. On auscultation early diastolic

murmer is heard in the right second intercostal space. ECG reveals the signs

of the left axis deviation; during physical exercise on bicycle ergometer the S-

T segment is depressed. An X-ray examination shows an increase of the

heart size, particularly to the left, and enlargement of the proximal portion of

the aorta.

Questions:

1. What forms of pathology developed in the patient? Substantiate your

answer.

2. What is the cause-effect association between these forms of pathology?

Substantiate your answer.

3. What mechanisms underlie each of the symptoms in this patient?

4. Are there any signs of the adaptive mechanisms compensating for the heart

insufficiency in this case? If there are, describe them, explain how do they

operate, and what is their significance.

N 96 A 42-year-old patient with acquired mitral valve insufficiency, associated

with rheumatic heart disease, presents with the following symptoms: an

increased area of the cardiac dullness, enlargement of the liver, accumulation

of fluid in the abdominal cavity, decreased daily urinary volume, a significant

edema of the lower extremeties, and an increased level of aldosterone in the

blood.

Questions:

1. What type of cardiac insufficiency developed in this patient? Substantiate

your answer.

2. What are the main mechanisms of decompensation of the myocardial

contractile function in this case?

Page 82: Collected PathPhys Situations 2010

82

3. What mechanisms underlie the development of aldosteronism in cardiac

insufficiency?

4. Taking into account the described symptoms, explain the pathogenesis of

edema in this patient.

N 97 Patient M. suffers from arterial hypertension. He is visiting his physician

with complaints of recurrent dyspnea with difficult inspiration, especially during

physical exertion. Several days before he had a night attack of severe

inspiratory dyspnea accompanied by shortness of breath and fear of death.

He called in an ambulance. When it came the ambulance physician gave him

an injection of a medicine that quelled the attack. The physician made a

preliminary diagnosis of asthma based on the patient's history. During the

examination in hospital the patient's blood pressure is 155/120 mm Hg, X-ray

investigation shows an increase in the size of the left ventricle of the heart.

Question:

1. What type of asthma (bronchial or cardiac) developed in the patient?

Substantiate your view.

2. What is the immediate cause of it? Is this cause related to overexertion of

the heart? If it is, which of the ventricles is affected, and by what factor:

preload or postload?

3. What is the pathological basis (on the cellular and subcellular levels) of

impairment of the cardiac contractility?

4. What are the main principles of treatment of heart failure?

DISORDERS OF SYSTEMIC BLOOD PRESSURE. ARTERIAL HYPOTENSION AND HYPERTENSION

Questions to prepare for classes and examination:

1. Principles of physiological control of systemic blood pressure.

2. Typical forms of disorder of systemic blood pressure.

3. Arterial hypotension: essential and symptomatic.

4. Types and mechanisms of development of symptomatic arterial

hypotension.

5. Definition and principles of classification of arterial hypertension.

Page 83: Collected PathPhys Situations 2010

83

6. Renal arterial hypertension: pathogenesis of renovascular, renoprival,

renal parenchymal hypertension.

7. Pathogenesis of endocrinogenic and neurogenic arterial hypertension.

8. Essential hypertension: etiology and pathogenesis.

9. Clinical variants, stages and principles of treatment of essential

hypertension.

N 98

A 39-year-old patient visited his physician with complaints of regular

headaches which he usually treated by aspirin. According to the patient's

history the headaches first appeared 4 months ago, and before that the

patient had no health problems, though occasionally he had elevations of the

blood pressure. On examination: the patient's skin and visible mucosal

membranes are hyperemic; blood pressure is 185/125 mm Hg; the liver is

enlarged: the shins and soles are swelled. The patient was addmitted to

hospital. Blood test performed in the hospital showed erythrocytosis (r.b.c.

count 7.2*1012/L), leukocytosis (w.b.c. count 11.2*109/L), thrombocytosis

(platelets count 650*109/L).

Questions.

1. What forms of pathology developed in the patient? Substantiate your

answer.

2. Define the type of arterial hypertension which the patient suffers from,

describe its pathogenesis.

3. How can the observed changes in the blood contribute to the

pathogenesis of arterial hypertension in this patient? Explain the mechanism.

N 99

A 36-year-old patient M. visited his physician with complaints of recurrent

episodes of severe headache, blurred vision, face flushing, sweating,

dizziness, palpitations, and anxiety. These symptoms usually occur during

physical exertion or in situations of intense psychologic stress. At rest the

patient's blood pressure is 136/85 mm Hg, heart rate 80 beats/min. Blood and

urine tests are unremarkable. After a loading exercise the patient's blood

pressure rose up to 230/165 mm Hg, heart rate 188 beats/min, glucose blood

Page 84: Collected PathPhys Situations 2010

84

level 190 mg/dl. In the urine sample, collected immediately after the exercise,

there is an increased concentration of catecholamines and their metabolites.

An X-ray investigation of the abdominal region shows enlargement of the right

adrenal gland.

Questions:

1. Name and describe forms of pathology developed in the patient.

2. What is the mechanisms of the increased blood pressure in this patient?

3. What mechanisms contribute to the development of each of the presented

symptoms?

N 100

A 46-year-old male patient was admitted to hospital with complaints of

palpitations, headache, sleep disturbances, nervousness, irritability, and

fatigability. On examination in the hospital: blood pressure is 160/100 mm Hg,

heart rate 92 beats/min; stroke volume is increased; vessels of the eye fundus

are constricted, their walls are thickened; angiography of the kidneys and

adrenal glands is unremarkable; the level of catecholamine products and

corticosteroids in the urine is elevated.

Questions:

1. What forms of pathology developed in this patient? Substantiate your

answer.

2. Which of them appears to be the primary factor of pathology in this patient?

What is the likely cause and mechanism of development of this form of

pathology? Substantiate your answer using the given clinical data.

3. What treatment approaches are relevant in this case?

N 101

A 25-year-old male sportsman visited his physician with

complaints of sudden muscle weakness, headache, dizziness during physical

exercise, increased frequency and volume of urination. These symptoms

appeared 3 weeks ago. On examination: blood cell count is within the normal

range, there is evidence of hypernatremia and hypokalemia; blood pressure is

165/110 mm Hg; daily urine volume is 3500 ml, hyposthenuria; an X-ray

examination of the chest shows increased size of the heart.

Page 85: Collected PathPhys Situations 2010

85

Questions:

1. What forms of pathology may be suspected in this patient? Substantiate

your answer.

2. What additional data are required to make an unequivocal conclusion about

the patient's disease? Explain clinical significance of these data.

3. What mechanisms underlie the symptoms presented by this patient?

N 102

A 58-year-old female patient K. suffers from arterial

hypertension. During the last 1.5 months she noticed her gaining body weight,

emergence of cold intolerance, numbness and pain in the gastroknemius

muscle on walking, and then at rest. The latter usually occurred at night

causing sleep disturbances. Five months ago an erosion appeared in the

lower one third of the right shin; later it transformed into an ulcer which was

painless and resistant to treatment. She had a persistent subfebrile fever

(37.2-37.4oC body temperature).

During her visit to the doctor the patient also complained of a dry mouth,

thirst, drinking much water (up to 4-5 L daily), frequent and excessive

urination. On examination: the skin of the shins is dry, pale, and cold;

palpation of the sole and the area behind the knee fails to reveal pulsation of

arteries. Blood tests show increased levels of cholesterol and fibrinogen;

elevated platelet count; fasting glucose blood level is 180 mg/dl.

Questions:

1. What form of pathology, in addition to arterial hypertension, can be

suspected in this patient? Substantiate your answer by the given clinical and

laboratory data.

2. What can be the possible cause of this form of pathology?

3. What mechanisms underlie the development of this pathology and the

presented symptoms?

4. Is there any cause-effect association between the observed pathology and

the shin ulcer in this patient? If there is, characterize this association. If there

is not, explain the development of ulcer in this case.

N 103

Page 86: Collected PathPhys Situations 2010

86

A 48-year-old patient K. visited his physician with complaints of recurrent

episodes of intense headache in the occipital area, blurred vision,

accompanied by rigor, nausea, and, usually, vomiting. These symptoms were

provoked by prolonged periods of psychologic stress. During the last episode,

which had ocurred 2 weeks ago, the physician of the ambulance had found

the increased blood pressure, 195/110 mm Hg, in the patient's right hand, and

200/115 mm Hg in the left hand; signs of coronary insufficiency and

paroxysmal ventricular tachycardia. On examination: blood pressure was

195/115 mm Hg; the patient experiences intense weakness, dizziness,

nausea; he has problems with coordination due to weakness of the right hand

and leg; his speech is slurred, responses are slow and often off the mark.

Questions:

1. What forms of pathology developed in this patient? Substantiate your

answer.

2. Which of these forms of pathology is the most significant in determining the

patient's state? What are their likely causes and mechanisms of

development?

3. Does the cause-effect association exist between the mentioned forms of

pathology? If it does, explain the point of this association.

N 104

A 57-year-old patient A., the chief of a big trading company, was

hospitalized after a profilactic examination, which disclosed the following:

blood pressure 170/100 mm Hg, the heart rhythm is regular, 89 beats per

minute, the circulating blood volume is 20% higher than normal, the heart area

is increased to the left, the apical impulse is accentuated. ECG reveals the

signs of the left ventricular hypertrophy. During funduscopy attenuated retinal

arterioles and increased tortuosity of the microvascular network are observed.

The breathing rate is 21 per min. Blood tests show the following: erythrocytes

6.0*1012/L, Hb 158 g/L, leukocytes 4.0*109/L, platelets 330*109/L;

hypernatremia, the aldosterone levels are within the normal range,

hypercholesterolemia. The patient is emotional, nervous, denies smoking.

Questions:

Page 87: Collected PathPhys Situations 2010

87

1. What form of pathology developed in the patient? Substantiate your

answer.

2. What is the likely cause and the main stages of pathogenesis of this

pathology?

3. How can you explain the presence of hypernatremia and an increased

blood volume in the setting of the normal aldosterone levels?

4. What pathologic state can be suspected on the basis of tachycardia,

hurried breathing and erythrocytosis? What are the causes and significance of

these symptoms?

N 105

A 40-year-old patient has been admitted to hospital with arterial

hypertension. Blood pressure 175/115 mm Hg. The patient complains of

muscle weakness and headaches. Laboratory testing shows polyuria, severe

hypokalemia, increased content of 17-hydroxycorticosteroids in the urine.

Ultrasound investigation revealed a mass in the right adrenal/

Questions:

1. What is the likely cause of arterial hypertension in this patient?

2. What are the presumable mechanisms of the sustained increase in blood

pressure in this case?

3. What additional tests are needed to specify the diagnosis? Describe their

likely results which could confirm your preliminary diagnosis.

PATHOPHYSIOLOGY OF RESPIRATORY SYSTEM Questions to prepare for classes and examination:

1. Typical forms of disorder of the respiratory system:

- disorders of alveolar ventilation,

- disorders of pulmonary perfusion, - disorder of ventilation-perfusion matching,

- disorder of diffusive capacity of the alveolar-capillary barrier.

2. Static and dynamic parameters of breathing used to characterize

ventilation.

3. Alveolar hyper- and hypoventilation: causes, mechanisms of development,

consequences, and typical changes in breathing volumes.

Page 88: Collected PathPhys Situations 2010

88

4. Disorders of the neurogenic control of breathing. Pathological forms of

breathing.

5. Types and consequences of perfusion dicorders: pulmonary arterial hyper-

and hypotension, venous hypertension.

6. Types and mechanisms of ventilation-perfusion mismatch.

7. Diseases characterized by abnormal diffusive capacity of the alveolar-

capillary barrier. Pneumoconioses.

8. Combined disorders respiratory function. Adults respiratory distress

syndrome.

N 106

A 38-year-old patient K. has a history of heavy smoking more than 1.5

packs of cigarettes per day for the last 12 years. He complains of recurrent

bronchitis and tracheitis, persistent cough with secretions, and dyspnea during

physical exertion. On examination: the patient has asthenic body habitus, the

barrel chest; he uses accessory muscles during breathing. X-ray of the chest

shows diminished lungs vascular markings, diaphragm depression and the

bronchioles' walls thickening.

Arterial blood gases:

PaO2 83 mm Hg

PaCO2 49 mm Hg

Oxygen carrying capacity 19.0 vol%

SaO2 96.1%

The results of spirometry (% to normal values):

Total lung capacity 119%

Vital capacity 80%

Inspiratory reserve volume 86%

Expiratory reserve volume 88%

Functional residual volume 112%

Residual volume 114%

Tiffeneau ratio(FEV1/FVC) 85%

The patient has signs of diminished diffusive capacity of the alveolocapillary

membrane; his breathing rate is 20 per min. After injection of a bronchodilator

drug aminophylline (euphylline) the value of Tiffeneau ratio increases by 7%.

Page 89: Collected PathPhys Situations 2010

89

Questions:

1. Does the patient have signs of impaired alveolar ventilation? If he does,

what type of hypoventilation is observed: obstructive or restrictive? What is

the cause of this disorder?

2. Are there signs of the lung perfusion disorder in this case? If there are,

explain its possible causes and mechanisms of development.

3. Are there signs of increase in the volume of small airway closure? If there

are, explain its cause. If there are no, substantiate your point of view.

4. Make a general conclusion about the condition of the respiratory function in

this patient?

N 107* A 36-year-old patient K., a mine worker, has been admitted to hospital with

a preliminary diagnosis of silicosis. He complains of dyspnea, more severe on

walking and physical exercise; sustained cough (occasionally with secretions),

chest pain.

___________________________________________________________

Arterial blood gases:

PaO2 90 mm Hg

PaCO2 40 mm Hg

Oxygen carrying capacity 19.2 vol%

SaO2 94.3%

Spirometry:

Forced vital capacity (FVC) 4.2 L

FVC in % to normal 92%

Forced expiratory volume in 1 s (FEV1) 2.6 L

Tiffeneau ratio (FEV1/FVC) ?

Pulmonary minute volume (% of normal) 124%

Additional data:

Breathing rate 19/min

___________________________________________________________

The voluntary hyperventilation test yields the PaO2 value of 92 mm Hg.

Questions:

Page 90: Collected PathPhys Situations 2010

90

1. Does the patient have disturbances of respiratory function? Describe the

signs of it, if he does.

2. Does the patient have disturbances of alveolar ventilation? If he does,

define its type (obstructive or restrictive).

3. Taking into account the preliminary diagnosis of pneumoconiosis how to

test the diffusive capacity of the lungs in this patient?

4. Make a final conclusion about the type of respiratory system disorder in

patient K.

N 108

A 56-year-old patient K. experienced myocardial infarction 4 weeks before.

He complains of progressive dyspnea, accompanied by a cough with scanty

blood-stained sputum. ECG demonstrates signs of the former myocardial

infarction. Evaluation of the respiratory function shows the following:

Breathing rate 26/min

Forced vital capacity (% of the normal value) 76

Total lung capacity (% of the normal value) 70

Pulmonary minute volume (% of the normal value) 140

Tiffeneau ratio 76%

Questions:

1. Does the patient have signs of impaired alveolar ventilation of the

obstructive type or restrictive type? Substantiate your answer.

2. Does the patient have signs of the pulmonary perfusion disorder? If he

does, describe its causes and mechanisms of development.

3. Are there any signs of diffusive capacity disorder? What tests can be used

to assess the diffusion of gases across the alveolocapillary membrane?

4. Make a final conclusion about the state of the respiratory system in this

patient.

N 109

A 65-year-old patient A. visited his physician with complaints of choking

sensation that emerges when he falls to sleep. Choking attacks are

accompanied by fear of death and severe palpitations, which are sometimes

followed by retrosternal pain radiating to the left scapula. The use of

Page 91: Collected PathPhys Situations 2010

91

nitroglycerin during choking attacks and pain in the heart had no effect.

However, if the patient wakes up and makes several deep voluntary

inspirations the unpleasant sensations subside. Blood test data and ECG are

unremarkable. The patient has a history of viral infection complicated by signs

of polyneuropathy; in his childhood he had polymyelytis.

Questions:

1. What form of pathology of the respiratory system does the patient suffer

from? Substantiate your answer.

2. What is the possible cause of this pathology? Can you define the level at

which the control of the ventilatory function is lost: respiratory center, afferent

pathways, efferent pathways? Substantiate your point of view.

3. Why is pulmonary disorder in this patient accompanied by episodes of

angina-like pain in the heart?

4. Why do choking episodes never occur when he is awake?

N 110

A 36-year-old patient K. has been admitted to hospital with complaints of

dyspnea, exacerbating on walking and physical exercise,and persistent

cough. Examination shows cyanosis of the mucous membranes, rhonchi

during auscultation of the lungs, hyperresonance on percussion of the chest.

Testing of pulmonary function and arterial blood gases reveals the following:

Arterial blood gases:

PaO2 84 mm Hg

PaCO2 50 mm Hg

Oxygen-carrying capacity 19.8 vol%

SaO2 94.4%

___________________________________________________________

Spirometry:

Total lung capacity Increased

Forced vital capacity Decreased

Inspiratory reserve volume Decreased

Expiratory reserve volume Decreased

Functional residual capacity Increased

Residual volume Increased

Page 92: Collected PathPhys Situations 2010

92

Tiffeneau ratio (FEV1/FVC) Decreased

The voluntary hyperventilation test yields the PaO2 value of 86 mm Hg.

Administration of bronchodilator drug aminophylline (euphylline) results in a

significant increase in Tiffeneau ratio in this patient.

Questions:

1. Does the patient have signs of reduced airways patency? If he does,

describe them, and characterize their diagnostic value.

2. Does the patient have signs of impaired elastic recoil of the lung tissue?

Describe them, if he does.

3. Can we suspect the disturbance of diffusive capacity in this patient?

Substantiate your answer.

4. Make a final conclusion about the state of respiratory function in this

patient.

N 111

A 56-year-old patient M. has been repeatedly admitted to the therapeutic

department of hospital with complaints of dyspnea during moderate physical

exertion, a nonproductive cough, episodes of choking, associated with cough

and wheezing which is more marked on expiration. Testing of respiratory

function and arterial blood gases reveals the following:

PaO2 90 mm Hg

PaCO2 30 mm Hg

Pulmonary minute volume (% of the normal value) 119%

Forced vital capacity (FVC) 3.6 L

FVC (% of the normal value) 86%

Forced expiratory volume in 1 s (FEV1) 2.1 L

Tiffeneau ratio ?

Residual volume/Total lung capacity

(% of the normal value) 110%

Maximal expiratory flow rate

(% of the normal value) 98%

After administration of bronchodilator drug aminophylline (euphylline)

Tiffeneau ratio increases by 15%. The cough at the end of a wheezing

episode produces thick, stringy mucus taking the form of casts of the distal

Page 93: Collected PathPhys Situations 2010

93

airways (Curschmann's spirals). Blood test reveals the following: Hb 136 g/L,

erythrocyte count 5.5*1012/L, leukocyte count 9*109/L, eosinophilia. The chest

roentgenogram shows increased retrosternal translucency.

Questions:

1. Are there some data in favor of changes in compliance (increased or

decreased) of the lung tissue? Substantiate your answer.

2. Can we suspect a disorder of alveolar ventilation of the

obstructive type in this case? If we can, what are its causes and mechanisms?

3. Does the patient have signs of increased volume of the small airway

closure? If he does, explain the mechanism of this phenomenon.

4. Make a final conclusion about the state of respiratory function in this

patient.

PATHOPHYSIOLOGY OF DIGESTION Questions to prepare for classes and examination:

1. Typical forms of functional pathology of the digestive tract: general etiology

and mechanisms.

2. Disorders of appetite, taste, and initial treatment of food in the oral cavity.

3. Disorders of swallowing: dysphagia, achalasia, diffuse esophageal spasm.

4. Disorders of digestion in the stomach: disturbance of secretory and motor

function. Dumping syndrome.

5. Disorders of digestion in the intestine: disturbance of secretion, motor

function, and absorption.

6. The syndrome of malabsorption: etiopathogenesis, manifestations and

consequences.

7. Pathogenesis of peptic ulcer of the stomach and duodenum.

8. Disorders of excretory function of the pancreas.

N 112

A 43-year-old patient N. suffers from severe chronic gastritis and enteritis.

He has come to see his physician with complaints of recurrent diarrhea,

progressive malaise, weakness, wasting, and generalized edema. The

examination reveals the following: hypochromatic anemia, signs of multiple

vitamin deficiencies, adrenal insufficiency, and immunodeficiency state. Stool

Page 94: Collected PathPhys Situations 2010

94

test findings: the presence of undigested muscle fibers (creatorrhea),

steatorrhea, and amylorrhea. The results of suction biopsy of the small

intestine are as follows: reduced activity of the intralumenal and brush border

fractions of digestive enzymes, atrophy of the enterocytes' microvilli.

Questions:

1. Define the syndrome developed in the patient. Substantiate your answer.

2. What are the possible causes and pathogenesis of this syndrome?

3. What disturbancies of digestion can be found in this patient?

4. Describe pathogenesis of each of the symptoms found in patient N.

N 113

A 35-year-old patient D. has been admitted to hospital with complaints of

severe episodes of "gnawing" pain (burning sensation) in the epigastrium

occurring 2-3 hours after a meal. Recently the pains have aggravated, and

became accompanied by nausea and, occasionally, vomiting. Vomiting makes

him feel better. The pains in the epigastrium also occur at night and wake him

up; they are relieved by food and alkalies. The patient reports about increased

nervousness and irritability; he acknowledges heavy smoking and excessive

alcohol consumption. Eight months ago he divorced and moved to a hostel

near his place of work. During the last months he lost 8 kg of his body weight.

On the basis of the patient's complaints and laboratory data the physician

made a diagnosis and prescribed a treatment.

Questions:

1. What form of pathology does the patient suffer from? Substantiate your

opinion.

2. What laboratory tests are required for accurate diagnosis, and what could

be their possible results?

3. What is the most likely cause of pathology in this case, and what are its

most significant risk factors in the given patient?

4. Explain the mechanisms of pain (burning) in the epigastrium, and severe

weight loss in this patient.

N 114

Page 95: Collected PathPhys Situations 2010

95

A 46-year-old patient B. has come to see his physician with complaints of

progressive pain in the periumbilical region of the abdomen. These episodes

usually occur after meal, and the pain radiates to the back. He also complains

of anorexia, recurrent diarrhea, nausea, frequent vomiting, weakness,

sensation of fullness or the presence of a foreign body in the epigastrium. Last

year the patient lost 12 kg of body weight. During the last 7 months he began

to feel persistent thirst (drank about 5 L of fluid daily), and frequently urinated.

The patient reports alcohol abuse during the last 15 years. Ten years ago

after an episode of excessive drinking he experienced an acute attack of

pancreatitis, and was treated in hospital. Later he had three episodes of acute

abdominal pain with dyspepsia, but did not see his physician or take any

treatment. He continued excessive drinking and did not keep to any specific

diet.

Laboratory testing shows the following: blood glucose level 15 mmol/L,

decreased glucose tolerance, urine glucose 4%, 24-h urine volume 3-4 L;

significant steatorrhea, 5-fold decrease in the maximum trypsin activity and

low bicarbonate concentration in the pancreatic juice. The results of the X-ray,

ultrasonic investigation, computer tomograpgy of the pancreas: diffuse

infiltration of the gland, foci of the increased density (calcareous deposits?).

Questions:

1. What forms of pathology can be distinguished in this patient?

2. What are the causes of each of these forms? Substantiate your answer.

3. Is there any pathogenic association between these forms of pathology. If

there is, what is their most likely sequence of development?

4. Explain mechanisms of the following symptoms in this patient: severe

wasting, anorexia, weakness, bloating, nausea.

PATHOPHYSIOLOGY OF THE LIVER. JAUNDICE. Questions to prepare for classes and examination:

1. General causes and mechanisms of liver dysfunction.

2. The syndrome of hepatic insufficiency. Typical disorders of carbohydrate,

aminoacid, protein, and lipid metabolism in hepatic insufficiency; failure of

detoxification function.

3. Causes, types and pathogenesis of hepatic coma.

Page 96: Collected PathPhys Situations 2010

96

4. Hemolytic jaundice: causes, mechanisms, clinical and laboratory

manifestations.

5. Obstructive jaundice: types, causes, mechanisms, clinical and laboratory

manifestations, consequences.

6. Hepatocellular jaundice: causes, mechanisms, stages, clinical and

laboratory manifestations.

7. Primary enzymopathic jaundices: etiopathogenesis and manifestations of

the Gilbert, Crigler-Najar, and Dubbin-Johnson syndromes.

N 115

A 46-year-old patient B. has visited his physician with complaints of right

upper quadrant tenderness, nausea, vomiting, weakness, fatigability. In the

patient's history there is an episode of jaundice at the age of seven. On

examination: the skin and conjunctiva appear jaundiced; blood pressure

110/75 mm Hg, heart rate 86 per min; the liver is enlarged and nontender on

palpation; the spleen is significantly enlarged; the stool is normally coloured;

the urine is dark. Blood test data: Hb 72 g/L, erythrocyte count 3.0*1012/L,

reticulocytes 11%; leukocyte count 3.7*109/L, white blood cell count

differential is unremarkable; ESR 9 mm/h.

Microscopy of the blood smear demonstrates anisocytosis,

poikilocytosis, and spherocytosis of erythrocytes. Osmotic fragility test shows

increased hemolysis. Blood biochemistry data: total protein 60 g/L (N:70-80

g/L), total bilirubin 5.8 mg/dl (N<1.3 mg/dl), conjugated bilirubin 0.15 mg/dl

(N<0.30 mg/dl), urobilinogen is detected; serum AST activity 52 U/L (N< 38

U/L); glucose level 75 mg/dl. Urine test data: 24-h urine volume 1450 ml,

specific gravity 1028, protein and glucose are not detectable; positive reaction

for Hb.

Questions:

1. What forms of pathology does the patient suffer from?

2. Are there signs of hepatic insufficiency in this patient?

3. What is your conclusion about the state of the bile pigment metabolism in

this patient? What is the pathological basis of changes in the bile pigment

metabolism in this case?

Page 97: Collected PathPhys Situations 2010

97

4. What changes in the bile pigment metabolism can be expected in acholia

and cholemia?

N 116

A 28-year-old patient C. has been admitted to hospital with complaints of

right upper quadrant tenderness, dyspepsia, weakness, fatigability, depressed

mood. The patient noticed that recently his stool has become clay-colored.

Two months ago the patient was bit by a dog and received injections of rabies

antiserum. On examination: the skin and conjunctiva appear jaundiced; the

liver is enlarged, soft, tender on palpation; the spleen is enlarged; signs of

moderate edema of the face, upper and lower extremities.

Blood test data: Hb 105 g/L, erythrocyte count 3.5*1012/L, leukocyte count

6.7*109/L, ESR 26 mm/h. Blood biochemistry results: hypo- and

dysproteinemia; total bilirubin 5.8 mg/dl (N<1.3 mg/dl), conjugated bilirubin 4.1

mg/dl (N<0.3 mg/dl), urobilinogen is present, glucose 75 mg/dl, serum ALT

510 U/L (N< 38 U/L), AST 495 U/L (N< 38).

Questions:

1. What forms of pathology can be distinguished in this patient? Substantiate

your answer.

2. Is there cause-effect association between these forms of pathology. If there

is, which of them is primary? Explain its possible cause, mechanisms of

development, and outcome.

3. Explain causes and pathogenesis of symptoms observed in this patient?

N 117

A 52-year-old patient B., an employee of the chemico-pharmaceutical

company, has been brought to hospital by an ambulance. On examination in

the emergency room: the patient appears drowsy and apathetic, his speech is

slow and incoherent; the tongue is covered with a yellow fur; body

temperature 36.9oC; the skin and conjunctiva appear jaundiced. There are

spider angiomata on the skin of his arms and upper torso, and erythema of

the palms. The abdomen is tensely distended due to the accumulation of

ascitic fluid making palpation of the liver difficult; there is evidence of the face

and extremities edema.

Page 98: Collected PathPhys Situations 2010

98

Results of the immediate blood test: Hb 102 g/L, erythrocyte count

3.6*1012/L, leukocyte count 4.8*109/L, ESR 35 mm/h. Blood biochemistry

data: hypo- and dysproteinemia, total bilirubin 70.6 umol/L (normal values:

8.6-20.5 umol/L), conjugated bilirubin 13.7 µmol/L (normal values < 5.14

µmol/L), cholic acids are present, plasma glucose 5.3 mmol/L (normal values:

4.4-6.7 mmol/L), increased level of ketone bodies, decreased concentration of

blood urea nitrogen (BUN), prothrombin index decreased, choline esterase

activity decreased, total creatine kinase activity normal, ALT activity

increased; HBsAg is not detectable.

Questions:

1. What forms of pathology does the patient suffer from? Substantiate your

answer.

2. What is the primary pathology in this case and what are its possible causes

and mechanisms of development?

3. Is there any association between the observed forms of pathology?

Substantiate your answer.

4. What are the possible outcomes of the patient's condition?

PATHOPHYSIOLOGY OF KIDNEYS Questions to prepare for classes and examination:

1. General causes and mechanisms of kidneys dysfunction. Disorders of

filtration, tubular reabsorption and excretion.

2. Renal insufficiency: definition, general causes, mechanisms and

manifestations.

3. Uremic syndrome: definition, typical manifestations and consequences.

Uremic coma.

4. Syndromes of acute and chronic renal insufficiency: typical features,

causes and manifestations.

5. Nephrotic and nefritic syndromes: typical features, causes and

manifestations.

6. Pyelonephritis: characteristic features, causes and pathogenesis.

7. Nephrolitiasis: causes, mechanisms of development, consequences.

8. Characteristic changes in urinalysis in syndromes of renal insufficiency.

Page 99: Collected PathPhys Situations 2010

99

N 118

A 48-year-old patient K. has been suffering from chronic diffuse

glomerulonephritis for 5 years. During the last several weeks he notes the

emergence of the following symptoms: dull aching in the region of the heart,

palpitations, generalized edema, especially prominent in the lower extremities

on walking.

Urine test data: 24-h urine volume 1100 ml, specific gravity 1042, protein

content 3.3%. The urinary sediment microscopic examination reveals the

presence of a large amount of granular and waxy casts. Blood pressure

170/95 mm Hg. Blood biochemistry data: blood urea nitrogen 6.9 mmol/L

(normal values: 3.3-6.6 mmol/L), total protein content 48 g/L (N: 70-80 g/L),

albumine 15 g/L (N:40-50 g/L), globulins 28 g/L (N: 20-30 g/L), hyperlipidemia,

hypernatremia. Acid-base status: pH 7.34, PaCO2 33 mm Hg, SB= 17.0

meq/L, BB= 36 meq/L, BE= -7 meq/L, titratable acidity 10 mmol/d, NH4+ 18

mmol/d.

Questions:

1. What forms of pathology are observed in this patient? What is the cause-

effect and chronologic relationship between these forms of pathology?

Substantiate your answer.

2. What specific physiologic processes in the kidneys are deranged to

produce excretory disorder in this patient?

3. What syndrome is characterized by the following combination of symptoms:

generalized edema, proteinuria, hypoalbuminemia, hyperlipidemia. What are

the mechanisms of each of these symptoms?

4. Are there signs of uremia in this patient? Substantiate your answer.

N 119

A 35-year-old patient M. complains of generalized edema, more evident

after sleep, back pains, fever, and body weight gain of 6 kg. These symptoms

developed 2 weeks after he had had an acute sinusitis. His urine test results

are as follows: 24-h urine volume 650 ml, specific gravity 1028, protein

content 0.1%, glucose and acetone - not detected. Microscopy of the urinary

sediment reveals 40 erythrocytes per high-power microscopic field, a great

Page 100: Collected PathPhys Situations 2010

100

amount of leukocytes, hyaline and erythrocyte casts in moderate quantities.

Blood pressure 150/110 mm Hg. Blood test data: BUN 10.4 mmol/L (normal

values: 3.3-6.6 mmol/L), total protein content 73 g/L (N:70-80 g/L),

hypernatremia, hypokalemia. Endogenous creatinine clearance 50 ml/min (N:

120 ml/min). Acid-base status: pH 7.3, PaCO2 32 mm Hg, SB= 16.5 meq/L,

BE= -8 meq/L, titratable acidity of the daily urine 8 mmol/d, NH4+ 15 mmol/d.

Questions:

1. What forms of pathology can be distinguished in this patient? What is their

cause-effect and chronologic association?

2. What factor caused injury to the kidneys?

3. Describe stages of nephropathy in this case. Define this form of

nephropathy.

4. What additional data are required to specify the diagnosis?

5. What are the mechanisms of each of the observed symptoms?

6. Does this patient have renal insufficiency? Substantiate your view.

N 120* A victim of the vehicle accident was transported to hospital 5 hours after

the accident. He was examined by a physician of the emergency service who

found multiple ribs fractures, contusions of soft tissues of the pelvis and lower

extremities with extensive hemorrhages. On admission the patient is

confused, markedly pale; he has a thready pulse, blood pressure 60/20 mm

Hg, periodic pattern of breathing. After a day of intense treatment by volume

expanders (he received 3 L of polyglukin) and infusion of 0.5 L of blood his

blood pressure increased to 110/60 mm Hg. On the next day after infusion

therapy diuresis was still absent, and during the following three days the

patient's condition remained guarded. He complained of severe headache,

dizziness, cyclic vomiting, inhibited state. The patient had short-termed

episodes of seizures, edema of the skin; bradycardia and extrasystole on

ECG records; his diuresis was at the level of 150-250 ml/day, but blood

pressure increased to 160/90 mm Hg. Blood test shows: BUN (blood urea

nitrogen) 90 mg/dl (N: 20-40 mg/dl), hyperkalemia, hypermagnesimea,

hyponatremia, hypochloremia, pH 7.30. Urine test: specific gravity 1.040, mild

Page 101: Collected PathPhys Situations 2010

101

proteinuria, myoglobinuria; sediment shows the presence of casts and few

leukocytes in the h.p.f.

On the 5-7th day in the hospital the patient developed great elevation of

diuresis (up to 2500 ml/day), and his condition markedly improved. Vomiting,

seizures and headache ceased, and the extent of edema reduced. Repeated

urinalysis shows: specific gravity 1.010-1.012, slight proteinuria, large quantity

of granular casts in the urine sediment.

Questions:

1. Define the type of renal syndrome in this patient. What are its causes?

2. What are the causes of anuria during the shock before normalization of

blood pressure in this patient?

3. Why was diuresis not restored despite extensive transfusion theraspy?

4. What are the mechanisms of symptoms observed on the 2-4th day after the

trauma?

N 121

A 22-year-old female patient K. developed pain in the back, dyspnea,

palpitations, and headache 2 weeks after she had had severe tonsillitis. She

gained 9 kg of body weight in 4 days. On examination: paleness of the face,

eyelids are edematous, the palpebral fissures are narrowed; trace pretibial

and feet edema; the area of cardiac dullness is increased. Blood pressure

140/95 mm Hg; the 24-h urine volume significantly decreased; there is marked

proteinuria. Microscopic examination of the urine sediment reveals the

presence of a great amount of erythrocytes, leukocytes, granular casts. In the

blood there are high titers of the antistreptolysin-O and antihyaluronidase

antibodies.

Questions:

1. What forms of pathology developed in the patient?

2. What is the cause-effect relationship between these forms of pathology?

Substabtiate your answer.

3. What are the cause and mechanisms of derangement of the renal function?

4. What are the mechanisms of each of the symptoms developed in this

patient? Which of the following mechanisms caused hyperhydration in this

Page 102: Collected PathPhys Situations 2010

102

patient: a decreased rate of renal excretion or extrarenal factors affecting fluid

retention in the body?

N 122

A 33-year-old patient N. had acute diffuse glomerulonephritis 4 years ago.

He is visiting his physician with complaints of recurrent headaches, malaise,

dizziness, and moderate generalized edema. Urine test results: 24-h urine

volume 3600 ml, specific gravity 1006, protein content 0.6%, glucose and

acetone – not detected. Microscopic examination of the urine sediment shows

the presence of occasional dysmorphic erythrocytes per high-power

microscopic field, a small quantity of hyaline and waxy casts. Blood pressure

160/95 mm Hg. Blood test data: BUN 11.5 mmol/L (normal values 3.3-6.6

mmol/L), total protein 56 g/L (N: 70-80 g/L). Endogenous creatinine clearance

50 ml/min (N: 120 ml/min). Acid-base status: pH=7.3, PaCO2= 36 mm Hg,

SB= 18 meq/L, BB= 36 meq/L, BE= -5 meq/L, titratable acidity of the daily

urine 12 mmol/d, NH4+= 10 mmol/d.

Questions:

1. What specific physiologic processes in the kidneys are deranged to

produce impairment of the excretory function in this patient?

2. What are the possible causes and pathogenesis of each of these

processes?

3. Can we assume the presence of renal insufficiency in this patient?

4. What are the mechanisms of polyuria, hyposthenuria, arterial hypertension,

hyperazotemia, hypo- and dysproteinemia?

N 123

A 52-year-old patient M. who has suffered from chronic glomerulonephritis

for 12 years visits his physician with complaints of recently appeared

symptoms of drowsiness in the day time and difficulty falling asleep at night,

fatigability, apathy, sensation of fullness in the retrosternal and epigastric

regions, nausea, diarrhea, and itching of the skin. On examination: blood

pressure is 165/95 mm Hg, the area of the cardiac dullness is increased to the

left, pericardial friction rub over the whole area of the heart during

auscultation. Blood test data: anemia, leukopenia, significant azotemia, hypo-

Page 103: Collected PathPhys Situations 2010

103

and dysproteinemia. Endogenous creatinine clearance is 45 ml/min (N: 120

ml/min). Urine test data: 24-h urine volume 450 ml, specific gravity 1029,

proteinuria. Microscopic examination of the urine sediment shows the

presence of 10 erythrocytes per high-power microscopic field, a great amount

of leukocytes, occasional granular and waxy casts. Acid-base status: pH=

7.32, PaCO2= 32 mm Hg, SB= 16 meq/L, titratable acidity of the daily urine 11

mmol/d, NH4+ 10 mmol/d.

Questions:

1. Define the condition developed in this patient.

2. What is the most likely cause of this condition? Substantiate your answer.

3. What are mechanisms of the psychoneurologic, cardiologic, and

hematologic symptoms, as well as of changes in the urine and acid-base

balance in this patient?

4. What condition could develop if the observed symptoms continue to

aggravate?

PATHOPHYSIOLOGY OF THE PITUITARY, ADRENALS AND SEX GLANDS Questions to prepare for classes and examination:

1. General etiology and pathogenesis of endocrine disorders.

2. Typical forms of functional pathology of adenopituitary. Hypopituitarism:

types, causes, mechanisms of development and manifestations.

Sheehan’s syndrome, dwarfism, hypogonadotropic hypogonadism.

3. Hyperpituitarism: types, causes, mechanisms of development and

manifestations. Acromegaly and gigantism; hyperprolactinemia.

4. Typical forms of disorders of neurohypophysis. Diabetes insipidus and

syndrome of inappropriate arginine-vasopressin secretion.

5. Typical forms of adrenals’ dysfunstion. Hypercortisolism: types,

pathogenesis and manifestations. Cushing’s disease and syndrome,

hyperaldosteronism.

6. Hypocortisolism: types, pathogenesis and manifestations. Acute adrenal

failure and Addison’s disease, hypoaldosteronism, syndromes of adrenal

androgen excess.

Page 104: Collected PathPhys Situations 2010

104

7. Disorders of adrenal’s medulla. Manifestations and consequences of

phyochromocytoma.

N 124

A 56-year-old man T. has met his relative after 6 years living apart. The

relative was amazed by dramatic changes in T.'s appearance: thickened and

ridged skin on the forehead, deepened frontal and nasolabial folds, the nose

and lips enlarged, the mandible and malar bones were prominent with

protrusion of orbital ridge, and the teeth became widely separated.

T. has visited his physician who suspected acromegaly, and sent the

patient's blood to laboratory to test the level of growth hormone. The results of

the test showed that growth hormone level in the T.'s blood is within the

normal range. Further examination revealed a decreased glucose tolerance,

an increased level of glucose in the blood (8.3 mmol/L); the presence of

hyperphosphatemia and glucosuria; pH of the blood 7.34, plasma

concentration of ketone bodies 2.8 mg/dl (N:0.5-2.5 mg/dl), and that of lactic

acid 2.2 mmol/L (N:0.99-1.75 mmol/L).

Questions:

1. Do you agree with the diagnosis of acromegaly in this patient? Substantiate

your answer.

2. Can we suspect the presence of diabetes mellitus in this patient? What

clinical data may support this diagnosis? What mechanisms may determine

the development of diabetes mellitus in this case?

3. What additional data are required for a final conclusion about the nature of

pathology in this patient?

4. What mechanisms underlie the symptoms presented by the patient?

N 125

A primary care physician has sent a 18-year-old male patient to hospital for

a thorough examination. On admission: the patient is 185 cm tall and weighs

69 kg; he has narrow shoulders, abnormally long legs and increased thigh

girth, high-pithched feminine voice, signs of gynecomastia, and diminished

hair growth on the trunk, limbs, and beard area. The patient has

underdeveloped musculature and decreased power (according to

Page 105: Collected PathPhys Situations 2010

105

dynamometric measuring), small penis and testes. Special tests revealed the

presence of azoospermia, low IQ, and a karyotype of 47,XXY.

Questions:

1. Name the form of pathology (or syndrome) presented by this patient, and

explain its likely cause.

2. Define the form of endocrine pathology in this patient, and explain its

cause.

3. What are the mechanisms of each of the symptoms observed in this

patient?

N 126

A 38-year-old patient M., a geologist, had tick-borne encephalitis 3 months

before during his last expedition. When he returned home from his expedition

he visited his doctor with complaints of malaise, frequent headaches,

persistent thirst (drinks daily about 6-8 L of fluid), and frequent excessive

urination. Blood test shows signs of normochromatic anemia, leukopenia,

thrombocytopenia; hematokrit is 0.28. Urine test data: 24-h urine

volume is 7.5 L, specific gravity is 1002, protein is not found, glucose and

ketone bodies - not detected. Acid-base status of the capillary blood: pH 7.37,

PaCO2 40 mm Hg, SB=22 meq/L, ketone bodies 1.1 mg/dl (N: 0.5-2.5 mg/dl),

lactic acid 1.32 mmol/L (N:0.99-1.75 mmol/L).

Questions:

1. What form of pathology does the patient suffer from? Substantiate your

answer.

2. What are its most likely cause and mechanisms of development?

3. What condition with similar manifestations should the current form be

differentiated from?

4. What are the mechanisms of the presented symptoms and changes in the

laboratory parameters?

N 127

A 40-year-old patient has been admitted to hospital with arterial

hypertension of unclear etiology. His blood pressure is 175/115 mm Hg. The

Page 106: Collected PathPhys Situations 2010

106

patient complains of muscle weakness and headaches. Laboratory testing

shows polyuria, severe hypokalemia, increased content of

17-hydroxycorticosteroids in the urine.

Questions:

1. What is the likely cause of arterial hypertension in this patient?

2. What are the presumable mechanisms of the sustained increase in blood

pressure in this case?

3. What additional tests are needed for acurate diagnosis? Describe their

likely results which could confirm your preliminary diagnosis.

N 128

A 36-year-old patient X. has visited his physician with complaints of severe

headache, dizziness, episodes of palpitation and anginal pain, sweating,

trembling and fear of death after intense physical exertion. On examination at

rest: blood pressure 140/90 mm Hg, heart rate 76 beats per min; complete

blood test and urinalysis are unremarkable. After controlled exercise the blood

pressure increases up to 230/165 mm Hg, heart rate 188 beats per min, blood

glucose 7.8 mmol/L. There is a significant increase in the urine content of

catecholamines and their metabolites. An X-ray examination of the low back

area reveals elevation of the left adrenal; size and contours of the kidneys are

unremarkable.

Questions:

1. What forms of pathology can be distinguished in this patient?

2. Is there any pathogenic association between these forms of pathology?

3. What factors cause the significant increase in systolic and diastolic blood

pressure in this case?

4. What are the mechanisms of the presented symptoms?

N 129

A 32-year-old male patient, a former professional boxer in heavy weight

category, visited his physician with complaints of progressive gaining of

weight (increased his body mass by 7 kg for 6 months), muscle weakness,

easy bruising, dizziness, headaches (more often in occipital area), recurring

Page 107: Collected PathPhys Situations 2010

107

blurring of vision (specks dancing before the eyes), persistent thirst (drinks

daily 5-6 L of fluid), and frequent excessive urination.

On examination: the patient has a hypersthenic body habitus with excess fat

deposition on the face (moon face), neck (buffalo hump), and over the

clavicles; there are purplish striae of the skin on the abdomen and excessive

hair growth on the chest and back; a large number of bruises of various colour

is observed on the upper and lower extremities. Blood pressure is 185/110

mm Hg. Blood test data: Hb 130 g/L, erythrocyte count 5.1*1012/L, leukocytes

count 10*109/L, neutrophilia, relative lymphopenia, and lack of eosinophils;

ESR 5 mm/h, glucose level 8.7 mmol/L, hypernatremia. Urine test results: 24-

h urine volume 6000 ml, specific gravity 1025, glucosuria, protein and ketone

bodies are not found; content of unbound cortisol is increased.

Questions:

1. What forms of endocrine pathology does the patient suffer from?

Substantiate your answer.

2. What is the cause-effect relationship between these forms of pathology?

Which of them is primary and secondary?

3. What are the causes and mechanisms of development of these pathologic

conditions?

4. What are the mechanisms of each of the symptoms presented by the

patient?

N 130

A 40-year-old patient B. has been transported to the emergency room by

an ambulance. On examination: the patient appears lethargic, his speech is

slowed; he complains of headache, severe weakness, pain in the abdominal

area, nausea, vomiting; body temperature 39.5oC, blood pressue 65/40 mm

Hg. He has a marked hyperpigmentation of the face, hands, lips, and skin

folds. According to the patient's report his condition became worse after he

had received news about his mother's death.

Examination in the hospital reveals sustained arterial hypotension (blood

pressure 70/50 mm Hg), hyperkalemia, hyponatremia, hypovolemia; blood

glucose 2.9 mmol/L. ECG shows the presence of extrasystoles, episodes of

atrial paroxysmal tachycardia and atrial flutter, atrio-ventricular conduction

Page 108: Collected PathPhys Situations 2010

108

disturbancies. Urine test demonstrates a decreased content of unbound

cortisol and 17-hydroxycorticosteroids. Test with intravenous injection of

corticotropin yields only a slight increase in the blood level of cortisol. A

special immunologic test shows the presence of antiadrenal antibodies. Two

years ago the patient had a gunshot wound in the back area.

Questions:

1. Define the state which the patient had at the moment of admission to the

hospital? What factor caused the development of this condition? Substantiate

your answer.

2. Define the form of pathology in this patient taking into account the results of

his examination in the hospital? What is the relationship between this form of

pathology and the state in which the patient was transported to hospital?

3. What is the likely cause and pathogenesis of this form of pathology?

4. What are the mechanisms of each of the symptoms presented by the

patient?

PATHOPHYSIOLOGY OF THYROID GLAND Questions to prepare for classes and examination:

1. Synthesis and control of secretion of thyroid hormones. Physiological

effects of thyroid hormones.

2. Hypothyroidism: types, causes, mechanisms of development and clinical

manifestations. Common hypothyroid states: myxedema, cretinism,

endemic goiter.

3. Hyperthyroidism: types, causes, mechanisms of development and clinical

manifestations.

4. Pathogenesis and manifestations of Grave’s disease, Hashimoto’s

thyroiditis, iod-basedow and Wolf-Chaikoff’s phenomena.

5. Thyrotoxic and myxedema coma.

6. Typical disorders of parathyroid gland. Causes and manifestations of

hypo- and hyperparathyroid states.

N 131

A 30-year-old female patient L. complains of fatigability, muscle weakness,

difficulty in falling asleep, palpitations, excessive sweating and heat

Page 109: Collected PathPhys Situations 2010

109

intolerance, frequent stool, and significant weight loss. Three years ago she

experienced a severe viral infection, and since that time suffered from

episodes of unexplained pain in the anterior neck in the region of the thyroid

gland. She ignored these symptoms and did not visit her doctor. On

examination: the thyroid gland is slightly increased, indurated, nontender on

palpation; heart rate 98 beats per min; blood pressure 150/80 mm Hg; signs of

the left heart ventricle hypertrophy; the skin feels warm and moist;

exophthalmos is evident. When the patient is instructed to look at the finger

moving downwards there is a moment before the upper lid catches up with the

globe (lid lag). Concentration of thyrotropin in the patient's blood serum is

decreased, while the content of IgG is increased.

Questions:

1. What form of pathology does the patient suffer from? Substantiate your

answer.

2. What is its cause and mechanisms of development? Does the increase in

serum IgG have pathogenic significance?

3. What mechanisms underlie the symptoms developed in the patient?

4. What additional laboratory tests can be performed to confirm the diagnosis?

N 132* An endocrinologist observes two patients during their follow-up visits: a 50-

year-old woman M. and her 26-year-old daughter D. Both patients have a

significant enlargement of their thyroid gland and clinical signs of

thyrotoxicosis. The results of their clinical and laboratory investigations

suggest the diagnosis of the Grave’s disease. The patient D.’s condition

improved after a treatment course, while the patient M.’s condition kept

worsening after an 8-months course of medication by thyrostatic drugs.

During reexamination M. presented with new complaints of sluggishness,

motor retardation, drowsiness during the day time and difficulty going to sleep

at night, memory problems, decreased working ability, edema of the face and

extremities and cold intolerance. These symptoms emerged after M. had

recovered from viral infection. The physician suspected the presence of

Hashimoto’s thyroiditis in M., and changed the treatment.

Questions:

Page 110: Collected PathPhys Situations 2010

110

1. Can we consider the emergence of new symptoms in M. as complication of

the thyrostatic therapy?

2. 2. What laboratory tests can be made to confirm the diagnosis in M.?

3. 3. What other clinical forms of thyroid gland pathology should be considered

4. to make the final diagnosis?

5. 4. Are there common mechanisms in the pathogenesis of Grave’s disease

and Hashimoto’s thyroiditis? 6.

7. N 133

A 46-year-old patient A. has presented with a diffuse enlargement of the

thyroid gland, an increase in basal metabolic rate, and subfebrile fever. His

heart rate is 140 beats per min; breathing rate 22 per min; he has evidence of

a fine tremor of the fingers and tongue; excessive sweating; marked

exophthalmos. Uptake of I131 by the thyroid gland is increased by 20%

compared with normal values. Blood levels of T3 and T4 are elevated; thyroid-

stimulating immunoglobulins are present in blood plasma. The patient

complains of nervousness, restlessness, irritability, difficulty in concentrating.

A 30-year-old patient B. also presents with diffuse enlargement of the

thyroid gland, which developed 2 years after he had resigned from the military

service on the atomic submarine. He has a decreased basal metabolic rate

and body temperature, diminished heart rate, and lowered levels of T3 and T4

in the blood; thyroid-stimulating immunoglobulins are not found. Uptake of I131

by the thyroid gland is decreased by 5% below the normal value. The patient

looks sluggish, apathetic, lethargic; his speech is slow and slurred; the voice

is hoarse and muffled. The physician initiated an iodine therapy in both of

these patients.

Questions:

1. What forms of the thyroid gland pathology developed in the patients A. and

B.? Substantiate your answer.

2. Explain the pathogenesis of the thyroid gland transformation in these

patients. Explain differences in the mechanisms of enlargement of the thyroid

gland in both cases?

Page 111: Collected PathPhys Situations 2010

111

3. Explain why the levels of T3 and T4 are increased in the patient A., and

decreased in the patient B., despite the augmented uptake of I131 by the

thyroid gland in both of them.

4. What are the mechanisms underlying each of the symptoms presented by

these patients?

5. Was the prescription of iodine preparation to these patients appropriate?

Can we expect the therapeutic effect iodine in both cases?

N 134

A 36-year-old female patient X. has been admitted to hospital with

complaints of recurrent attacks of retrosternal pain, palpitations, irregular

heartbeats, fatigability, weakness, excessive sweating, frequent diarrhea,

significant weight loss for the last 4 months, and subfebrile fever. According to

the patient's history, the symptoms emerged 7 months ago in connection with

emotional stress caused by family problems. On examination: the skin feels

moist and warm; there is evidence of fine tremor of the fingers and tongue;

signs of exophthalmos. ECG shows tachycardia, paroxysms of fibrillation,

elevation of the S-T segment; the thyroid gland is slightly increased; blood

pressure is 145/75 mm Hg. The physician made a preliminary diagnosis of

"neurocirculatory asthenia", and prescribed tranquilizers and cardiotonic

medicines. However, this therapy failed to improve the patient's condition.

After additional evaluations the treatment regimen was changed to

administration of propylthiouracil (a drug that blocks synthesis of thyroid

hormones). This therapy was successful.

Questions:

1. What additional data are needed for the final conclusion about the form of

pathology presented by the patient?

2. What form of pathology does the patient suffer from given the presented

symptoms and the assumed results of the laboratory tests?

3. What are possible variants of pathogenesis of this form of pathology?

4. Why did tranquilizers and cardiotonic drugs fail, and propylthiouracil

succeed in improving the patient's condition?

5. What form of the cardiac pathology developed in the patient? Explain its

pathogenesis.

Page 112: Collected PathPhys Situations 2010

112

N 135

A 42-year-old female patient N. has visited her physician with complaints

of muscle weakness, persistent drowsiness, fatigability, constipation, cold

intolerance, excessive weight gain (she gained 7 kg in the previous 2 months),

and loss of hearing ability. She noted that her voice became hoarse over the

last 3 months. Two years ago the patient had infectious parotitis and acute

thyroiditis. On examination: the patient has excessive body weight, sluggish in

her movements, answers slowly to questions; the skin feels dry and rough and

has a yellowish tint; the face is puffy, with coarse features; the upper and

lower extremities are pasty, but rebound fast after pressing with the finger tip.

The patient’s body temperature is 35.60C; blood pressure is 100/60 mm Hg;

heart rate 65 beats per min; breathing rate 13 per min. The thyroid gland is

slightly enlarged, thick and tender on palpation; Achilles and other tendon

reflexes are retarded. Special tests show a decrease in I131 uptake by the

thyroid gland, an increase in blood level of thyrotropin and a high titer of

antithyroglobulin antibodies.

Questions:

1. What form of endocrine pathology developed in the patient? Substantiate

your answer.

2. What is the likely cause of this pathology and its pathogenesis?

3. What mechanisms underlie each of the symptoms presented by the

patient?

N 136

A 66-year-old female patient T. has suffered from chronic thyroiditis

complicated by hypothyroid state for 6 years. Two weeks ago she

experienced a severe Salmonella food poisoning. Recently she went to the

marketplace to do the shopping. It was a cold day (-160C), and after 10 min

standing in a queue she felt bad. The bystanders called in an ambulance, and

she was taken to hospital. On examination in the emergency room: the

patient's condition is guarded; her mental status is depressed; the face is

pasty; the hands, forearms, shins and feet appear puffy, but the tissue

rebounds fast after pressing with the finger tip. Blood pressure is 60/40 mm

Page 113: Collected PathPhys Situations 2010

113

Hg; heart rate 55 beats per min; breathing rate 13 per min; body temperature

35.30C; deep tendon reflexes are slowed; abdominal distention is remarkable.

Questions:

1. What pathologic state developed in the patient? Substantiate your answer.

2. What is the cause of this state? What are the main steps of its

pathogenesis?

3. What mechanisms underlie each of the symptoms presented by the

patient?

4. What approaches can be used to treat the patient in this pathologic

condition?

N 137

A 7-year-old boy has been admitted to hospital. On examination: the

patient has signs of physical retardation - his height is below the physiological

norm for the chronologic age group; he has a short neck and extremities, a big

head, a pot belly, a flat nose bridge; the face is pasty and has a flat

expression; he has thick lips and a large protruding tongue; the teeth are

irregularly placed, and many are affected by caries. The patient's speech is

slurred; he pronounces words incorrectly, responds slowly, and has difficulties

in answering the simple questions; he looks capricious and nervous. The

patient's blood pressure is 90/55 mm Hg; heart rate 65 per min. The thyroid

gland has normal size, nontender on palpation. Blood test shows a significant

decrease in T3 and T4 levels. The patient's mother during her pregnancy spent

some time in the area contaminated by radioactive fission products.

Questions:

1. What form of pathology does the patient suffer from? Substantiate your

answer.

2. What are the likely cause of this pathology and its pathogenesis?

3. What mechanisms underlie each of the symptoms presented by the

patient?

PATHOPHYSIOLOGY OF THE NERVOUS SYSTEM. DISORDERS OF LOCOMOTION AND SENSATION Questions to prepare for classes and examination:

Page 114: Collected PathPhys Situations 2010

114

1. General etiology, pathogenesis, and typical forms of pathology of the

nervous system.

2. Disorders of locomotion: manifestations of the upper (central) and

lower (peripheral) motor neuron damage.

3. Disorders of the extrapyramidal system: muscle dystonia and

dyskinesia.

4. Hypokinetic and hyperkinetic movement disorders: types, etiology

and pathogenesis.

5. Disorders of sensation: general etiology and classification.

6. Positive and negative abnormal sensory phenomena.

7. Hypo- and hypersthesia, dysesthesia: types and mechanisms of

disorders.

8. Levels of sensation disorders.

8. Localization of sensory abnormalities.

9. Types and mechanisms of ataxia.

10. Disorders of trophism. Denervation syndrome.

N 138

A 59-year-old patient D., a physician, the day before his admission to

hospital woke up in the morning, got up from bed, and noticed that he had

difficulty in keeping the upright posture, permanently tilting to the left. After he

was assisted in lying down on the bed he felt severe vertigo and nausea.

When he again called for help he noticed the emergence of aphonia (a).

About an hour later he noted emergence and then aggravation of paresthesia

(b) on the right side of the body. When he was eating liquid meal (he could not

eat solid meals because of nausea) regurgitaion (c) occurred.

Neurologic evaluation shows the following: the leftside paresis (d) of the

soft palate; a horizontal nistagmus (e) on lateral gaze, especially manifest

when the patient is looking to the left; the left side facial and truncal

hemihypesthesia (f); muscle hypotonia (g) and hyporeflexia (h) in the left

extremities; finger-to-nose and heel-to-shin tests reveal ataxia (i) and coarse

tremor (j) on the left side. The patient's blood pressure is 195/106 mm Hg; the

area of cardiac dullness is increased to the left by 1.5 cm; heart rate is 90

beats per min.

Page 115: Collected PathPhys Situations 2010

115

Questions:

1. What types of the nervous system pathology developed in the patient?

Substantiate your answer.

2. What is the likely cause of these types of pathology? Localize the foci of

injury in the structures of the nervous system. Substantiate your answer.

3. Explain the meaning of the terms marked by letters.

4. What is the likely cause of the each of the observed symptoms?

N 139

A 30-year-old patient K. experienced trauma of the left thigh with severe

bleeding and supposed injury of the sciatic nerve. Two weeks later he noted

the emergence of prickling and tingling sensation (a) in the shin and the sole

of the injured leg. Then he began to suffer from attacks of spontaneous,

diffuse, hardly tolerable, burning pain (b) that aggravated when the patient

attempted to warm the affected leg. Thus, to soothe the pain he used to

immerse his foot in cold water, but this measure gave only a temporary relief.

Examination of the injured leg 2 months after the trauma shows: the skin of

the foot is pale, dry, and slightly sloughing (c); light touch evokes pain (c);

circumference of the thigh in its middle third is decreased by 4 cm (d)

compared to the intact leg.

Questions:

1. Make a conclusion about the form of the patient's pathology taking into

account the observed symptoms. Substantiate your answer.

2. Name the symptoms marked by letters using the appropriate medical

terms.

3. What are the possible mechanisms of the pain syndrome in this case?

What factors are responsible for the pain in the second week and the second

month after the trauma?

N 140

A 42-year-old patient B. has been admitted to hospital with complaints of

progressive weakness and loss of pain sensation in the right hand after injury;

superficial skin abrasions in this hand usually give rise to persistent

suppuration. These symptoms first appeared 5 years ago. During the last 6

Page 116: Collected PathPhys Situations 2010

116

months the patient noticed difficulty swallowing solid meals; his voice acquired

nasal intonation. Examination by a neuropathologist revealed narrowing of the

right palpebral fissure due to ptosis, a decrease of pain sense in the right face,

drooping of the soft palate, absence of gag and cough reflexes in the pharynx,

signs of vocal cords paresis on the right side, atrophy of the interosseous

muscles and absence of periosteal reflexes in the right hand; segmental

dissociated loss of pain and temperature sensations on the right side; tendon

reflexes in the lower extremities are brisk.

Questions:

1. Make a conclusion about the form of pathology in this patient using the

described symptoms.

2. What are the likely mechanisms of development of the observed

symptoms?

3. At what level of the structural organization of the nervous system could the

observed injury produce: 1) the disorders of sensation, and 2) the locomotor

abnormalities in this patient?

4. Taking into account the course of the disease and the pattern of the

functional deficit, make a presumption about the likely cause of the nervous

system pathology in this patient.

N 141

A 38-year-old female patient K. about a year ago noted the emergence of

shooting pain in the right neck after she had recovered from a viral infection.

Later she noticed the development of numbness (a) in the great toe of the left

foot. This loss of sensation gradually spreaded upward in the course of

several weeks, and reached the level of the clavicle. In addition the patient

began to suffer from progressive weakness of the left leg, and then of the left

hand. Her right hand lost sensitivity to cold and warmth (b).

Neurologic evaluation shows: tenderness of the neck paravertebral zones

on the left side (c); atrophy of the left neck muscles; loss of deep sensation in

the fingers of the left extremities; voluntary movements of the fingers and toes

of the left extremities are limited (d); brisk tendon knee-jerk reflex; the

presence of Babinski's reflex on the left side; column sensory loss (e) of pain

and temperature sensation below C5 level. Cerebrospinal fluid examination

Page 117: Collected PathPhys Situations 2010

117

reveals: protein content 500 mg/L (N: 150-350 mg/L), total cell count 6 per ml

(N: 3-4 per ml), hydrostatic pressure is 150 mm H2O (N: 100-180 mm H2O).

Questions:

1. Make a conclusion about the form of pathology presented by the patient.

Substantiate your answer.

2. Name the symptoms marked by letters using the appropriate medical

terms.

3. What are the likely causes of the locomotion and sensation disorders in this

patient? What pathologic process(es) could cause these disorders?

4. Positive Babinski's reflex characterizes what type of the nervous system

disorder? What other neurologic symptoms are typical for this disorder?

NEUROSES Questions to prepare for classes and examination:

1. Basics of conditioning: conditioned and uncoditioned responses.

2. Failure of the cortex functions as a pathological basis of experimental

neurosis.

3. Types of experimental neurosis: neurosis with domination excitation,

neurosis with domination of inhibition, neurosis due to overexertion of the

lability function.

4. Neurosis in humans: definition of the concept, characteristic features,

relation to experimental neurosis.

5. The main causes, prevalence, and predisposing factors of neurosis

in humans.

6. Pathogenetic models of neuroses: biological, psychosocial, behavioral.

7. The main types of neuroses: neurasthenia, hysteric neurosis,

compulsory neurosis; their characteristic features and pathological

basis.

N 142

A 29-year-old female patient C. has been admitted to hospital with

complaints of poor appetite, depressive mood, episodes of the vision, hearing,

and taste sense losses; occasionally, "loss of voice"; frequent headaches.

Page 118: Collected PathPhys Situations 2010

118

According to her history in her childhood she was nervous, sensitive,

capricious, cry-baby; lacked self-confidence. The symptoms presented on

admission developed during the last year when she encountered serious life

problems related to divorce and the need to find a new accomodation, faced

conflicts at her work.

During her stay in hospital the patient was unduly exacting, required

persistent care of the personnel, blamed the hospital and her doctor for poor

attention to her personality. She occasionally developed vomiting during her

meal or immediately after it, especially when she was being observed by the

medical personnel. When she was taking her physiotherapy procedures

(massage, shower) she lost her consciousness three times. This was

accompanied by a temporary paresis of hands. Somatic status is

unremarkable.

Questions:

1. What form of the nervous system pathology developed in this patient?

Substantiate your answer.

2. What are causes of this pathology and conditions that promoted its

development?

3. What other types of this pathology do you know? Characterize their main

features.

4. Describe mechanisms which may underlie the development of this

pathology.

N 143

A 42-year-old patient N. was brought up by parents who thought that

attaining high social status is the necessary prerequisite of happy life.

Because of his parents' pressure the patient tried hard to excel his classmates

at academic results. He spent a lot of time preparing for lessons though he

had mediocre capabilities and studies came hard to him. After finishing high

school he entered a technical college, since his parents insisted that he

should continue education. His studies at the college required more efforts,

and he spent much time preparing for his classes, sometimes working at

night. When he graduated from the college he found a position of foreman at

the factory. Soon after that a position of the shop superintendent became

Page 119: Collected PathPhys Situations 2010

119

vacant, and he took it though his professional basis and experience was not

sufficient to get along with his new duties. He also lacked experience as a

manager. Soon he met with serious problems. The shop ceased to fulfil the

output programs, and he incurred criticism and reproaches of his bosses and

the staff.

At that time he started to suffer from headaches, painful sensations in the

area of the heart, difficulty in falling asleep, irritability, fatigability, and a

decreased capacity for work; he became over-anxious about his health. The

current complaints are weakness, fatigability, difficulty in concentrating and

falling asleep, dizziness, sweating, painful sensations in the heart area and

skeletal muscles, rapid exhaustion after minimal mental activity, depressive

mood, worry about decreasing mental and bodily well-being, anhedonia. On

examination: blood pressure is 120/65 mm Hg, heart rate 72 beats/min; focal

neurologic symptoms are not found.

Questions:

1. What form of pathology developed in this patient?

2. What is the cause of this pathology in this case?

3. What factors and what personal traits of the patient could facilitate the

development of this disorder?

II. CLINICO-LABORATORY PROBLEMS Normal parameters of the aterial blood gases: Patm. 750-765 mm Hg (atmospheric pressure)

Patm.O2 158-160 mm Hg (partial pressure of O2 in the atmosphere)

Palv.O2 105-110 mm Hg (partial pressure of O2 in the alveolar air)

PaO2 95-100 mm Hg (partial tension of O2 in the arterial blood)

PvO2 45-55 mm Hg (partial tension of O2 in the venous blood)

PaCO2 38-42 mm Hg (partial tension of CO2 in the arterial blood)

SaO2 97-98% (saturation of the arterial blood by O2)

SvO2 60-75% (saturation of the venous blood by O2)

Oxygen-carrying capacity 19.5-21.0 vol%

Pulmonary minute volume 6-8 L/min

Circulation minute volume 5-6 L/min

Page 120: Collected PathPhys Situations 2010

120

Define the type of hypoxia; explain its possible cause(s) and

mechanisms of development:

N 144 N 145

Patm.O2 160 mm Hg Patm.O2 158 mm Hg

Palv.O2 105 mm Hg Palv.O2 105 mm Hg

PaO2 96 mm Hg PaO2 95 mm Hg

PvO2 60 mm Hg PvO2 38 mm Hg

PaCO2 30 mm Hg PaCO2 40 mm Hg

SaO2 98% MetHb 40%

SvO2 91% SaO2 70%

Pulmonary min. SvO2 20%

volume 7.3 L/min Pulmonary min.

Circulation min. volume 8.8 L/min

volume 6.9 L/min Circulation min.

pH 7.31 volume 7.0 L/min

Lactic acid 26.5 mg/dl pH 7.30

Lactic acid 20.5 mg/dl

N 146 N 147

Patm.O2 150 mm Hg Patm.O2 158 mm Hg

Palv.O2 94 mm Hg Palv.O2 105 mm Hg

PaO2 76 mm Hg PaO2 96 mm Hg

PvO2 21 mm Hg PvO2 18 mm Hg

PaCO2 48 mm Hg PaCO2 28 mm Hg

SaO2 90% Hb 140 g/L

SvO2 32% SaO2 95%

Pulmonary min. SvO2 27%

volume 4.6 L/min Pulmonary min.

Circulation min. volume 8.8 L/min

volume 6.4 L/min Circulation min.

pH 7.31 volume 2.9 L/min

Lactic acid 25 mg/dl pH 7.31

Page 121: Collected PathPhys Situations 2010

121

Lactic acid 26.5 mg/dl

N 148 N 149*

Patm.O2 105 mm Hg Patm.O2 158 mm Hg

Palv.O2 55 mm Hg Palv.O2 88 mm Hg

PaO2 40 mm Hg PaO2 61 mm Hg

PvO2 12 mm Hg PvO2 16 mm Hg

PaCO2 58 mm Hg PaCO2 59 mm Hg

SaO2 67% SaO2 88%

SvO2 11% SvO2 25%

Pulmonary min. Pulmonary min.

volume 4.5 L/min volume 2.85 L/min

Circulation min. Circulation min.

volume 3.4 L/min volume 8.50 L/min

pH 7.28 pH 7.25

Titratable acidity of the

daily urine 60 mmol/d

Hb 140 g/L

Normal parameters of the acid-base homeostasis of the capillary blood:

Parameter

pH

PCO2

SB

BB

BE

Ketone bodies

Lactic acid

Titratable

acidity (of the daily

urine)

NH4+ (of the daily

urine)

Value

7.35-7.45

36-45 mm Hg

21-25 mmol/L

45-52 mmol/L

+/- 2.5 mmol/L

0.5-2.5 mg/dl

6-16 mg/dl

20-40 mmol/d

30-50 mmol/d

Page 122: Collected PathPhys Situations 2010

122

Define the type of acid-base disorder and explain its possible mechanism

of development:

N 150 N 151

pH 7.18 pH 7.25

PCO2 75 mm Hg PCO2 52 mm Hg

SB 27 mmol/L SB 22 mmol/L

BE +2.5 mmol/L BB 45 mmol/L

________________________ BE +2 mmol/L

The patient is undergoing __________________________

surgical operation with The patient has a bronchial

cardiopulmonary bypass asthma attack

N 152* N 153

pH 7.20 pH 7.36

PCO2 37 mm Hg pCO2 36 mm Hg

SB 14 mmol/L SB 19,5 mmol/L

BB 29 mmol/L BB 39 mmol/L

BE -12 mmol/L BE -5 mmol/L

____________________ Ketone bodies 7.5 mg/dl

The patient is undergoing Titratable acidity 37 mmol/d

surgical operation with _________________________

cardiopulmonary bypass The patient suffers from diabetes

mellitus

N 154 N 155

pH 7.30 pH 7.30

pCO2 38 mm Hg pCO2 35 mm Hg

SB 18 mmol/L SB 16.5 mmol/L

BB 36 mmol/L BB 35 mmol/L

BE -6 mmol/L BE -9 mmol/L

Lactic acid 26 mg/dl Titratable acidity 8 mmol/d

Titratable acidity 45 mmol/d NH4+ 17 mmol/d

________________________ __________________________

The patient is admitted to The patient suffers from

Page 123: Collected PathPhys Situations 2010

123

hospital with a preliminary "acute glomerulonephritis"

diagnosis of "acute myocardial

infarction"

N 156 N 157

pH 7.22 pH 7.22

pCO2 36 mm Hg pCO2 47 mm Hg

SB 14 mmol/L SB 18.5 mmol/L

BB 24 mmol/L BB 40.5 mmol/L

BE -8 mmol/L BE -7 mmol/L

____________________ Lactic acid 28 mg/dL

The patient has an Titratable acidity 8 mmol/d

intestinal fistula; NH4+ 15 mmol/d

he has been losing ________________________

intestinal juice for a The patient suffers from

long period of time severe toxic injury of liver

and kidneys with oligouria

N 158 N 159

pH 7.11 pH 7.17

pCO2 50 mm Hg pCO2 51 mm Hg

SB 15.5 mmol/L SB 18 mmol/L

BE -13 mol/L BB 45 mmol/L

BB 38 mmol/L BE -8 mmol/L

Ketone bodies 58 mg/dL _____________________

Titratable acidity 70 mmol/d The patient suffers from

___________________________ acute left ventricular

The patient has coma insufficiency with

lungs edema

N 160 N 161

pH 7.52 pH 7.51

PCO2 25 mm Hg PCO2 36 mm Hg

SB 20 mmol/L SB 28 mmol/L

BE -2 mmol/L BB 57 mmol/L

Page 124: Collected PathPhys Situations 2010

124

BB 40 mmol/L BE +5,5 mmol/L

___________________________ _________________________

The patient is undergoing The patient suffers from

surgery with mechanical uncontrolable vomiting, tetanus

ventilation

N 162 N 163

pH 7.59 pH 7.27

PCO2 30 mm Hg PCO2 25 mm Hg

SB 28 mmol/L SB 11 mmol/L

BE +5 mmol/L BB 27 mmol/L

BB 50 mmol/L BE -17 mmol/L

_______________________ Hematokrit 0.36

The patient with brain __________________________

contusion has cyclic The patient had an acute blood

vomiting and dyspnea loss

N 164 N 165

pH 7.22 pH 7.33

PCO2 49 mm Hg PCO2 35 mm Hg

SB 19.5 mmol/L SB 18 mmol/L

BE -3 mmol/L BB 42 mmol/L

BB 44 mmol/L BE -5,5 mmol/L

Lactic acid 23 mg/dL Titratable acidity 12 mmol/L

Titratable acidity 15 mmol/d Hematokrit 0.3

_________________________ __________________________

The patient has been transported The patient has been admitted to

to the intensive care unit after hospital with a diagnosis of

surgical removal of thrombus hemolytic anemia of unclear

in the abdominal aorta etiology

N 166 N 167

pH 7.57 pH 7.45

pCO2 28 mm Hg pCO2 30 mm Hg

SB 25 mmol/L SB 20 mmol/L

Page 125: Collected PathPhys Situations 2010

125

BE +4 mmol/L BB 50.5 mmol/L

BB 48 mmol/L BE -0.5 mmol/L

Lactic acid 12 mg/dL Hematokrit 0.4

Titratable acidity 20 mmol/d ________________________

________________________ The patient suffers from

The patient has had the hysteria brain contusion and bouts

hyperventilation episode just of vomiting

before the analysis

N 168 N 169

pH 7.09 pH 7.36

pCO2 51 mm Hg pCO2 49 mm Hg

SB 15 mmol/L SB 26,5 mmol/L

BE -12,5 mmol/L BB 52,5 mmol/L

BB 38 mmol/L BE +6 mmol/L

Lactic acid 25 mg/dL Titratable acidity 18 mmol/d

Titratable acidity 18 mmol/d ________________________

_______________________ The patient was admitted to

The patient suffers from purulent hospital 9 days before with

peritonitis a diagnosis of acute

myocardial infarction

Normal results of the complete blood test with differential:

Hemoglobin (Hb) 130-160 g/L (men)

120-140 g/L (women)

Erythrocytes 4.0-5.0*1012/L (men)

3.9-4.7*1012/L (women)

Mean corpuscular Hb (MCH) 25-33 pg

Reticulocytes 0.3-1.2%

Platelets 180-320*1012/L

Leukocytes 4.0-9.0*109/L

Neutrophils:

myelocytes 0

metamyelocytes 0

Page 126: Collected PathPhys Situations 2010

126

band 1-6%

segmented 47-72%

Eosinophils 0.5-5.0%

Basophils 0-1%

Lymphocytes/plasmocytes 19-37%/0

Monocytes 3-11%

Erythrocytes sedimentation rate (ESR) 2-10 mm/h (men)

2-15 mm/h (women)

Hematokrit (Ht) 0.40-0.48 (men)

0.36-0.42 (women)

Characterize changes in complete blood tests with differential, and indicate

their possible causes, mechanisms of development and consequences. Make

a general conclusion on the results of tests.

Symbol “ - “ indicates that the referred parameter has not been measured.

Symbol “ ? “ indicates that the referred parameter should be calculated.

N 170 N 171

Hb 150 g/L 112 g/L

Erythrocytes 4.7*1012/L 4.1*1012/L

MCH ? ?

Reticulocytes 0.5% 0.2%

Platelets 220*109/L 130*109/L

Leukocytes 17.0*109/L 21.5*109/L

Neutrophils:

myelocytes 0 0

metamyelocytes 4% 0

band 16% 0

segmented 60% 3%

Eosinophils 1% 0

Basophils 0 0

Lymphocytes 15% 89%

Monocytes 4% 8%

Few lymphoblasts

are present

Page 127: Collected PathPhys Situations 2010

127

N 172 N 173

Hb 34 g/L 80 g/L

Erythrocytes 1.0*1012/L 3.0*1012/L

MCH ? ?

Reticulocytes 0% 0%

Platelets few 85*109/L

Leukocytes 11.3*109/L 0.9*109/L

Neutrophils: 23%

myelocytes 0

metamyelocytes 0

band 2%

segmented 10%

Eosinophils 0% 10%

Basophils 0 0

Lymphocytes 78% 55%

Lymphoblasts 7%

Monocytes 3% 12%

ESR 68 mm/h

* **

* Anisocytosis and microcytosis of erythrocytes.

**High blood levels of IgG, IgM, and factors of complement; increased

blood concentration of unconjugated bilirubin.

N 174 N 175

Hb 18 g/L 162 g/L

Erythrocytes 0.8*1012/L 4.7*1012/L

MCH ? ?

Reticulocytes 0.3% 1.0%

Platelets 180*109/L 250*109/L

Leukocytes 88.0*109/L 39.0*109/L

Promyelocytes 0 3%

Neutrophils:

myelocytes 0 3%

Page 128: Collected PathPhys Situations 2010

128

metamyelocytes 0 10.5%

band 1% 19%

segmented 2% 25.5%

Eosinophils 1% 0

Basophils 0 0

Lymphocytes 60% 27%

Lymphoblasts 35% 0

Monocytes 1% 12%

N 176 N 177

Hb 100 g/L 95 g/L

Erythrocytes 3.6*1012/L 3.7*1012/L

MCH ? ?

Reticulocytes 0.6% 3%

Platelets 200*109/L 110*109/L

Leukocytes 36.0*109/L 2.9*109/L

Myeloblasts 0 33%

Promyelocytes 0 1%

Neutrophils:

myelocytes 0 0

metamyelocytes 6% 0

band 9% 2%

segmented 11% 10%

Eosinophils 60% 20%

Basophils 0 0

Lymphocytes 12% 32%

Monocytes 2% 2%

N 178 N 179

Hb 190 g/L 170 g/L

Erythrocytes 7.4*1012/L 5.3*1012/L

MCH ? ?

Reticulocytes 1.6% 0.5%

Platelets 550*109/L 480*109/L

Page 129: Collected PathPhys Situations 2010

129

Leukocytes 11.0*109/L 16.5*109/L

Myeloblasts 0 33%

Promyelocytes 0 1%

Neutrophils:

myelocytes 0 0

metamyelocytes 0.5% 3%

band 16% 16%

segmented 56% 70%

Eosinophils 11% 1%

Basophils 1.5% 0

Lymphocytes 12% 3%

Monocytes 3% 7%

ESR 10 mm/h 1 mm/h

* **

* Significant polychromatophylia of erythrocytes, few normoblasts.

**Anisocytosis and macrocytosis of erythrocytes.

N 180* N 181

Hb 82 g/L 64 g/L

Erythrocytes 3.02*1012/L 2.0*1012/L

MCH ? ?

Reticulocytes 0 0

Platelets 0.15*109/L 120*109/L

Leukocytes 0.5*109/L 84.0*109/L

Neutrophils 0 2%

Eosinophils 10% 0

Basophils 0 0

Lymphocytes 78% 2.5%

Lymphoblasts 0 90.5%

(or myeloblasts?)

Monocytes 12% 5%

* **

* Anisocytosis and microcytosis of erythrocytes.

**Positive reaction for myeloperoxidase.

Page 130: Collected PathPhys Situations 2010

130

N 182 N 183

Hb 168 g/L 34 g/L

Erythrocytes 4.8*1012/L 1.08*1012/L

MCH ? ?

Reticulocytes 0.1%

Platelets 240*109/L few

Leukocytes 41.0*109/L 1.3*109/L

Promyelocytes 2% 0

Neutrophils:

myelocytes 5.5% 0

metamyelocytes 28% 0

band 20.5% 2%

segmented 26.5% 10%

Eosinophils 0 20%

Basophils 0 0

Lymphocytes 6% 50%

Lymphoblasts 0 16%

Monocytes 11.5% 2%

ESR - 80 mm/h

N 184 N 185

Hb 76 g/L 99 g/L

Erythrocytes 2.8*1012/L 3.8*1012/L

MCH ? ?

Reticulocytes 0.1% 0

Platelets 0.8*109/L 5.0*109/L

Leukocytes 128*109/L 2.8*109/L

Myeloblasts 97% 31%

Promyelocytes 0.5% 1%

Neutrophils:

myelocytes 0 0

metamyelocytes 0 0

band 0 4%

Page 131: Collected PathPhys Situations 2010

131

segmented 2.5% 10%

Eosinophils 0 22%

Basophils 0 0

Lymphocytes 0 30%

Monocytes 0 2%

N 186 N 187

Hb 58 g/L 62 g/L

Erythrocytes 3.1*1012/L 3.9*1012/L

MCH ? ?

Leukocytes 182*109/L 67.5*109/L

Myeloblasts 4% 0

Promyelocytes 12% 10%

Neutrophils:

myelocytes 4.5% 4.5%

metamyelocytes 10% 6%

band 8% 39%

segmented 37.5% 22%

Eosinophils 5% 0.5%

Basophils 9% 0

Lymphocytes 0 14.5%

Monocytes 0 3.5%

N 188 N 189

Hb 98 g/L 108 g/L

Erythrocytes 3.9*1012/L 4.0*1012/L

MCH ? ?

Platelets 200*109/L 490*109/L

Leukocytes 38.0*109/L 23.9*109/L

Promyelocytes 0 2.5%

Neutrophils:

myelocytes 0 4%

metamyelocytes 3% 6%

band 14% 14%

Page 132: Collected PathPhys Situations 2010

132

segmented 62% 43%

Eosinophils 8% 4.5%

Basophils 0 3%

Lymphocytes 12% 18%

Monocytes 1% 5%

N 190 N 191

Hb 115 g/L 80 g/L

Erythrocytes 3.5*1012/L 2.9*1012/L

MCH ? ?

Reticulocytes 0.4% 0.1%

Leukocytes 14.8*109/L 38*109/L

Promyelocytes 0 2%

Neutrophils:

myelocytes 0 6%

metamyelocytes 0 10%

band 3.5% 17%

segmented 42% 51%

Eosinophils 0.5% 0.5%

Basophils 0 0

Lymphocytes 43% 12%

Monocytes 11% 1.5%

N 192 N 193

Hb 128 g/L 118 g/L

Erythrocytes 4.1*1012/L 3.6*1012/L

Ht 0.38

MCH ? ?

Reticulocytes 0.4% 0.3%

Leukocytes 42*109/L 3.8*109/L

Promyelocytes 2% 0

Neutrophils:

myelocytes 3% 0

metamyelocytes 9% 0

Page 133: Collected PathPhys Situations 2010

133

band 14% 4%

segmented 44% 56%

Eosinophils 4.5% 4.5%

Basophils 1.5% 1%

Lymphocytes 20% 30%

Monocytes 2% 4.5%

Normal parameters of bilirubin pigments metabolism Peripheral blood: Total bilirubin < 1.3 mg/dl

Unconjugated bilirubin < 1.0 mg/dl

Conjugated bilirubin < 0.3 mg/dl

Urobilinogen not detectable

Stercobilinogen detectable

Define changes in bile pigments level and composition in biological fluids

and explain possible causes and mechanisms of their development. Make a

general conclusion on the given data.

N 194* blood: urine: stool:

Total bilirubine 2.6 mg/dl - -

Conjugated bilirubine 0.15 mg/dl not found -

Urobilinogen + + -

Stercobilinogen increased increased increased

Cholic acids not found not found -

N 195

blood: urine: stool:

Total bilirubine 2.4 mg/dl - -

Conjugated bilirubine 1.4 mg/dl + -

Urobilinogen trace amounts trace amounts -

Stercobilinogen trace amounts trace amounts very low

Cholic acids + + -

Page 134: Collected PathPhys Situations 2010

134

Alanine aminotransferase increased - -

Aspartate aminotransferase increased - -

N 196

blood urine: stool:

Total bilirubine 1.5 mg/dl - -

Conjugated bilirubine 0.5 mg/dl trace amounts -

Urobilinogen + + -

Stercobilinogen norm norm -

Cholic acids not found not found -

N 197

blood: urine: stool:

Total bilirubine 2.43 mg/dl - -

Conjugated bilirubine 0.35 mg/dl trace amounts -

Urobilinogen not found not found -

Stercobilinogen decreased decreased decreased

Cholic acids + + -

Alanine aminotansferase very high - -

Aspartate aminotransferase very high - -

N 198

blood: urine: stool:

Total bilirubine 2.4 mg/dl - -

Conjugated bilirubine 1.4 mg/dl + -

Urobilinogen not found not found -

Stercobilinogen not found not found not found

Cholic acids + + -

Alanine aminotransferase norm - -

Aspartate aminotransferase norm - -

N 199

blood: urine: stool:

Total bilirubine 1.8 mg/dl - -

Page 135: Collected PathPhys Situations 2010

135

Conjugated bilirubine 0.2 mg/dl + -

Urobilinogen not found not found -

Stercobilinogen norm norm norm

Cholic acids not found not found -

N 200

blood: urine: stool:

Total bilirubine 3.5 mg/dl - -

Conjugated bilirubine 2.1mg/dl + -

Urobilinogen not found not found -

Stercobilinogen trace amounts trace amounts decreased

Cholic acids + + -

Alanine aminotransferase increased - -

N 201

blood: urine: stool:

Total bilirubine 4.4 mg/dl - -

Conjugated bilirubine 0.35 mg/dl trace amounts -

Urobilinogen + + -

Stercobilinogen increased increased norm

Cholic acids not found not found -

Alanine aminotransferase increased - -

N 202

blood urine: stool:

Total bilirubine 2.1 mg/dl - -

Conjugated bilirubine 0.2 mg/dl not found -

Urobilinogen not found not found -

Stercobilinogen increased increased increased

Cholic acids not found not found -

Alanine aminotransferase norm - -

N 203

blood: urine: stool:

Page 136: Collected PathPhys Situations 2010

136

Total bilirubine 3.6 mg/dl - -

Conjugated bilirubine 2.5 mg/dl + -

Urobilinogen not found not found -

Stercobilinogen not found not found trace amount

Cholic acids not found not found -

Alanine aminotransferase norm - -

Normal parameters of renal function: Diuresis 800-1200 ml/d

Specific gravity 1.018-1.025

Protein absent

Glucose absent

Ketone bodies absent

Microscopy of the sediment

Erythrocytes: few per high power field (h.p.f.)

Leukocytes: few per h.p.f.

Hyaline casts: few in the sample.

Additional data

Glucose of blood plasma 65-110 mg/dl

N 204* Diuresis 800 ml

Specific gravity 1029

Protein 1 g/l

Glucose not found

Ketone bodies not found

Microscopy: 40-50 erythrocytes per high power field; few hyaline and

red blood cell casts.

Additional data: blood pressure 165/105 mm Hg;

blood urea nitrogen 50 mg/dl (N:20-40 mg/dl).

High titers of antistreptolysin O in blood plasma.

N 205

Diuresis 2800 ml

Page 137: Collected PathPhys Situations 2010

137

Specific gravity 1009

Protein 2 g/l

Glucose not found

Ketone bodies not found

Microscopy: few erythrocytes per high power field;

hyaline casts.

Additional data: blood pressure 185/100 mm Hg;

blood urea nitrogen 80 mg/dl (N:20-40 mg/dl).

N 206

Diuresis 420 ml

Specific gravity 1011

Protein 2 g/l

Glucose not found

Ketone bodies not found

Microscopy: few erythrocytes per high power field;

hyaline, waxy, granular casts.

Additional data: blood pressure 175/95 mm Hg;

blood urea nitrogen 190 mg/dl (N:20-40 mg/dl).

N 207

Diuresis 1000 ml

Specific gravity 1037

Protein 33 g/l

Glucose not found

Ketone bodies not found

Microscopy: large amount of granular and waxy casts.

Additional data: blood pressure 120/65 mm Hg;

blood urea nitrogen 35 mg/dl (N:20-40 mg/dl).

N 208

Diuresis 1600 ml

Specific gravity 1025

Protein not found

Page 138: Collected PathPhys Situations 2010

138

Glucose 2,5%

Ketone bodies not found

Blood glucose 80 mg/dl

N 209

Diuresis 5500 ml

Specific gravity 1040

Protein not found

Glucose 4%

Ketone bodies positive reaction

Blood glucose 300 mg/dl (N:80-120 mg/dl)

N 210

Diuresis 340 ml

Specific gravity 1035

Protein 1,5 g/l

Glucose not found

Ketone bodies not found

Microscopy: 30-35 erythrocytes per high power field;

granular and erythrocyte casts 2-3 per h.p.f.

Additional data: blood pressure 145/100 mm Hg;

blood urea nitrogen 210 mg/dl (N:20-40 mg/dl)

N 211

Diuresis 165 ml

Specific gravity 1015

Protein 3,5 g/l

Glucose not found

Ketone bodies not found

Microscopy: few erythrocytes; hyaline casts.

Additional data: blood pressure 145/90 mm Hg;

blood urea nitrogen 48 mg/dl (N:20-40 mg/dl).

N 212

Page 139: Collected PathPhys Situations 2010

139

Diuresis 3800 ml

Specific gravity 1014

Protein not found

Glucose not found

Ketone bodies not found

Microscopy: few hyaline casts per h.p.f.

Additional data: blood pressure 115/80 mm Hg;

blood urea nitrogen 35 mg/dl (N:20-40 mg/dl).

N 213 Diuresis 1600 ml

Specific gravity 1020

Protein not found

Glucose 1,5%

Ketone bodies not found

Titratable acidity 5 mmol/L

Microscopy: large amounts of salt cristals.

Additional data: positive reaction for cystein and arginine in the urine,

high concentration of phosphates.

III. EXAMPLES OF CASES’ DISCUSSION N 93

A 40-year-old patient Z. suffers from tuberculosis. He has come to see his

physician with complaints of dyspnea, pains in the right upper quadrant, and

subfebrile fever. These symptoms emerged 4 weeks ago and were

progressing. On examination: the face is pale and rounded, orthopnea is

evident (the patient is sitting upright with his hands resting upon chair; this

posture is necessary to ease breathing); the area of cardiac dullness is

increased to the left and to the right by 2 cm, heart rate is 100 per min, blood

pressure is 90/60 mm Hg. On auscultation: muffled heart sounds, crackles at

the base of the lungs. Breathing rate is 26 per min. The neck veins are

distended; the liver is tender on palpation, and extends 3 cm below the costal

margin; there is trace pretibial and feet edema. X-ray examination of the chest

shows the globular shadow of the heart.

Page 140: Collected PathPhys Situations 2010

140

Questions:

1. Does the patient have signs of cardiac insufficiency? Substantiate your

answer.

2. What additional tests are required to specify the form of the heart pathology

in this case?

3. Make a conclusion about the form of cardiac insufficiency in this patient.

Answers:

1. The patient has cardiac insufficiency manifested by orthopnea, tachycardia,

rales in the lungs, distention of the neck veins, enlargement of the liver,

edema of the lower extremities.

2. Ultrasound investigation may show the presence of fluid in the pericardial

cavity.

3. Total diastolic heart insufficiency caused by tuberculous pericarditis which

complicates tuberculosis in this patient. Fluid in the pericardial cavity

compresses the heart and disturbs filling of ventricles.

N 107

A 36-year-old patient K., a mine worker, has been admitted to hospital with

a preliminary diagnosis of silicosis. He complains of dyspnea, more severe on

walking and physical exercise; sustained cough (occasionally with secretions),

chest pain.

_____________________________________________________

Arterial blood gases:

PaO2 90 mm Hg

PaCO2 40 mm Hg

Oxygen carrying capacity 19.2 vol%

SaO2 94.3%

Spirometry:

Forced vital capacity (FVC) 4.2 L

FVC in % to normal 92%

Forced expiratory volume in 1 s (FEV1) 2.6 L

Tiffeneau ratio (FEV1/FVC) ?

Pulmonary minute volume (% of normal) 124%

Additional data:

Page 141: Collected PathPhys Situations 2010

141

Breathing rate 19/min

_________________________________________

The voluntary hyperventilation test yields the PaO2 value of 92 mm Hg.

Questions:

1. Does the patient have disturbances of respiratory function? Describe the

signs of it, if he does.

2. Does the patient have disturbances of alveolar ventilation? If he does,

define its type (obstructive or restrictive).

3. Taking into account the preliminary diagnosis of pneumoconiosis how to

test the diffusive capacity of the lungs in this patient?

4. Make a final conclusion about the type of respiratory system disorder in

patient K.

Answers:

1. Symptoms of respiratory disorder in this patient include dyspnea,

increased breathing rate (BR) and pulmonary minute volume (PMV),

hypoxemia, decreased Tiffeneau ratio, lack of significant changes in PaO2

after voluntary hyperventilation.

2. The patient has signs of disturbance of alveolar ventilation mainly of

obstructive type. The former is confirmed by a decrease in the Tiffeneau ratio

(< 62%) and increased BR and PMV values.

3. The diffusive capacity of the lungs may be evaluated by the voluntary

hyperventilation test. In this case PaO2 (92 mm Hg) shows little changes

during the test as compared with the initial values. Hence, the diffusive

capacity of the lungs for O2 is decreased.

4. General conclusion: the patient has a disorder of respiratory function due to

alveolar hypoventilation of the obstructive type (presumably, as a result of

obstruction of the airways by mucous secretions), and due to a decrease of

the diffusive capacity of the aerohematic barrier .

N 120

A victim of the vehicle accident was transported to hospital 5 hours after

the accident. He was examined by a physician of the emergency service who

found multiple ribs fractures, contusions of soft tissues of the pelvis and lower

extremities with extensive hemorrhages. On admission the patient is

Page 142: Collected PathPhys Situations 2010

142

confused, markedly pale; he has a thready pulse, blood pressure 60/20 mm

Hg, periodic pattern of breathing. After a day of intense treatment by volume

expanders (he received 3 L of polyglukin) and infusion of 0.5 L of blood his

blood pressure increased to 110/60 mm Hg. On the next day after infusion

therapy diuresis was still absent, and during the following three days the

patient's condition remained guarded. He complained of severe headache,

dizziness, cyclic vomiting, inhibited state. The patient had short-termed

episodes of seizures, edema of the skin; bradycardia and extrasystole on

ECG records; his diuresis was at the level of 150-250 ml/day, but blood

pressure increased to 160/90 mm Hg. Blood test shows: BUN (blood urea

nitrogen) 90 mg/dl (N: 20-40 mg/dl), hyperkalemia, hypermagnesimea,

hyponatremia, hypochloremia, pH 7.30. Urine test: specific gravity 1.040, mild

proteinuria, myoglobinuria; sediment shows the presence of casts and few

leukocytes in the h.p.f.

On the 5-7th day in the hospital the patient developed great elevation of

diuresis (up to 2500 ml/day), and his condition markedly improved. Vomiting,

seizures and headache ceased, and the extent of edema reduced. Repeated

urinalysis shows: specific gravity 1.010-1.012, slight proteinuria, large quantity

of granular casts in the urine sediment.

Questions:

1. Define the type of renal syndrome in this patient. What are its causes?

2. What are the causes of anuria during the shock before normalization of

blood pressure in this patient?

3. Why was diuresis not restored despite extensive transfusion theraspy?

4. What are the mechanisms of symptoms observed on the 2-4th day after the

trauma?

Answers.

1. Acute renal failure results from traumatic shock and occlusion of renal

tubules by myoglobin (myoglobin-induced nephrosis).

2. Low blood pressure in the renal glomeruli. At blood pressure levels below

70 mm Hg renal blood flow decreases by 90% or more, and filtration pressure

in the glomerular capillaries becomes negligible.

3. Renal ischemia caused by shock and obturation of renal tubules by

myoglobin leads to detachment of tubular epithelia and blocking of the

Page 143: Collected PathPhys Situations 2010

143

tubules' lumen. The latter increases the interstitial pressure in the medulla and

may further compress the tubules. High pressure of the primary filtrate

reduces the efficiency of glomerular filtration. Combination of these factors

precludes the normalization of diuresis.

4. Severe headache, dizziness, frequent ( intractable) vomiting, psychomotor

retardation suggest the development of brain edema caused by retention of

water in the body. Brain edema is also promoted by increased arterial

pressure. Increased levels of blood urea nitrogen, hyperkalemia and

hypermagnesimea result from a decreased rate of glomerular filtration and

impairment of tubular reabsorption. Decreased blood levels of sodium and

chloride are caused by intractable vomiting that could have led to

development of hypochloremic alkalosis. However, disorders of the tubular

function result in impairment of acidogenesis and ammoniogenesis leading to

renal excretory acidosis. The following increase in daily urine output and low

specific gravity of the urine suggest a disturbance of the kidney concentrating

ability.

N 132

An endocrinologist observes two patients during their follow-up visits: a 50-

year-old woman M. and her 26-year-old daughter D. Both patients have a

significant enlargement of their thyroid gland and clinical signs of

thyrotoxicosis. The results of their clinical and laboratory investigations

suggest the diagnosis of the Grave’s disease. The patient D.’s condition

improved after a treatment course, while the patient M.’s condition kept

worsening after a 8-months course of medication by thyrostatic drugs. During

reexamination M. presented with new complaints of sluggishness, motor

retardation, drowsiness during the day time and difficulty going to sleep at

night, memory problems, decreased working ability, edema of the face and

extremities and cold intolerance. These symptoms emerged after M. had

recovered from viral infection. The physician suspected the presence of

Hashimoto’s thyroiditis in M., and changed the treatment.

Questions:

1. Can we consider the emergence of new symptoms in M. as complication of

the thyrostatic therapy?

Page 144: Collected PathPhys Situations 2010

144

2. What laboratory tests can be made to confirm the diagnosis in M.?

3. What other clinical forms of thyroid gland pathology should be considered

to make the final diagnosis?

4. Are there common mechanisms in the pathogenesis of Grave’s disease

and Hashimoto’s thyroiditis?

Answers.

1. Thyrotoxicosis is characterized by:

- cardiovascular disorders,

- disorders of the central nervous and autonomous system,

- activation of catabolic processes that manifest in wasting, subfebrile

body temperature, myopathy, and osteochondrosis,

- gastrointestinal disorders,

- disorders of endocrine glands, such as adrenal insufficiency, ovarian

dysfunction, fibrocystic breast disease.

Inappropriate use of thyrostatic drugs may induce hypothyroidism.

2. A more accurate diagnosis may be obtained by determination of

T3,T4 and TSH levels in blood, immunological tests (antithyroglobulin and

antithyroid peroxidase antibodies, antibodies against the TSH receptor),

needle aspiration of the thyroid gland (it may reveal the histologic signs of

the autoimmune thyroiditis).

3. Differential diagnosis may include acute and subacute thyroiditis.

4. The common step is the involvement of immunologic mechanisms.

Antithyroglobulin and antimicrosomal antibodies as well as

thyroid-stimulating immunoglobulins have been found in plasma of patients

with Hashimoto’s thyroiditis. These antibodies form immune complexes with

the relevant antigens and cause destruction of thyrocytes. At the

initial stage injury to the thyroid gland may release high quantities of thyroid

hormones in the blood producing thyrotoxicosis. Later, with progressive

destruction and lymphocytic infiltration of follicles the hyperfunctional state

transfers into the functional insufficiency with a characteristic clinical

picture.

N 149

Define the type of hypoxia and explain its possible cause(s) and

Page 145: Collected PathPhys Situations 2010

145

mechanisms of development:

Patm.O2 158 mm Hg

Palv.O2 88 mm Hg

PaO2 61 mm Hg

PvO2 16 mm Hg

PaCO2 59 mm Hg

SaO2 88%

SvO2 25%

Pulmonary

minute volume 2.85 L/min (N: 6-8 L)

Circulation

minute volume 8.50 L/min (N: 5-6 L)

pH 7.25

Lactic acid 20.0 mg/dl (N: 6-16 mg/dl)

Titratable acidity of

the daily urine 60 mmol/d

Hb 140 g/L

Conclusion.

Complex respiratory and circulatory type of hypoxia. The respiratory type is

due to hypoventilation because PaO2 is decreased and PaCO2 is increased l

while pulmonary minute volume is low. The circulatory type is confirmed by

high arterial-venous difference in O2 : SaO2 - SvO2. A decrease in pH is

caused by accumulation of lactic acid and H2CO3 in blood. Renal function,

judging by the ability to excrete H+ (titratable acidity), is normal.

N 152

Define the type of acid-base disorder and explain its possible

mechanisms.

pH 7.20

pCO2 37 mm Hg

SB 14 mm/L

BB 29 mm/L

BE -12 mm/L

The patient is undergoing surgical operation with cardiopulmonary bypass

Page 146: Collected PathPhys Situations 2010

146

Conclusion

The patient has an uncompensated acidosis (pH level is below 7.35). This

acidosis is not respiratory since there is no increase in pCO2 and SB.

Decrease in SB suggests the accumulation of nonvolatile acids in blood

plasma. Taking into account the described circumstances we may assume the

presence of metabolic acidosis caused by circulatory insufficiency.

N 180

Characterize changes in blood test and explain their possible causes,

mechanisms of development and consequences. Make a general conclusion

on the given data.

Hb 82 g/L

Erythrocytes 3.02*1012/L

MCH to be calculated

Reticulocytes 0

Platelets 0.15*109/L

Leukocytes 0.5*109/L

Neutrophils 0

Eosinophils 10%

Basophils 0

Lymphocytes 78%

Monocytes 12%

* Anisocytosis and microcytosis of erythrocytes.

Conclusion.

There are signs of hypochromatic aregeneratory anemia of the

normoblastic type; thrombocytopenia, leukopenia; relative eosinophylia, but

the absolute content of eosinophils is within the normal range; relative

lymphocytosis and absolute lymphopenia; relative monocytosis and absolute

monocytopenia. Aplastic anemia may be preliminary diagnosed since there is

evidence of impairment of all the hematopietic cell lines.

N 194

Page 147: Collected PathPhys Situations 2010

147

Define changes in bile pigments content and composition in biological

fluids and explain their possible causes and mechanisms of development.

Make a general conclusion on the given data.

blood: urine: stool:

Total bilirubine 2.6 mg/dl - -

Conjugated bilirubine 0.15 mg/dl not found -

Urobilinogen + + -

Stercobilinogen increased increased increased

Cholic acids not found not found -

Conclusion.

The patient has increased levels of total blood bilirubine owing to

unconjugated fraction (2.60 - 0.15 = 2.45 mg/dl). The increased content of

stercobilinogen in feces and urine suggests the increased release of

conjugated bilirubine in the small intestine. These data suggest that elevation

of unconjugated bilirubine in blood is caused by its increased production due

to hemolysis of erythrocytes. Elevation of urobilinogen levels in blood and

urine results from its enhanced absorption in the intestine and failure of

hepatocytes to completely remove it from the blood due to saturation of the

membrane transporter.

General conclusion: the patient has hemolytic jaundice.

N 204

Define changes in urinalysis and explain their possible causes and

mechanisms of development. Make a general conclusion on the given data.

Diuresis 800 ml

Specific gravity 1.029

Protein 1 g/L

Glucose not found

Ketone bodies not found

Microscopy of the urine sediment:

Erythrocytes 40-50 per high power field, few hyaline and red blood cell

casts.

Additional data:

Blood pressure 165/105 mm Hg

Page 148: Collected PathPhys Situations 2010

148

BUN 50 mg/dl

High titers of antistreptolysin O in blood plasma.

Conclusion:

1. Oligouria.

2. Hypersthenuria.

3. Proteinuria.

4. Hematuria.

5. Significant excess of hyaline and red blood cell casts.

6. Arterial hypertension.

7. Azotemia.

8. The presence of antistreptolysin O in blood.

The patient has signs of nephritic syndrome. Hypersthenuria suggests that

tubular function is not affected. Oligouria, hematuria, arterial hypertension and

azotemia point to the presence of injury to the renal glomeruli. This pathologic

process is supposed to be acute, since the chronic process is characterized

by injury to all parts of the nephron.

General conclusion: acute diffuse glomerulonephritis caused by

streptococcal infection; renal arterial hypertension.